ATT_1441627983757_Remedial Law part three.pdf

July 21, 2018 | Author: kddc0606 | Category: Lawsuit, Pleading, Prejudice (Legal Term), Complaint, Cause Of Action
Share Embed Donate


Short Description

Download ATT_1441627983757_Remedial Law part three.pdf...

Description

2014 BAR EXAMINATIONS

REMEDIAL LAW NOTES Part III By:

ELEUTERIO L. BATHAN Presiding Judge, RTC, Br. 92, (NCJR) Quezon City Acting Presiding Judge, RTC, Br. 5, Lemery, Batangas Judge Designate, RTC, Br. 39, Calapan City, Or. Mindoro Judge Designate, RTC, Br. 40, Calapan City, Or. Mindoro Former Executive/Presiding Judge, MTCC, Br. 2, Batangas City Former Acting Judge: MTCC, Br. 1, Batangas City; MTC, Lian, Batangas; Sablayan, Occidental Mindoro; Former Judge Designate, MTC, Romblon, Romblon Law Professor, Remedial Law Review and Political Law University of Batangas (U.B.) (Batangas & Lipa City Campuses) MCLE Lecturer Author, Handbook on Local Governance (Rex Book Store, Inc.) Awardee, Judicial Excellence (2014), Rotary Club of Sta. Mesa District 3780, Quezon City Outstanding Graduate (2014), SSC-R, College of Law Awardee, Quezon City RTCJA 2012 Outstanding Judge Former Member, Sangguniang Bayan, San Jose, Batangas Former Law Practitioner A.B.; Ll.B., San Sebastian College-R (Manila)

---0---

REMEDY OF PLAINTIFF IF DEFENDANT’S MOTION TO DISMISS COMPLAINT IS GRANTED If defendant’s motion to dismiss is granted, the remedy of the plaintiff is to appeal the order of dismissal by way of filing a notice of appeal, because an order of dismissal is a final order (See Rules 40 and 41, 1997 Revised Rules of Civil Procedure). An order of dismissal, whether correct or not, is a final order. It is not interlocutory because the proceedings were terminated; it leaves nothing more to be done by the lower court. Therefore, the remedy of the plaintiff is to appeal the order (Dael vs. Spouses Beltran, 4). The plaintiff may resort first to the filing of a motion for reconsideration before filing a notice of appeal. In situation like this, filing a motion for reconsideration is optional. It is discretionary on the part of the aggrieved party or plaintiff. If he chooses to file answer, he may avail of an extension of time to file one (See above-discussions; See also Virata vs. Sandiganbayan).

---0---

REMEDY OF DEFENDANT IF HIS MOTION TO DISMISS COMPLAINT IS DENIED If the defendant’s motion to dismiss is denied, the following are the remedies available to the defendant: 1. upon denial of the motion for reconsideration to the order of denial, he may file a petition for certiorari under Rule 65; or to 2. file an answer

Though filing a petition for certiorari under Rule 65 is the most dangerous option, but still, defendant can avail of this remedy. The danger or the risk is that instead of focusing on the alternative mode which is the filing of an answer, defendant, upon plaintiff’s motion might be declared by the court in default for failure to file an answer. So, if he chooses to resort to the filing of certiorari under Rule 65, and in order to avoid the possibility of declaring him in default later on, he shall see to it that his petition should be accompanied with an application for ancillary relief of temporary restraining order and/or preliminary injunction, but also take note of the limitations provided under A.M. No. 07-7-12-SC dated December 4, 2007 [effective December 27, 2007] amending Rule 65 of the 1997 Revised Rules of Civil Procedure). As a general rule, an order denying a motion to dismiss is merely interlocutory and cannot be subject of appeal until final judgment or order is rendered (Newsweek, Inc vs. IAC,). It is interlocutory in the sense that it does not finally dispose of the case, and, in effect, directs the case to proceed until final adjudication by the court (Marmo et al., vs. Anacay,). The ordinary procedure to be followed in such a case is to file an answer, go to trial and if the decision is adverse, reiterate the issue on appeal from the final judgment, but this may be subject to certain exceptions (Newsweek, Inc vs. IAC,).

EXCEPTIONS: Only when the court issues an order outside or in excess of jurisdiction or with grave abuse of discretion, and the remedy of appeal would not afford adequate and expeditious relief, will certiorari be considered an appropriate remedy to assail an interlocutory order (Heirs of Hinog vs. Melicor,). If the court, in denying the motion to dismiss, acts without or in excess of jurisdiction or with grave abuse of discretion, then certiorari or prohibition lies. The reason is that it would be unfair to require the defendant to undergo the ordeal and expense of a trial if the court has no

jurisdiction over the subject matter, or is not the court of proper venue, or if the denial of the motion to dismiss is made with grave abuse of discretion or a whimsical and capricious exercise of judgment. In such cases, the ordinary remedy of appeal cannot be plain and adequate. The following are a few examples of the exceptions to the general rule (Newsweek, Inc vs. IAC,). In De Jesus vs. Garcia, , upon the denial of a motion to dismiss based on lack of jurisdiction over the subject matter, the High Court granted the petition for certiorari and prohibition against the City Court of Manila and directed the respondent court to dismiss the case. However, in Bank of America vs. CA, , it was held that the remedy of the defendant is to file an answer to the complaint, proceed to trial and await judgment before making an appeal instead of petition of certiorari under Rule 65. The High Court in Bank of America: “It is a well-settled rule that the order denying the motion to dismiss cannot be the subject of petition for certiorari. Petitioners should have filed an answer to the complaint, proceed to trial and await judgment before making an appeal. As repeatedly held by this Court: “An order denying a motion to dismiss is interlocutory and cannot be the subject of the extraordinary petition for certiorari or mandamus. The remedy of the aggrieved party is to file an answer and to interpose as defenses the objections raised in his motion to dismiss, proceed to trial, and in case of an adverse decision, to elevate the entire case by appeal in due course. xxx Under certain situations, recourse to certiorari or mandamus is considered appropriate, i.e., (a) when the trial court issued the order without or in excess of jurisdiction; (b) where there is patent grave abuse of discretion by the trial court; or (c)

appeal would not prove to be a speedy and adequate remedy as when an appeal would not promptly relieve a defendant from the injurious effects of the patently mistaken order maintaining the plaintiff’s baseless action and compelling the defendant needlessly to go through a protracted trial and clogging the court dockets by another futile case.” Records show that the trial court acted within its jurisdiction when it issued the assailed Order denying petitioners’ motion to dismiss. Does the denial of the motion to dismiss constitute a patent grave abuse of discretion? Would appeal, under the circumstances, not prove to be a speedy and adequate remedy? We will resolve said questions in conjunction with the issues raised by the parties. First issue. Did the trial court commit grave abuse of discretion in refusing to dismiss the complaint on the ground that plaintiffs have no cause of action against defendants since plaintiffs are merely stockholders of the corporations which are the registered owners of the vessels and the borrowers of petitioners? No. Petitioners’ argument that private respondents, being mere stockholders of the foreign corporations, have no personalities to sue, and therefore, the complaint should be dismissed, is untenable. A case is dismissible for lack of personality to sue upon proof that the plaintiff is not the real party-in-interest. Lack of personality to sue can be used as a ground for a Motion to Dismiss based on the fact that the complaint, on the face thereof, evidently states no cause of action. In San Lorenzo Village Association, Inc. vs. Court of Appeals, this Court clarified that a complaint states a cause of action where it contains three essential elements of a cause of action, namely: (1) the legal right of the plaintiff, (2) the correlative obligation of the defendant, and (3) the act or omission of the defendant in violation of

said legal right. If these elements are absent, the complaint becomes vulnerable to a motion to dismiss on the ground of failure to state a cause of action. To emphasize, it is not the lack or absence of cause of action that is a ground for dismissal of the complaint but rather the fact that the complaint states no cause of action. “Failure to state a cause of action” refers to the insufficiency of allegation in the pleading, unlike “lack of cause of action” which refers to the insufficiency of factual basis for the action. “Failure to state a cause of action” may be raised at the earliest stages of an action through a motion to dismiss the complaint, while “lack of cause of action” may be raised any time after the questions of fact have been resolved on the basis of stipulations, admissions or evidence presented. In the case at bar, the complaint contains the three elements of a cause of action. It alleges that: (1) plaintiffs, herein private respondents, have the right to demand for an accounting from defendants (herein petitioners), as trustees by reason of the fiduciary relationship that was created between the parties involving the vessels in question; (2) petitioners have the obligation, as trustees, to render such an accounting; and (3) petitioners failed to do the same. Petitioners insist that they do not have any obligation to the private respondents as they are mere stockholders of the corporation; that the corporate entities have juridical personalities separate and distinct from those of the private respondents. Private respondents maintain that the corporations are wholly owned by them and prior to the incorporation of such entities, they were clients of petitioners which induced them to acquire loans from said petitioners to invest on the additional ships. We agree with private respondents. As held in the San Lorenzo case, “xxx assuming that the allegation of facts constituting plaintiffs’ cause of action is not as

clear and categorical as would otherwise be desired, any uncertainty thereby arising should be so resolved as to enable a full inquiry into the merits of the action.” As this Court has explained in the San Lorenzo case, such a course, would preclude multiplicity of suits which the law abhors, and conduce to the definitive determination and termination of the dispute. To do otherwise, that is, to abort the action on account of the alleged fatal flaws of the complaint would obviously be indecisive and would not end the controversy, since the institution of another action upon a revised complaint would not be foreclosed.”

In Enriquez vs. Macadaeg, , upon the denial of a motion to dismiss based on improper venue, this Court granted the petition for prohibition and enjoined the respondent judge from taking cognizance of the case except to dismiss the same. In Manalo vs. Mariano, , upon the denial of a motion to dismiss based on bar by prior judgment, this Court granted the petition for certiorari and directed the respondent judge to dismiss the case. In Yuviengco vs. Dacuycuy, , upon the denial of a motion to dismiss based on the Statute of Frauds, the High Court granted the petition for certiorari and dismissed the amended complaint. Take note however, that in Marmo et al., vs. Anacay, , the High Court ruled that the trial court did not grave abuse its discretion when it denied the motion to dismiss the complaint for failure of the plaintiff to implead indispensable parties. ---0---

EFFECT OF DISMISSAL

IF MOTION TO DISMISS IS GRANTED If the dismissal is based on the grounds that: (1) the cause of action is barred by a prior judgment or by the statute of limitations (Paragraph (f), Section 1, Rule 16, 1997 Revised Rules of Civil Procedure); (2) the claim or demand set forth in the plaintiff’s pleading has been paid, waived, abandoned, or otherwise extinguished (Paragraph (h), Section 1, Rule 16, 1997 Revised Rules of Civil Procedure); and (3) the claim on which the action is founded is unenforceable under the provisions of the statute of frauds; and (Paragraph (i), Section 1, Rule 16, 1997 Revised Rules of Civil Procedure), subject to the right of appeal, shall bar the refiling of the same action or claim (Section 5, Rule 16, 1997 Revised Rules of Civil Procedure). The dismissal is with prejudice.

Strongworld Construction Corporation vs. Judge Perello, said: “Section 1, Rule 16 of the 1997 Revised Rules of Civil Procedure enumerates the grounds for which a Motion to Dismiss may be filed, viz.: SECTION 1. Grounds. – Within the time for but before filing the answer to the complaint or pleading asserting a claim, a motion to dismiss may be made on any of the following grounds: (a) (b) (c) (d)

(e)

That the court has no jurisdiction over the person of the defending party; That the court has no jurisdiction over the subject matter of the claim; That venue is improperly laid;That the plaintiff has no legal capacity to sue; That there is another action pending between the same parties for the same cause; That the cause of action is barred by a prior judgment or by the statute of limitations;

(f) (g)

(h)

(i)

That the pleading asserting the claim states no cause of action; That the claim or demand set forth in the plaintiff’s pleading has been paid, waived, abandoned, or otherwise extinguished; That the claim on which the action is founded is unenforceable under the provisions of the statute of frauds; and That a condition precedent for filing the claim has not been complied with.

Section 5 of the same Rule, recites the effect of a dismissal under Sections 1(f), (h), and (i), thereof, thus: SEC. 5. Effect of dismissal. – Subject to the right of appeal, an order granting a motion to dismiss based on paragraphs (f), (h), and (i) of section 1 hereof shall bar the refiling of the same action or claim. Briefly stated, dismissals that are based on the following grounds, to wit: (1) that the cause of action is barred by a prior judgment or by the statute of limitations; (2) that the claim or demand set forth in the plaintiff’s pleading has been paid, waived, abandoned or otherwise extinguished; and (3) that the claim on which the action is founded is unenforceable under the provisions of the statute of frauds, bar the refiling of the same action or claim. Logically, the nature of the dismissal founded on any of the preceding grounds is “with prejudice” because the dismissal prevents the refiling of the same action or claim. Ergo, dismissals based on the rest of the grounds enumerated are without prejudice because they do not preclude the refiling of the same action. Verily, the dismissal of petitioners’ Complaint by the court a quo was not based on any of the grounds specified in Section 5, Rule 16 of the 1997 Revised Rules of Civil Procedure; rather, it was grounded on what was

encapsulated in Section 1(g), Rule 16 of the 1997 Revised Rules of Civil Procedure. As the trial court ratiocinated in its 9 January 1998 Order, the Complaint is not prosecuted by the proper party in interest. Considering the heretofore discussion, we can say that the order of dismissal was based on the ground that the Complaint states no cause of action. For this reason, the dismissal of petitioners’ Complaint cannot be said to be a dismissal with prejudice which bars the refiling of the same action. As has been earlier quoted, Section 1(h), Rule 41 of the 1997 Revised Rules of Civil Procedure mandates that no appeal may be taken from an order dismissing an action without prejudice. The same section provides that in such an instant where the final order is not appealable, the aggrieved party may file an appropriate special civil action under Rule 65.” -0-0-0DISMISSAL ON VARIOUS GROUNDS AND ITS EFFECT (WITH PREJUDICE AND WITHOUT PREJUDICE) On filing of initiatory pleading, If act of lawyer or counsel constitute willful and deliberate forum shopping, shall be a ground for the summary dismissal of the case with prejudice (Section 5, Rule 7, 1997 Rules of Court). Regular Procedure (1997 Rules of Court): plaintiff’s failure to appear at the pre-trial conference, the dismissal is with prejudice, unless otherwise ordered by the court (Section 5, Rule 18). General Rule: dismissal is always with prejudice, unless the order says that dismissal is without prejudice. If the order is silent, the dismissal is with prejudice; PLAINTIFF’S failed – defendant may be allowed to present evidence on his counter-claim, if any; DEFENDANT’S failed – plaintiff’s presentation of ex-parte evidence, and the court to render judgment on the basis thereof.

Summary Procedure: plaintiff’s failure to appear at the preliminary conference can cause the dismissal of the action. The dismissal is with prejudice, unless otherwise ordered by the court (Apply Section 5, Rule 18, 1997 Rules of Court, suppletory application). General Rule: dismissal is always with prejudice, unless the order says that dismissal is without prejudice. If the order is silent, the dismissal is with prejudice. Plaintiff’s failure to file pre-trial brief (in ordinary cases) has the same effect of failure to appear at the pre-trial conference (See: Section 6, Rule 18, 1997 Rules of Court). Plaintiff’s failure to file preliminary conference brief (pre-trial brief under summary procedure) has the same effect with failure to appear at the preliminary conference (See: Rule 18, 1997 Rules of Court suppletory application). Rule on Small Claims: Failure of plaintiff to appear at the trial can cause the dismissal of the claim. Take note: The dismissal is always without prejudice (Section 18, Rule on Small Claim). Rules of Procedure for Environmental Cases – repeated and unjustified failure of plaintiff to appear at the pre-trial conference, or to file pre-trial brief, the court shall dismiss the case. The dismissal shall be without prejudice (Section 7, Rule 3 of the Rule). Lack of certification against forum shopping under Section 5, Rule 7, 1997 Rules, shall be cause for the dismissal of the case – dismissal is without prejudice, unless otherwise provided. ---0---

DEFENDANT MAY FILE MOTION FOR EXTENSION OF TIME TO ANSWER

This discussion is confined only to all (civil) cases governed by the rules on regular procedure, excluding the cases governed by the Rules on Summary Procedure (See separate discussion on the matter). If after plaintiff has complied with the order of the court requiring him to submit a bill of particulars, and/or defendant’s motion to dismiss (and/or the motion for reconsideration thereto) is denied, the defendant has three (3) more remaining remedial options. The defendant may either: (1) may still ask for extension of time to file answer; or (2) file an answer; or (3) he may deliberately disregard the complaint by inaction. The defendant may still ask for an extension of time to file answer or motion to dismiss. This is a non-litigious motion which can be done and filed ex-parte. Some trial courts do not anymore rule or resolve the motion for extension of time, and that is, equivalent to allowance. If that is the case, it is incumbent upon the movant to file the intended pleading on or before the expiration of the period prayed for within which to file the same. The rule says: 1. Upon motion and on such terms as may be just, the court may extend the time to plead provided in these Rules (Section 11, Rule 11, 1997 Revised Rules of Civil Procedure). 2. The court may also, upon like terms, allow an answer or other pleading to be filed after the time fixed by the Rules (Paragraph 2, Section 11, Rule 11, 1997 Revised Rules of Civil Procedure). ---0--FILING AN ANSWER TO THE COMPLAINT:

If the defendant’s motion to dismiss is denied his other remedial option aside from filing a certiorari petition (under Rule 65) is to file answer.

A. When to file answer The defendant shall file his answer to the complaint within fifteen (15) days after service of summons, unless a different period is fixed by the court (Section 1, Rule 11, 1997 Revised Rules of Civil Procedure). Where the defendant is a foreign private juridical entity and service of summons is made on the government official designated by law to receive the same, the answer shall be filed within thirty (30) days after receipt of summons by such entity (Section 2, Rule 11, 1997 Revised Rules of Civil Procedure). When the plaintiff files an amended complaint as a matter of right, the defendant shall answer the same within fifteen (15) days after being served with a copy thereof (Section 3, Rule 11, 1997 Revised Rules of Civil Procedure). Where its filing is not a matter of right, the defendant shall answer the amended complaint within ten (l0) days from notice of the order admitting the same. An answer earlier filed may serve as the answer to the amended complaint if no new answer is filed (Paragraph 2, Section 3, Rule 11, 1997 Revised Rules of Civil Procedure).

B. Defenses and objections not pleaded waived Defenses and objections not pleaded either in a motion to dismiss or in the answer are deemed waived (Section 1, Rule 9, 1997 Revised Rules of Civil Procedure).

C. Contents of answer

An answer shall contain in a methodical and logical form, a plain, concise and direct statement of the ultimate facts on which the defendant relies for his defense, omitting the statement of mere evidentiary facts (Section 1, Rule 8, 1997 Revised Rules of Civil Procedure). If a defense relied on is based on law, the pertinent provisions thereof and their applicability to him shall be clearly and concisely stated (Paragraph 2, Section 1, Rule 8, 1997 Revised Rules of Civil Procedure). A defendant may set forth two or more statements of defense alternatively or hypothetically, either in one defense or in separate defenses (Section 2, Rule 8, 1997 Revised Rules of Civil Procedure). Facts showing the capacity of a party to sue or be sued or the authority of a party to sue or be sued in a representative capacity or the legal existence of an organized association of person that is made a party, must be averred. A party desiring to raise an issue as to the legal existence of any party or the capacity of any party to sue or be sued in a representative capacity, shall do so by specific denial, which shall include such supporting particulars as are peculiarly within the pleader's knowledge (Section 4, Rule 8, 1997 Revised Rules of Civil Procedure). In all averments of fraud or mistake the circumstances constituting fraud or mistake must be stated with particularity. Malice, intent, knowledge, or other condition of the mind of a person may be averred generally (Section 5, Rule 8, 1997 Revised Rules of Civil Procedure). Whenever a defense is based upon a written instrument or document, the substance of such instrument or document shall be set forth in the answer, and the original or a copy thereof shall be attached to the answer as an exhibit, which shall be deemed to be a part of the answer, or said copy may with like effect be set forth in the pleading (Section 7, Rule 8, 1997 Revised Rules of Civil Procedure). When a defense is founded upon a written instrument, copied in or attached to the corresponding pleading as provided in the preceding section, the genuineness and due execution of the instrument shall be deemed admitted unless the adverse party, under oath specifically denies them, and sets forth what he claims to be the facts, but the requirement of an oath does not apply when the adverse party does not appear to be a

party to the instrument or when compliance with an order for an inspection of the original instrument is refused (Section 8, Rule 8, 1997 Revised Rules of Civil Procedure). In pleading an official document or official act, it is sufficient to aver that the document was issued or the act done in compliance with law (Section 9, Rule 8, 1997 Revised Rules of Civil Procedure). A defendant must specify each material allegation of fact the truth of which he does not admit and, whenever practicable, shall set forth the substance of the matters upon which he relies to support his denial. Where a defendant desires to deny only a part of an averment, he shall specify so much of it as is true and material and shall deny only the remainder. Where a defendant is without knowledge or information sufficient to form a belief as to the truth of a material averment made to the complaint, he shall so state, and this shall have the effect of a denial (Section 10, Rule 8, 1997 Revised Rules of Civil Procedure). Material averment in the complaint, other than those as to the amount of unliquidated damages, shall be deemed admitted when not specifically denied. Allegations of usury in a complaint to recover usurious interest are deemed admitted if not denied under oath (Section 11, Rule 8, 1997 Revised Rules of Civil Procedure).

DENIAL AND CONTESTING THE GENUINENESS AND DUE EXECTION OF ACTIONABLE DOCUMENT A. How to deny the genuineness and due execution of an actionable document? The defendant is required to make an effective specific denial. This means that the defendant must declare under oath that he did not sign the document or that it is otherwise false or fabricated. Neither does the statement of the answer to the effect that the instrument was procured by fraudulent representation raise any issue as to its genuineness or due execution. On the contrary such a plea is an admission both of the genuineness and due execution thereof, since it seeks to avoid the instrument upon a ground not affecting either (Songco vs. Sellner,).

B. Case No. 1 Plaintiff alleged in the complaint that defendant is indebted to the former in the amount of P1M attaching in the complaint several annexes including the photocopy of the promissory note. In his answer, he alleged that he has never signed the promissory note attached to the complaint in his personal and/or individual capacity, thus ineffective, unenforceable and void for lack of valid consideration. In his verification, he stated that: (1) that the defendant, after having been duly sworn to in accordance with law, hereby depose and declare that she is the named defendant in the above-entitled case; he has cause the preparation of the answer upon facts and figures supplied by his to his retained counsel; have read each and every allegations contained therein and hereby certify that the same are true and correct of her own knowledge and information. At the pre-trial, plaintiff was able to prove that the defendant received the amount of P1M as loan after signing the Promissory Note (Annex A), that defendant, upon receipt of the demand letter made a reply seeking an extension to pay her obligation. On the Formal Offer of Exhibits Plaintiff sought the admission of the duplicate original of the PN on the ground that the original copy could no longer be found. The trial court initially admitted into evidence the duplicate original of the PN, and allowed Defendant to amend her answer to conform with this new evidence. Upon Defendant’s motion for reconsideration arguing that the duplicate Original PN was not properly identified and there were markings in the photocopy which were not contained in duplicate original, the trial court granted the MR and dismissed the case on the ground that Plaintiff had no longer possessed any proof of Defendant’s alleged indebtedness. The trial court found that there can be no dispute to the fact that the allegations in the answer of defendant, she denied generally and specifically under oath the genuineness and due execution of the promissory note and by way of special and affirmative defenses herein states that she never signed the promissory note attached to the complaint in his personal and/or individual capacity. She also deny generally and specifically the rest of the allegations. It would be considered that there is a sufficient compliance of the requirement of the law for specific denial. Is the denial specific?

Solidbank vs. Del Monte Motor Works, Inc., said defendant’s denial is not specific. Section 8, Rule 8, 1997 Rules of Court provides that when an action or defense is founded upon a written instrument, copied in or attached to the corresponding pleading as provided in the preceding section, the genuineness and due execution of the instrument shall be deemed admitted unless the adverse party, under oath, specifically denies them and sets forth what he claims to be the facts.

Is secondary evidence still needed? Since Defendant failed to deny specifically the execution of the promissory note, there was no need for the Plaintiff to present the original of the promissory note in question. Defendant’s judicial admission with respect to the genuineness and execution of the promissory note sufficiently established her liability to Plaintiff regardless of the fact that Plaintiff failed to present the original of said note. Therefore, there is thus no need of proof of execution and authenticity with respect to the loan document because of Defendant’s implied admission of loan transaction. There is no need to comply with what Section 22, Rule 132 of the Rules of Court which requires that before a private documents can be received in evidence, presentation and examination of witnesses to testify to prove its due execution and authenticity (Solidbank vs. Del Monte Motor Works, Inc.,). C. Case No. 2 Defendant’s answer contained the following: 1. The allegations in par. 2, Complaint, on the existence of the alleged loan of P2-M, and the purported documents evidencing the same, only the signature appearing at the back of the promissory note, Annex “A” seems to be that of herein defendant. However, as to any liability arising therefrom, the receipt of the said amount of P2-M shows that the amount was received by another person, not the herein defendant. Hence, no liability

attaches and as further stated in the special and affirmative defenses that, assuming the promissory note exists, it does not bind much less is there the intention by the parties to bind the herein defendant. In other words, the documents relative to the loan do not express the true intention of the parties. 2. (Verification) “I, Defendant, of age, am the defendant in this case, that I caused the preparation of the complaint and that all the allegations thereat are true and correct; that the promissory note sued upon, assuming that it exists and bears the genuine signature of herein defendant, the same does not bind him and that it did not truly express the real intention of the parties as stated in the defenses”

Is this an effective specific denial as contemplated by law? No. A reading of Defendant’s Answer, shows that he did not specifically deny that he signed the loan documents. What he merely stated in his Answer was that the signature appearing at the back of the promissory note seemed to be his. Defendant also denied any liability on the promissory note as he allegedly did not receive the amount stated therein, and the loan documents do not express the true intention of the parties. Defendant reiterated these allegations in his “denial under oath,” stating that “the promissory note sued upon, assuming that it exists and bears the genuine signature of herein defendant, the same does not bind him and that it did not truly express the real intention of the parties as stated in the defenses. His answer amounts to an implied admission of the due execution and genuineness of the promissory note. The admission of the genuineness and due execution of a document means that the party whose signature it bears admits that he voluntarily signed the document or it was signed by another for him and with his authority; that at the time it was signed it was in words and figures exactly as set out in the pleading of the party relying upon it; that the document was delivered; and that any formalities required by law, such as a seal, an acknowledgment, or revenue stamp, which it

lacks, are waived by him. Also, it effectively eliminated any defense relating to the authenticity and due execution of the document, e.g., that the document was spurious, counterfeit, or of different import on its face as the one executed by the parties; or that the signatures appearing thereon were forgeries; or that the signatures were unauthorized. Therefore, Defendant is deemed to have admitted the loan documents and acknowledged his obligation with Plaintiff; and with Defendant’s implied admission, it was not necessary for Plaintiff to present further evidence to establish the due execution and authenticity of the loan documents sued upon (Permanent Savings and Loan Bank vs. Mariano Velarde,).

D. Adoptive admission concept and meaning A party may, by his words or conduct, voluntarily adopt or ratify another's statement. Where it appears that a party clearly and unambiguously assented to or adopted the statements of another, evidence of those statements is admissible against him. This is the essence of the principle of adoptive admission (Republic vs. Kenrick Development Corporation,). An adoptive admission is a party's reaction to a statement or action by another person when it is reasonable to treat the party's reaction as an admission of something stated or implied by the other person. By adoptive admission, a third person's statement becomes the admission of the party embracing or espousing it (Republic vs. Kenrick Development Corporation, G.R. No. 149576, August 8, 2006). E. Adoptive admission concept and meaning Adoptive admission may occur when a party: (a) expressly agrees to or concurs in an oral statement made by another; (b) hears a statement and later on essentially repeats it;

(c) utters an acceptance or builds upon the assertion of another; (d) replies by way of rebuttal to some specific points raised by another but ignores further points which he or she has heard the other make or (e) reads and signs a written statement made by another (Republic vs. Kenrick Development Corporation,).

F. Answer must be signed An unsigned answer is invalid and produces no legal effect. Defendant may be declared in default and allow plaintiff to present evidence ex-parte (Republic vs. Kenrick Development Corporation,). ---0---

COUNTERCLAIM

A. What is a counterclaim? A counterclaim is any claim which a defending party may have against an opposing party (Section 6, Rule 6, 1997 Revised Rules of Civil Procedure). This may be a claim for money or other relief (See Sapugay vs. CA,).

B. Kinds, Principles and its Limitations Counterclaim may be compulsory or permissive. Counterclaims are generally allowed in order to avoid a multiplicity of suits and to facilitate the disposition of the whole controversy in a single action, such that the defendant’s demand may be

adjudged by a counterclaim rather than by an independent suit (Lafarge Cement Philippines, Inc., et al, vs. Continental Cement Corporation, et al,). The only limitations to this principle are (1) that the court should have jurisdiction over the subject matter of the counterclaim, and (2) that it could acquire jurisdiction over third parties whose presence is essential for its adjudication suit (Lafarge Cement Philippines, Inc., et al, vs. Continental Cement Corporation,).

COMPULSORY COUNTERCLAIM A compulsory counterclaim is one which, being cognizable by the regular courts of justice, arises out of or is connected with the transaction or occurrence constituting the subject matter of the opposing party's claim and does not require for its adjudication the presence of third parties of whom the court cannot acquire jurisdiction. Such a counterclaim must be within the jurisdiction of the court both as to the amount and the nature thereof, except that in an original action before the Regional Trial Court, the counter-claim may be considered compulsory regardless of the amount (Section 7, Rule 6, 1997 Revised rules of Civil Procedure). It is compulsory in the sense that it is within the jurisdiction of the court. It does not require for its adjudication the presence of third parties over whom the court cannot acquire jurisdiction, and will be barred in the future if not set up in the answer to the complaint in the same case. Any other counterclaim is permissive (Cruz-Agana vs. Judge SantiagoLagman,). A compulsory counterclaim set up in the answer is not an “initiatory” or similar pleading. The initiatory pleading is the plaintiff’s complaint. A defendant has no choice but to raise a compulsory counterclaim the moment the plaintiff files the complaint, otherwise defendant waives the compulsory counterclaim. In short, the compulsory counterclaim is a reaction or response, mandatory upon pain of waiver, to an initiatory pleading which is the complaint (Cruz-Agana vs. Judge Santiago-Lagman,).

A. Criteria or Tests of Compulsoriness The following criteria to determine whether a counterclaim is compulsory or permissive: (1) Are issues of fact and law raised by the claim and by the counterclaim largely the same? (2) Would res judicata bar a subsequent suit on defendant’s claim, absent the compulsory counterclaim rule? (3) Will substantially the same evidence support or refute plaintiff’s claim as well as defendant’s counterclaim? (4) Is there any logical relation between the claim and the counterclaim? A positive answer to all four questions would indicate that the counterclaim is compulsory (NAMARCO vs. Federation of United Namarco Distributors,). In short, if (1) the issues of fact or law raised by the claim and the counterclaim largely the same; (2) res judicata will bar a subsequent suit on defendant’s claim absent the compulsory counterclaim rule; (3) the same evidence will substantially support or refute plaintiff’s claim as well as the defendant’s counterclaim; and (4) there is any logical relation between the claim and the counterclaim, the claim is compulsory. A counterclaim to be considered as compulsory it must passed the so-called “compelling test of compulsoriness”, and that is, the test of “logical relationship” between the main claim and the counterclaim must exist. There exists such a relationship when conducting separate trials of the respective claims of the parties would entail substantial duplication of time and effort by the parties and the court; when the multiple claims involve the same factual and legal issues; or when the claims are offshoots of the same basic controversy between the parties (Quintanilla vs. CA). B. Situationer

If for example, X moves to intervene as party defendant in a suit between Plaintiff and Defendant, and in his answer he asks plaintiff for attorney’s fees, moral and exemplary damages as counterclaim for, it is humbly opines that his counterclaim is permissive, because, he voluntarily participated in the case without waiting first to be impeladed as party defendant. It would have been different had Plaintiff later on amends his complaint by impleading X as party defendant (indispensable or necessary) and X files his answer pursuant to summons served upon him, this time, it is humbly opines that counter claim for attorney’s fees, moral and exemplary damages pleaded in his answer is compulsory. Therefore, if the attorney’s fees, moral and exemplary damges is permissive, then he is obliged to pay docket fees because, payment of correct docket fees is jurisdictional.

C. Compulsory Counterclaim not set up on time is barred A compulsory counterclaims not set up shall be barred (Section 2, Rule 9, 1997 Revised Rules of Civil Procedure). The same should be set up in the same action, otherwise, they would be barred forever (Lafarge Cement Philippines, Inc., et al, vs. Continental Cement Corporation,). If it is not set up shall be barred if not raised on time and the party in error is precluded from setting it up in a subsequent litigation on the ground of res judicata, the theory being that what are barred by prior judgment are not only the matters actually raised and litigated upon, but also such matters as could have been raised but were not (Metals Engineering Resources Corporation vs. CA,). It cannot be made the subject of a separate action but should be asserted in the same suit involving the same transaction or occurrence giving rise to it (Metals Engineering Resources Corp. vs. Court of Appeals). This is because it is an auxiliary to the proceeding in the original suit and derives its jurisdictional support therefrom, inasmuch as it arises out of or is necessarily connected with the transaction or occurrence that is the subject matter of the complaint (Metals Engineering Resources Corp. vs. Court of Appeals.), and if made the subject of a separate suit, it may be abated upon a plea of auter action pendant or litis pendentia, and/or

dismissed on the ground of res judicata (Visayan Packing Corporation vs. Reparations Commission,).

D. Compulsory Counterclaim is not an initiatory pleading A compulsory counterclaim does not require a certificate of nonforum shopping because a compulsory counterclaim is not an initiatory pleading. Lack of certificate of non-forum shopping is immaterial (CruzAgana vs. Judge Santiago-Lagman,). E. Effect of Dismissal of Plaintiff’s Complaint The dismissal of the complaint due to the fault of plaintiff does not necessarily carry with it the dismissal of the counterclaim, compulsory or otherwise. In fact, the dismissal of the complaint is without prejudice to the right of defendants to prosecute the counterclaim (Pinga vs. Heirs of Santiago,). In short, the defendant’s counterclaims must be disposed of based on the merit of the counterclaim itself and not on the survival of the main complaint; if the counterclaim is palpably without merit or suffers jurisdictional flaws which stand independent of the complaint, the trial court is not precluded from dismissing it under the amended rules, provided that the judgment or order dismissing the counterclaim is premised on those defects; and if the counterclaim is justified, counterclaim is protected by the rules from peremptory dismissal by reason of the dismissal of the complaint (Pinga vs. Heirs of Santiago,). It is worthy to note that what was given emphasis in Pinga is dismissal of actions under Sections 2 and 3 of Rule 17 of the 1997 Revised Rules of Civil Procedure. Dismissal of plaintiff’s complaint by reason of defendant’s motion to dismiss is not touched in Pinga. So, by analogy application of the ruling in Pinga, if the complaint is dismissed by reason of defendant’s motion (to dismiss), the defendant’s compulsory counterclaim survives, as complaint’s dismissal does not result in the automatic dismissal of the counterclaim.

Pinga case abandoned the ruling in Financial Building Corporation vs. Forbes Park Association,) that dismissal of the main action results in the dismissal of the counterclaim already filed. Therefore, the rationale that “it stands to reason that the filing of a motion to dismiss the complaint is an implied waiver of the compulsory counterclaim because the grant of the motion ultimately results in the dismissal of the counterclaim” in Financial Building Corporation case is no longer controlling. It also abandoned the ruling in Metals Engineering Resources Corp. vs. CA). In that case the complaint was dismissed through defendant’s motion on the ground of lack of jurisdiction for non-payment of docket fees which dismissal was affirmed by the High Court holding that that by reason of said dismissal, it is as if no claim was filed against the defendant, hence the counterclaim has no leg to stand on. The doctrinal pronouncement in Metals that “if the court does not have jurisdiction to entertain the main action of the case and dismisses the same, the compulsory counterclaim, being ancillary to the principal controversy, must likewise be dismissed since no jurisdiction remained for any grant of relief under the counterclaim” is also no longer controlling. Now, it is no longer tree that dismissal of the complaint necessarily carries the dismissal of the compulsory counterclaim.

F. Remedy to file by defendant if he has a compulsory counterclaim against the Plaintiff: Either Answer or Motion to Dismiss The filing of a motion to dismiss and the setting up of a compulsory counterclaim are incompatible remedies (Financial Building Corporation vs. Forbes Park Association). In the event that a defending party has a ground for dismissal and a compulsory counterclaim at the same time, he must choose only one remedy. If he decides to file a motion to dismiss, he will lose his compulsory counterclaim. But if he opts to set up his compulsory counterclaim, he may still plead his ground for dismissal as an affirmative

defense in his answer (Financial Building Corporation vs. Forbes Park Association, Inc.,). In short, the defendant may either file a motion to dismiss or file an answer and asserts therein his motion to dismiss as an affirmative defense. This of course, goes without saying that in the event that if defendant opts to file a motion to dismiss, he may still file an answer with compulsory counterclaim if his motion to dismiss will be denied.

G. Effects of plaintiff’s failure to appear during pre-trial conference to defendant’s compulsory counterclaim The defendant’s compulsory counterclaim is not dismissible by reason of dismissal of the plaintiff’s complaint for plaintiff’s failure to appear during the pre-trial. According to Spouses Corpuz vs. Citibank, N.A,), plaintiff’s failure to appear during the pre-trial conference pursuant to Section 3, Rule 17, 1997 Revised Rules of Civil Procedure amounts to failure to comply with the Rules or any Order of the Court. ---0---

PERMISSIVE COUNTERCLAIM A counterclaim is permissive if it does not arise out of or is not necessarily connected with the subject matter of the opposing party’s claim (Lopez v. Gloria,). It is essentially an independent claim that may be filed separately in another case (Lafarge Cement Philippines, Inc., et al, vs. Continental Cement Corporation,), or is capable of proceeding independently of the main case (Bungcayao, Sr. vs. Fort Ilocandia Property Holdings and Development Corporation,).

A. Set-off and Recoupment as grounds for counterclaims

A set-off and recoupment can be a subject of counterclaims (Korea Exchange Bank vs. Judge Gonzales,) provided that the following must concur: (1) the same be essentially a genuine action of the defendant against the plaintiff; (2) the same should have as its object to neutralize, wholly or partially, that which the plaintiff is trying to obtain; (3) the same does not have for its object to destroy directly the action of the plaintiff; and (4) the same ought not to pray for a positive remedy distinct from the payment of money (Lopez vs. Gloria).

B. Set-off (Compensacion) distinguished

and

recoupment

(reconvencion)

A set-off (compensacion) is a money demand by the defendant against the plaintiff arising upon contract and constituting a debt independent of and unconnected with the cause of actions set forth in the complaint, and may be used to offset a plaintiff’s claim but not to recover affirmatively (Lopez vs. Gloria). A recoupment (reconvencion) differs from a counterclaim (contrarreclamacion) in that, under a counterclaim, the defendant may have an affirmative judgment where he is able to prove a demand in excess of the plaintiff’s demand, whereas in the case of recoupment, whatever the damages proved by the defendant, they can go only to reduce or extinguish the claim against him. Recoupment must arise out of the contract or transaction upon which the plaintiff’s claim is founded. Recoupment is of French origin and means the “cutting back of the plaintiff’s claim by the defendant.” It thus implies an admission of the plaintiff’s claim (Lopez vs. Gloria).

C. Payment of docket fees for permissive counterclaim is jurisdictional The counterclaimant is bound to pay the prescribed docket fees (Sandejas v. Ignacio, Jr.,). A permissive counterclaim also requires payment of the prescribed docket fees, because it is not only the filing of the complaint or

appropriate initiatory pleading, but the payment of the prescribed docket fees that vests the court with jurisdiction over the subject matter of the action (Tan vs. Planters Products,). The non-payment of which is a ground for the dismissal of the permissive counterclaim on ground of lack of jurisdiction over it, and the court by authority of Section 1, Rule 9 of the 1997 Revised Rules of Civil Procedure may dismiss it motu proprio or sans motion to dismiss (Tan vs. Planters Products, Inc,). Any decision rendered without jurisdiction with respect to permissive counterclaim is a total nullity and may be struck down at any time, even on appeal before this Court (Sandejas v. Ignacio, Jr.,). In Sapugay vs. CA , it was held that payment of docket fees is specifically applicable to permissive counterclaims only. Compulsory counterclaim is excluded from its purview.

D. Counterclaimant should be ordered to pay the prescribed docket fees If the trial court made a mistake by saying that the counterclaim is compulsory and on appeal it found out that the counterclaim is permissive, the counterclaim is not dismissible, the counterclaimant should be given a reasonable period of time, but in no case beyond the reglementary period, within which to pay the prescribed docket fees because the counterclaimant should not be made to suffer due to the mistake of the trial court (Calibre Traders, Inc. vs. Bayer Philippines,).

E. Permissive Counterclaim requires a certificate of non-forum shopping A permissive counterclaim requires a certificate of non-forum shopping because it is an initiatory pleading. Lack of it is fatal (CruzAgana vs. Judge Santiago-Lagman,), and shall be cause for the dismissal of the counterclaim. The dismissal is always without prejudice, unless

otherwise provided (See Section 5(2) Rule 7, 1997 Revised Rules of Civil Procedure).

F. Summons is not required to file answer in counterclaim Well settled is the rule that the purpose of summons is to enable the court to acquire jurisdiction over the person of the defendant. But as regards counterclaim, the Rules of Court does not require that summons should first be served on the defendant before an answer to counterclaim must be made. Although a counterclaim is treated as an entirely distinct and independent action, the defendant in the counterclaim, being the plaintiff in the original complaint, has already submitted to the jurisdiction of the court (Francisco Motors Corporation vs. CA,).

G. Rule on default in counterclaim If the defendant (plaintiff in the complaint) fails within the reglementary period, that is, ten (10) days from service (Section 4, Rule 11, 1997 Revised Rules of Civil Procedure) within which to file an answer, he, upon motion of the counterclaimant (or the plaintiff in the counterclaim) may be declared in default (Section 3, Rule 9, 1997 Rules of Civil Procedure) (Francisco Motors Corporation vs. CA,).

H. Moral, actual and exemplary damages can either be compulsory or permissive Counterclaim for moral, actual and exemplary damages and attorney’s fees on account of plaintiff’s malicious and unfounded complaint is compulsory (Tiu Po vs. Bautista,), but in Korea Exchange Bank vs. Judge Gonzales), a counterclaim for moral and exemplary damages is considered permissive. Therefore, not all claims for moral and exemplary damages is always compulsory, because this may also be in the nature of permissive counterclaim. It can be compulsory or permissive depending upon the circumstances by which the same is claimed.

---0---

PARTIES’ REMEDIES AFTER ANSWER IS FILED The following are the remedies available to the parties after an answer is filed: 1. File a Motion for Judgment Based on Pleadings; 2. File a Motion for Summary Judgment; or 3. (Defendant) to file Motion for Preliminary Hearing on his affirmative defense asserted in the answer (under Section 6, Rule 16, 1997 Revised Rules of Civil Procedure).

I. FILE A MOTION FOR JUDGMENT BASED ON PLEADINGS

A. The rule Where an answer fails to tender an issue, or otherwise admits the material allegations of the adverse party's pleading, the court may, on motion of that party, direct judgment on such pleading. xxx xxx xxx (Section 1, Rule 34, 1997 Revised Rules of Civil Procedure). B. Concept Judgment on the pleadings is improper when the answer to the complaint tenders several issues. It is proper when the answer admits all the material averments of the complaint. But where several issues are properly

tendered by the answer, a trial on the merits must be resorted to in order to afford each party his day in court (Municipality of Tiwi, vs. Betito, ). Where a motion for judgment on the pleadings is filed, the essential question is whether there are issues generated by the pleadings. In a proper case for judgment on the pleadings, there is no ostensible issue at all because of the failure of the defending party’s answer to raise an issue. The answer would fail to tender an issue, of course, if it does not deny the material allegations in the complaint or admits said material allegations of the adverse party’s pleadings by confessing the truthfulness thereof and/or omitting to deal with them at all (Tan v. De la Vega,). When what is left are not genuinely issues requiring trial but questions concerning the proper interpretation of the provisions of some written contract attached to the pleadings, judgment on the pleadings is proper (Philippine National Bank vs. Utility Assurance & Surety Co., Inc.,). A motion for judgment on the pleadings admits the truth of all the material and relevant allegations of the opposing party and the judgment must rest on those allegations taken together with such other allegations as are admitted in the pleadings. It is proper when an answer fails to tender an issue, or otherwise admits the material allegations of the adverse party’s pleading. However, when it appears that not all the material allegations of the complaint were admitted in the answer for some of them were either denied or disputed, and the defendant has set up certain special defenses which, if proven, would have the effect of nullifying plaintiff’s main cause of action, judgment on the pleadings cannot be rendered (Municipality of Tiwi, vs. Betito,). The trial court has the discretion to grant a motion for judgment on the pleadings filed by a party if there is no controverted matter in the case after the answer is filed. A judgment on the pleadings is a judgment on the facts as pleaded, and is based exclusively upon the allegations appearing in the pleadings of the parties and the accompanying annexes (Sunbanun vs. Go,). If it is the defendant who moves for a judgment on the pleadings without offering proof as to the truth of her own allegations and without giving plaintiff the opportunity to introduce evidence, defendant is deemed

to have admitted the material and relevant averments of the complaint, and to rest her motion for judgment based on the pleadings of the parties (Rodriguez vs. Llorente,). If the plaintiff asks the court for judgment on the pleadings and defendant has consented to it, and if judgment is not favorable to the defendant, he can no longer question the validity of the judgment. In Tropical Homes, Inc. vs. CA, , the High Court said: “As to the amount of damages awarded as a consequence of this violation of plaintiff’s rights, the lower court based its award from the allegations and prayer contained in the complaint. The defendant, however, questions this award for the reason that, according to the defendant, the plaintiff, in moving for judgment on the pleadings, did not offer proof as to the truth of his own allegations with respect to the damages claimed by him, and gave no opportunity for the appellant to introduce evidence to refute his claims. We find this objection without merit. It appears that when the plaintiff moved to have the case decided on the pleadings, the defendant interposed no objection and has practically assented thereto. The defendant, therefore, is deemed to have admitted the allegations of fact of the complaint, so that there was no necessity for plaintiff to submit evidence of his claim.” If the court can resolve the issues presented by the complaint and the answer can be resolved within the four corners of said pleadings without need to conduct further hearings, and what remains to be done is the proper interpretation of the contracts or documents attached to the pleadings, then judgment on the pleadings is proper (Pacific Rehouse Corporation vs. EIB Securities, Inc.,). In an action for sum of money and defendants have admitted that they obtained the loan; have admitted the due execution of the loan documents and their receipt of the demand letter made by the plaintiff, and only issue is whether the obligation is matured or not, the High Court held that the case can be decided summarily, because when plaintiff made its

demand, the obligation matured. The matter proferred as a defense could be resolved judiciously by plain resort to the stipulations in the promissory note (Wood Technology vs. Equitable Bank,).

II. FILE A MOTION FOR SUMMARY JUDGMENT A. The rules A party seeking to recover upon a claim, counterclaim, or crossclaim or to obtain a declaratory relief may, at any time after the pleading in answer thereto has been served, move with supporting affidavits, depositions or admissions for a summary judgment in his favor upon all or any part thereof (Section 1, Rule 35, 1997 Revised Rules of Civil Procedure). A party against whom a claim, counterclaim, or cross-claim is asserted or a declaratory relief is sought may, at any time, move with supporting affidavits, depositions or admissions for a summary judgment in his favor as to all or any part thereof (Section 2, Rule 35, 1997 Revised Rules of Civil Procedure). The motion shall be served at least ten (10) days before the time specified for the hearing. The adverse party may serve opposing affidavits, depositions, or admissions at least three (3) days before the hearing. After the hearing, the judgment sought shall be rendered forthwith if the pleadings, supporting affidavits, depositions, and admissions on file, show that, except as to the amount of damages, there is no genuine issue as to any material fact and that the moving party is entitled to a judgment as a matter of law (Section 3, Rule 35, 1997 Revised Rules of Civil Procedure). If on motion under this Rule, judgment is not rendered upon the whole case or for all the reliefs sought and a trial is necessary, the court at the hearing of the motion, by examining the pleadings and the evidence before it and by interrogating counsel shall ascertain what material facts exist without substantial controversy and what are actually and in good faith controverted. It shall thereupon make an order specifying the facts

that appear without substantial controversy, including the extent to which the amount of damages or other relief is not in controversy, and directing such further proceedings in the action as are just. The facts so specified shall be deemed established, and the trial shall be conducted on the controverted facts accordingly (Section 4, Rule 35, 1997 Revised Rules of Civil Procedure). Supporting and opposing affidavits shall be made on personal knowledge, shall set forth such facts as would be admissible in evidence, and shall show affirmatively that the affiant is competent to testify to the matters stated therein. Certified true copies of all papers or parts thereof referred to in the affidavit shall be attached thereto or served therewith (Section 5, Rule 35, 1997 Revised Rules of Civil Procedure). Should it appear to its satisfaction at any time that any of the affidavits presented pursuant to this Rule are presented in bad faith, or solely for the purpose of delay, the court shall forthwith order the offending party or counsel to pay to the other party the amount of the reasonable expenses which the filing of the affidavits caused him to incur including attorney's fees, it may, after hearing further adjudge the offending party or counsel guilty of contempt (Section 6, Rule 35, 1997 Revised Rules of Civil Procedure).

B. Concept and purpose A summary judgment is granted to settle expeditiously a case if, on motion of either party, there appears from the pleadings, depositions, admissions, and affidavits that no important issues of fact are involved, except the amount of damages. In such event, the moving party is entitled to a judgment as a matter of law. Trial courts have limited authority to render summary judgments and may do so only when there is clearly no genuine issue as to any material fact (). In Calubaquib vs. Republic, , the High Court explained the concept of summary judgment, thus: “Summary judgments are proper when, upon motion of the plaintiff or the defendant, the court finds that the answer filed

by the defendant does not tender a genuine issue as to any material fact and that one party is entitled to a judgment as a matter of law. A deeper understanding of summary judgments is found in Viajar v. Estenzo: Relief by summary judgment is intended to expedite or promptly dispose of cases where the facts appear undisputed and certain from the pleadings, depositions, admissions and affidavits. But if there be a doubt as to such facts and there be an issue or issues of fact joined by the parties, neither one of them can pray for a judgment. Where the facts pleaded by the parties are disputed or contested, proceedings for a judgment cannot take the place of a trial. An examination of the Rules will readily show that a judgment is by no means a hasty one. It assumes a scrutiny of facts in a summary hearing after the filing of a motion for judgment by one party supported by affidavits, depositions, admissions, or other documents, with notice upon the adverse party who may file an opposition to the motion supported also by affidavits, depositions, or other documents x x x. In spite of its expediting character, relief by judgment can only be allowed after compliance with the minimum requirement of vigilance by the court in a summary hearing considering that this remedy is in derogation of a party's right to a plenary trial of his case. At any rate, a party who moves for judgment has the burden of demonstrating clearly the absence of any genuine issue of fact, or that the issue posed in the complaint is so patently unsubstantial as not to constitute a genuine issue for trial, and any doubt as to the existence of such an issue is resolved against the movant. “A summary judgment is permitted only if there is no genuine issue as to any material fact and [the] moving party is

entitled to a judgment as a matter of law.” The test of the propriety of rendering summary judgments is the existence of a genuine issue of fact, “as distinguished from a sham, fictitious, contrived or false claim.” “[A] factual issue raised by a party is considered as sham when by its nature it is evident that it cannot be proven or it is such that the party tendering the same has neither any sincere intention nor adequate evidence to prove it. This usually happens in denials made by defendants merely for the sake of having an issue and thereby gaining delay, taking advantage of the fact that their answers are not under oath anyway.” In determining the genuineness of the issues, and hence the propriety of rendering a summary judgment, the court is obliged to carefully study and appraise, not the tenor or contents of the pleadings, but the facts alleged under oath by the parties and/or their witnesses in the affidavits that they submitted with the motion and the corresponding opposition. Thus, it is held that, even if the pleadings on their face appear to raise issues, a summary judgment is proper so long as “the affidavits, depositions, and admissions presented by the moving party show that such issues are not genuine.”

In Nocom vs. Camerino, , a summary judgment is explained as a procedural device resorted to in order to avoid long drawn out litigations and useless delays. When the pleadings on file show that there are no genuine issues of fact to be tried, the Rules allow a party to obtain immediate relief by way of summary judgment, that is, when the facts are not in dispute, the court is allowed to decide the case summarily by applying the law to the material facts. Conversely, where the pleadings tender a genuine issue, summary judgment is not proper. A “genuine issue” is such issue of fact which requires the presentation of evidence as distinguished from a sham, fictitious, contrived or false claim. Section 3 of the said rule provides two (2) requisites for summary judgment to be proper: (1) there must be no genuine issue as to any material fact, except

for the amount of damages; and (2) the party presenting the motion for summary judgment must be entitled to a judgment as a matter of law. A summary judgment is permitted only if there is no genuine issue as to any material fact and a moving party is entitled to a judgment as a matter of law. A summary judgment is proper if, while the pleadings on their face appear to raise issues, the affidavits, depositions, and admissions presented by the moving party show that such issues are not genuine. The law itself determines when a summary judgment is proper. Under the rules, summary judgment is appropriate when there are no genuine issues of fact which call for the presentation of evidence in a fullblown trial. Even if on their face the pleadings appear to raise issues, when the affidavits, depositions and admissions show that such issues are not genuine, then summary judgment as prescribed by the rules must ensue as a matter of law. What is crucial for determination, therefore, is the presence or absence of a genuine issue as to any material fact (Vadel Realty and Development Corporation vs. Spouses Soriano,). The “the amount of damages” is excluded in the coverage, because in order to prove the issue as to amount of damages, presentation of evidence is necessary. In Bungcayao, Sr. vs. Fort Ilocandia Property Holdings, the trial court in an action for declaration of nullity of contract noted the stipulated issues and admissions been made by both parties, made to confirm the agreement of the parties to cancel the Deed of Assignment, Release, Waiver and Quitclaim and the return of the money to the respondent, and summarily decided the case dismissing the plaintiff’s claim for damages for want of legal basis, and directed to immediately vacate the premises subject of the case.

C. Whether a case is appropriate for summary judgment, determining guide In other words, in a motion for summary judgment, the crucial question is: are the issues raised in the pleadings genuine, sham or

fictitious, as shown by affidavits, depositions or admissions accompanying the motion? (Manufacturers Hanover Trust Co. vs. Guerrero, ). So, if there is genuine issue to be resolved and necessitates the presentation of evidence to resolve the issue, then summary judgment is not proper.

A. Genuine issue, concept/definition A “genuine issue” is an issue of fact which requires the presentation of evidence as distinguished from a sham, fictitious, contrived or false claim. When the facts as pleaded appear uncontested or undisputed, then there is no real or genuine issue or question as to the facts, and summary judgment is called for. The party who moves for summary judgment has the burden of demonstrating clearly the absence of any genuine issue of fact, or that the issue posed in the complaint is patently unsubstantial so as not to constitute a genuine issue for trial. Trial courts have limited authority to render summary judgments and may do so only when there is clearly no genuine issue as to any material fact. When the facts as pleaded by the parties are disputed or contested, proceedings for summary judgment cannot take the place of trial (Evadel Realty and Development Corporation v. Soriano,). In Cotabato Timberland Co., Inc. vs. C. Alcantara and Sons, Inc., , it was held that summary judgment is not proper because there is a necessity of trial on the merits and presentation of evidence for the trial court to properly determine which among the parties must shoulder the loss. In that case, the special and affirmative defenses raised by private respondents invoking, inter alia, the alleged fault and negligence of petitioner as the proximate cause of the loss of the subject logs indubitably tender a genuine and factual issue as regards the proximate cause of the loss. A full-blown trial on the merits and presentation of additional evidence is called for. When variance in the allegations of the parties in the pleadings is evident, and the facts as pleaded by the parties are disputed or contested, the case on appeal should be remanded to the trial court for further proceedings and proper disposition according to the rudiments of a regular trial on the merits and not

through an abbreviated termination of the case by summary judgment, because proceedings for summary judgment cannot take the place of trial (Atty. Ferrer vs. Spouses Diaz,).

B. Sham issue or claim, concept A factual issue raised by a party is considered as sham when by its nature it is evident that it cannot be proven or it is such that the party tendering the same has neither any sincere intention nor adequate evidence to prove it. This usually happens in denials made by defendants merely for the sake of having an issue and thereby gaining delay, taking advantage of the fact that their answers are not under oath anyway (Calubaquib vs. Republic, ).

F. Filing of motion and conduct of hearing necessary, non-compliance warrants setting aside of the summary judgment The filing of a motion and the conduct of a hearing on the motion are therefore important because these enable the court to determine if the parties’ pleadings, affidavits and exhibits in support of, or against, the motion are sufficient to overcome the opposing papers and adequately justify the finding that, as a matter of law, the claim is clearly meritorious or there is no defense to the action. The non-observance of the procedural requirements of filing a motion and conducting a hearing on the said motion warrants the setting aside of the summary judgment (Eland Philippines, Inc. vs. Garcia, ).

In Calubaquib vs. Republic, , it was held that when the trial court proceeded to render summary judgment with neither of the parties filing a motion therefor it disragarded and ignored the guidelines and safeguards for the rendition of a summary judgment thereby violating the defendant’s due process right to a trial where they can present their evidence and prove their defense. In this case, it was the trial court that directed the plaintiff to file a motion for summary judgment which was opposed by the defendant.

Also in Calubaquib vs. Republic, , it was held that it was improper for the trial court to have persisted in rendering summary judgment; to make a baseless assumption is premature and unfair.

III. DEFENDANT TO FILE MOTION FOR PRELIMINARY HEARING ON HIS AFFIRMATIVE DEFENSE ASSERTED IN THE ANSWER A. The rule Preliminary hearing is not mandatory, but subject to the discretion of the trial court.1 If no motion to dismiss has been filed, any of the grounds for dismissal provided for in this Rule may be pleaded as an affirmative defense in the answer and, in the discretion of the court, a preliminary hearing may be had thereon as if a motion to dismiss had been filed (Section 6, Rule 16, 1997 Revised Rules of Civil Procedure). The allowance for a preliminary hearing, while left in the discretion of the court, is authorized only if no motion to dismiss has been filed but any of the grounds for a motion to dismiss had been pleaded as an affirmative defense in the answer (Rasdas vs. Estenor,). Section 6, Rule 16 of the 1997 Revised Rules of Civil Procedure is explicit in stating that the defendant may reiterate any of the grounds for dismissal provided under Rule 16 of the Rules of Court as affirmative defenses but that a preliminary hearing may no longer be had thereon if a motion to dismiss had already been filed (Abrajano vs. Heirs of Salas, Jr.,).

1

Municipality of Biñan, Laguna vs. CA, 219 SCRA 69, February 17, 1993 and Regalado, Remedial Law Compendium, Vol. 1, pp. 163-164 (1993) cited in California and Hawaiian Sugar Company vs. Pioneer Insurance and Surety Corporation, G.R. No. 139273, November 28, 2000.

B. Preliminary hearing may no longer be had thereon if a motion to dismiss had already been filed, exception The rule does not contemplate a situation where there are several defendants but only one filed a motion to dismiss. Other defendants who did not file a motion to dismiss may still ask and may be allowed for a preliminary hearing on their own affirmative defenses, otherwise it will be prejudicial to other defendants who did not file a motion to dismiss (Abrajano vs. Heirs of Salas,). Not only that, it will violate the due process and equal protection clauses enshrined in our Constitution.

B. Preliminary hearing allowed even an answer is filed after a motion to dismiss was filed and deferred its resolution When a motion to dismiss is filed and its resolution is deferred, and thereafter directed the defendant to file answer, and if answer is filed defendant is still entitled to a preliminary hearing on his affirmative defenses asserted in the Answer (California and Hawaiian Sugar Company vs. Pioneer Insurance and Surety Corporation,). In California and Hawaiian Sugar Company vs. Pioneer Insurance and Surety Corporation, G it was held that defendant is still entitled to preliminary hearing despite of filing a motion to dismiss, because the trial court deferred the resolution of the motion to dismiss and required the defendant to file answer instead. The High Court said in California: “True, Section 6, Rule 16 of the 1997 Rules, specifically provides that a preliminary hearing on the affirmative defenses may be allowed only when no motion to dismiss has been filed. Section 6, however, must be viewed in the light of Section 3 of the same Rule, which requires courts to resolve a motion to dismiss and prohibits them from deferring its resolution on the ground of indubitability. Clearly then, Section 6 disallows a

preliminary hearing of affirmative defenses once a motion to dismiss has been filed because such defense should have already been resolved. In the present case, however, the trial court did not categorically resolve petitioners’ Motion to Dismiss, but merely deferred resolution thereof.” The case of Rasdas vs. Estenor is quite peculiar, because the High Court affirmed the order of dismissal of the case on ground of res judicata pleaded in the answer after a preliminary hearing was conducted. The rationale advanced by the High Court is that disallowing the dismissal would violate the primordial objective of procedural law to secure a just, speedy and inexpensive disposition of every action and proceeding. The High Court said in Rasdas: “We observe at the onset that it does appear that the RTC’s act of staging preliminary hearing on the affirmative defense of lack of jurisdiction and res judicata is not in regular order. Under Section 6, Rule 16 of the 1997 Rules of Civil Procedure, the allowance for a preliminary hearing, while left in the discretion of the court, is authorized only if no motion to dismiss has been filed but any of the grounds for a motion to dismiss had been pleaded as an affirmative defense in the answer. In this case, respondents had filed a motion to dismiss on the ground of res judicata, but the same was denied. They thus filed an answer alleging res judicata as a special affirmative defense, but later presented a Motion for Preliminary Hearing which was granted, leading to the dismissal of the case. The general rule must be reiterated that the preliminary hearing contemplated under Section 6, Rule 16 applies only if no motion to dismiss has been filed. This is expressly provided under the rule, which relevantly states “[i]f no motion to dismiss has been filed, any of the grounds for dismissal provided for in [Rule 16] may be pleaded as an affirmative defense in the answer and, in the discretion of the court, a preliminary hearing may be had thereon as if a motion to dismiss had been filed.” An exception was carved out in California and Hawaiian Sugar

Company v. Pioneer Insurance, wherein the Court noted that while Section 6 disallowed a preliminary hearing of affirmative defenses once a motion to dismiss has been filed, such hearing could nonetheless be had if the trial court had not categorically resolved the motion to dismiss. Such circumstance does not obtain in this case, since the trial court had already categorically denied the motion to dismiss prior to the filing of the answer and the motion for preliminary hearing. We observe in this case that the judge who had earlier denied the motion to dismiss, Hon. Teodulo E. Mirasol, was different from the judge who later authorized the preliminary hearing, Hon. Isaac R. de Alban, a circumstance that bears some light on why the RTC eventually changed its mind on the motion to dismiss. Still, this fact does not sanction the staging of a preliminary hearing on affirmative defenses after the denial of the motion to dismiss. If a judge disagrees with his/her predecessor’s previous ruling denying a motion to dismiss, the proper recourse is not to conduct a preliminary hearing on affirmative defenses, but to utilize the contested ground as part of the basis of the decision on the merits. On the part of the movant whose motion to dismiss had already been filed and denied, the proper remedy is to file a motion for reconsideration of the denial of the motion. If such motion for reconsideration is denied, the ground for the dismissal of the complaint may still be litigated at the trial on the merits. Clearly, the denial of a motion to dismiss does not preclude any future reliance on the grounds relied thereupon. However, nothing in the rules expressly authorizes a preliminary hearing of affirmative defenses once a motion to dismiss has been filed and denied. Thus, the strict application of Section 6, Rule 16 in this case should cause us to rule that the RTC erred in conducting the preliminary hearing.

However, there is an exceptional justification for us to overlook this procedural error and nonetheless affirm the dismissal of the complaint. The complaint in question is so evidently barred by res judicata, it would violate the primordial objective of procedural law to secure a just, speedy and inexpensive disposition of every action and proceeding should the Court allow this prohibited complaint from festering in our judicial system. Indeed, the rule sanctioning the liberal construction of procedural rules is tailor-made for a situation such as this, when a by-thenumbers application of the rule would lead to absurdity, such as the continued litigation of an obviously barred complaint.”

C. Though discretionary, denial of preliminary hearing sometimes constitutes grave abuse of discretion Also in California and Hawaiian Sugar Company vs. Pioneer Insurance and Surety Corporation, , it was held that the trial court gravely abused its discretion when it denied the motion for preliminary hearing, thus: “The more crucial question that we must settle here is whether the trial court committed grave abuse of discretion when it denied petitioners’ Motion for a Preliminary Hearing on their affirmative defense of lack of cause of action. Undeniably, a preliminary hearing is not mandatory, but subject to the discretion of the trial court. In the light of the circumstances in this case, though, we find that the lower court committed grave abuse of discretion in refusing to grant the Motion. We note that the trial court deferred the resolution of petitioners’ Motion to Dismiss because of a single issue. It was apparently unsure whether the charter party that the bill of lading referred to was indeed the Baltimore Berth Grain Charter Party submitted by petitioners.

Considering that there was only one question, which may even be deemed to be the very touchstone of the whole case, the trial court had no cogent reason to deny the Motion for Preliminary Hearing. Indeed, it committed grave abuse of discretion when it denied a preliminary hearing on a simple issue of fact that could have possibly settled the entire case. Verily, where a preliminary hearing appears to suffice, there is no reason to go on to trial. One reason why dockets of trial courts are clogged is the unreasonable refusal to use a process or procedure, like a motion to dismiss, which is designed to abbreviate the resolution of a case.” ---0---

RULE ON DEFAULT There are times that a defendant after service of summons fails to file answer to the complaint. This maybe because he does not know what to do, or maybe because he is not really interested in the suit, he will deliberately disregards the complaint by his inaction because he opt not to be bothered by it. If this happens, the plaintiff is on the advantageous position because if the case is governed by the Rules on Summary Procedure judgment can be rendered as may be warranted by the facts alleged in the complaint under Section 6 of the 1991 Revised Rules on Summary Procedure; and if the case is governed by the regular the defendant can be declared in default under Section 3, Rule 9 of the 1997 Revised Rules of Civil Procedure. In either, his case can be expeditiously disposed of.

A. Cases governed by the Revised Rules on Summary Procedure;remedy when defendant did not file his answer

Section 6 of the 1991 Revised Rules on Summary Procedure provides: “Sec. 6. Effect of failure to answer. – Should the defendant fail to answer the complaint within the period above provided, the court, motu proprio, or on motion of the plaintiff, shall render judgment as may be warranted by the facts alleged in the complaint and limited to what is prayed for therein: xxx xxx xxx.”

B. Basic principles and guidelines; interpretation of the rules, strict rather than liberal; exception The use of the word “shall” in the Rule on Summary Procedure underscores mandatory character, not directory. Giving the provisions a directory application would subvert the nature of the Rule on Summary Procedure and defeat its objective of expediting the adjudication of suits (Gachon vs. Devera,). Liberal interpretation and application of the rules applies only in proper cases and under justifiable causes and circumstances (Gachon vs. Devera, Jr.,). Filing an answer beyond the prescribed period is a ground for the court to render judgment under Section 6 of the 1991 Revised Rules on Summary Procedure, except when the late filing for justifiable causes and circumstances. This is where the liberal interpretation and application of Rules applies (Gachon vs. Devera, Jr., (). The case of Leynes vs. CA is somehow peculiar. Defendant’s last day to file answer fell on Saturday. When his answer was filed the following working day, a Monday, the trial court granted plaintiff’s motion to render judgment pursuant to Section 6 of the 1991 Revised Rules on Summary Procedure holding that on that Saturday there were employees present in court as required by A.M. Circular No. 2-99 primarily to act on petitions for bail and other urgent matters. The High Court ordered that defendant’s answer be admitted.

C. Summary Judgment or Judgment on the Pleading not proper Motions for Summary Judgment or Judgment on the Pleading are not proper. While it is true that motion for motions for summary judgment or judgment on the pleading are not prohibited (Section 16, 1991 Revised Rules on Summary Procedure). The reason is obvious. For summary judgment to apply there must exists a genuine issue (Narra Integrated Corporation vs. CA,). Section 6 of the 1991 Revised Rules on Summary Section applies because defendant’s inaction to the complaint. Same is true with motion for judgment on the pleading. It presupposes that an answer is filed, because a judgment on the pleadings is proper when an answer fails to tender an issue, or otherwise admits the material allegations of the adverse party’s pleading (Municipality of Tiwi vs. Betito,).

D. Cases governed by regular procedure, remedy when defendant did not file his answer A declaration or order of default is issued as a punishment for unnecessary delay in joining issues (Vlason Enterprises Corporation vs. CA,). Section 3, Rule 9 of the 1997 Revised Rules of Civil Procedure provides: Section 3. Default; declaration of. — If the defending party fails to answer within the time allowed therefor, the court shall, upon motion of the claiming party with notice to the defending party, and proof of such failure, declare the defending party in default. Thereupon, the court shall proceed to render judgment granting the claimant such relief as his pleading may warrant, unless the court in its discretion requires the claimant to submit evidence. Such reception of evidence may be delegated to the clerk of court.

From the foregoing procedure, if defendant did not file an answer the trial court cannot motu proprio declare him in default. There must be a motion to that effect. The case of Spouses delos Santos vs. Judge Carpio) enumerated the three requirements to which claiming party must be complied with before the court may declare the defending party in default, to wit: (1) the claiming party must file a motion asking the court to declare the defending party in default; (2) the defending party must be notified of the motion to declare him in default; (3) the claiming party must prove that the defending party has failed to answer within the period provided by the Rule.

E. The claiming party must file a motion asking the court to declare the defending party in default; the defending party must be notified of the motion to declare him in default The claiming party must file a motion asking the court to declare the defending party in default. In short, there must be a motion. The plaintiff must file a motion to that effect. And considering that the trial court cannot resolve the motion without asking the defendant of his comment and/or opposition, the same is considered as motion of litigious character. And therefore, the same must comply with the rules on motions, particularly the requisite notice, service and hearing (See Sections 4, 5 and 6, Rule 15 of the 1997 Revised Rules of Civil Procedure; see also discussions on Rules on Motions). The case of Spouse delos Santos vs. Judge Carpio ) clarified that present rule expressly requires that the motion of the claiming party should be with notice to the defending party. The purpose of a notice of a motion is to avoid surprises on the opposite party and to give him time to study and meet the arguments. The notice of a motion is required when the party has the right to resist the relief sought by the motion and principles of natural justice demand that his right be not affected without an opportunity to be heard (Spouse delos Santos vs. Judge Carpio,). Therefore, as the present rule on default requires the filing of a motion and notice of such motion to the defending party, it is not enough that the

defendant failed to answer the complaint within the reglementary period to be a sufficient ground for declaration in default. The motion must also be heard (Spouse delos Santos vs. Judge Carpio, ).

F. The claiming party must prove that the defending party has failed to answer within the period provided by the Rule To prove this element, the plaintiff must prove that the fifteen day period, or the period fixed by the court for the defendant to file responsive pleading has expired or lapsed. This can be done by giving emphasis to the Sheriff’s Return of Summons as proof that the fifteen day period, or the period fixed by the court for the defendant to file responsive pleading has expired or lapsed.

G. Effect to defendant if declared in default A party in default is entitled to notice of subsequent proceedings but shall not take part in the trial (Section 3 (a), Rule 9, 1997 Revised Rules of Civil Procedure). He loses his standing in court and his right to adduce evidence and to present his defense (Rural Bank of Sta. Catalina vs. Land Bank of the Philippines,); and if pleading is filed, he should expect the trial court to act upon their pleadings ((Vlason Enterprises Corporation vs. CA,).

H. Plaintiff not automatically entitled to the relief prayed for when defendant is declared in default; I. A declaration of default not an admission of the truth or the validity of the plaintiff’s claims; and J. The trial court has no authority to order plaintiff to present evidence ex parte without declaring first the defendant in default

Declaration of default is somehow like a foul shot in a basketball game wherein the free-thrower is allowed to throw his shot free to garn a point with no player allowed to defend the shot. Like a free-throw in a basketball game, the plaintiff in default proceedings is not automatically entitled to the relief prayed for in the complaint. Though plaintiff is allowed to present evidence ex-parte, he is still duty bound to present sufficient evidence to establish his cause of action because failure to do so will result in the denial of his claim or dismissal of his action. The plaintiff is duty bound to establish by competent evidence all the material allegations of his complaint upon which he bases his prayer for relief (Lopez vs. Mendezona, ;Luxuria Homes and/or Posadas vs. CA,), because a judgment by default against a defendant does not imply a waiver of rights except that of being heard and of presenting evidence in his favor (De los Santos v. De la Cruz, cited in Luxuria Homes and/or Posadas vs. CA,). Declaration of default does not imply admission by the defendant of the facts and causes of action of the plaintiff, because the codal section requires the latter to adduce his evidence in support of his allegations as an indispensable condition before final judgment could be given in his favor. Nor could it be interpreted as an admission by the defendant that the plaintiff’s causes of action finds support in the law or that the latter is entitled to the relief prayed for (Luxuria Homes and/or Posadas vs. CA,). A declaration of default is not an admission of the truth or the validity of the plaintiff’s claims (Macondray & Co. vs. Eustaquio,). Although defendant may be declared in default, the plaintiff is still required to substantiate his allegations in the complaint because the judgment of default against defendant who has not appeared or filed his answer does not imply a waiver of all his rights, except his right to be heard and to present evidence in his favor; the defendant’s failure to answer does not imply his admission of the facts and the causes of action of the plaintiff, because plaintiff is still required to adduce evidence to support his allegation, thus the defaulting defendant is still protected by law (Vlason Enterprises Corporation vs. CA,).

Not only that, the only evidence that may be allowed by the trial court are those evidence that will tend to support the relief sought and specified in the complaint (Vlason Enterprises Corporation vs. CA), and no award greater than or different in kind from that specified in the complaint can be granted (Vlason Enterprises Corporation vs. CA,), and if a relief other than that specifically prayed for in the complaint is awarded in a judgment by default, such an award is null and void because the court is bereft of jurisdiction to grant such an award (Policarpio vs. RTC of Quezon City,). Vlason Enterprises Corporation vs. CA (made to emphasize that without a declaration that petitioner is in default as required in Section 1, Rule 18 of the Rules of Court, the trial court has no authority to order the presentation of evidence ex parte against petitioner to render judgment against it by default. Once the defendant is declared in default the plaintiff is not automatically entitled to the relief prayed for. The court can grant the favorable relief sought only after it has ascertained that the evidence offered and the facts proven by the presenting party warrant the grant of the same (Pascua vs. Florendo cited in Luxuria Homes and/or Posadas vs. CA,), because it would be meaningless to require presentation of evidence if everytime the other party is declared in default, a decision would automatically be rendered in favor of the non-defaulting party and exactly according to the tenor of his prayer (Luxuria Homes and/or Posadas vs.). A defaulted defendant is not actually thrown out of court. The rules see to it that any judgment against him must be in accordance with law. The evidence to support the plaintiff’s cause is, of course, presented in his absence, but the court is not supposed to admit that which is basically incompetent. Although the defendant would not be in a position to object, elementary justice requires that only legal evidence should be considered against him. If the evidence presented should not be sufficient to justify a judgment for the plaintiff, the complaint must be dismissed. And if an unfavorable judgment should be justifiable, it cannot exceed the amount or be different in kind from what is prayed for in the complaint (Luxuria Homes and/or Posadas vs. CA,).

K. Plaintiff must prove his cause by preponderance of evidence In civil cases, the party having the burden of proof must establish his case by a preponderance of evidence. In determining where the preponderance or superior weight of evidence on the issues involved lies, the court may consider all the facts and circumstances of the case, the witnesses’ manner of testifying, their intelligence, their means and opportunity of knowing the facts to which they are testifying, the nature of the facts to which they testify, the probability or improbability of their testimony, their interest or want of interest, and also their personal credibility so far as the same may legitimately appear upon the trial. The court may also consider the number of witnesses, though the preponderance is not necessarily with the greater number (Section 1, Rule 133, Rules of Court). Plaintiff is not excused from not establishing his claims alleged in the complaint by the required quantum of proof under Section 1 of Rule 133; any advantage they he have gained from the ex parte presentation of evidence does not lower the degree of proof required (Gajudo, et al., vs. Traders Royal Bank,).

L. Section 3 of Rule 9 and Section 1 of Rule 133 of the Rules of Court are not incompatible with each other In one case the quantum of evidence for judgments flowing from a default order under Section 3 of Rule 9 was raised as it is not the same as that provided for in Section 1 of Rule 133. This issue had already been clarified by the High Court in Gajudo, et al., vs. Traders Royal Bank holding that Section 3 of Rule 9 and Section 1 of Rule 133 of the Rules of Court are not incompatible with each other. The High Court said: “For ease of discussion, these two rules will be reproduced below, starting with Section 3 of Rule 9 of the Rules of Court:

“Sec. 3. Default; declaration of. – If the defending party fails to answer within the time allowed therefor, the court shall, upon motion of the claiming party with notice to the defending party, and proof of such failure, declare the defending party in default. Thereupon, the court shall proceed to render judgment granting the claimant such relief as his pleading may warrant, unless the court in its discretion requires the claimant to submit evidence. Such reception of evidence may be delegated to the clerk of court. “(a) Effect of order of default. – A party in default shall be entitled to notice of subsequent proceedings but not to take part in the trial. “(b) Relief from order of default. – A party declared in default may at any time after notice thereof and before judgment file a motion under oath to set aside the order of default upon proper showing that his failure to answer was due to fraud, accident, mistake or excusable negligence and that he has a meritorious defense. In such case, the order of default may be set aside on such terms and conditions as the judge may impose in the interest of justice. “(c) Effect of partial default. – When a pleading asserting a claim states a common cause of action against several defending parties, some of whom answer and the others fail to do so, the court shall try the case against all upon the answers thus filed and render judgment upon the evidence presented. “(d) Extent of relief to be awarded. – A judgment rendered against a party in default

shall not exceed the amount or be different in kind from that prayed for nor award unliquidated damages. “(e) Where no defaults allowed. – If the defending party in an action for annulment or declaration of nullity of marriage or for legal separation fails to answer, the court shall order the prosecuting attorney to investigate whether or nor a collusion between the parties exists, and if there is no collusion, to intervene for the State in order to see to it that the evidence submitted is not fabricated.” We now quote Section 1 of Rule 133: “SECTION 1. Preponderance of evidence, how determined. – In civil cases, the party having the burden of proof must establish his case by a preponderance of evidence. In determining where the preponderance or superior weight of evidence on the issues involved lies, the court may consider all the facts and circumstances of the case, the witnesses’ manner of testifying, their intelligence, their means and opportunity of knowing the facts to which they are testifying, the nature of the facts to which they testify, the probability or improbability of their testimony, their interest or want of interest, and also their personal credibility so far as the same may legitimately appear upon the trial. The court may also consider the number of witnesses, though the preponderance is not necessarily with the greater number.” Between the two rules, there is no incompatibility that would preclude the application of either one of them. To begin with, Section 3 of Rule 9 governs the procedure

which the trial court is directed to take when a defendant fails to file an answer. According to this provision, the court “shall proceed to render judgment granting the claimant such relief as his pleading may warrant,” subject to the court’s discretion on whether to require the presentation of evidence ex parte. The same provision also sets down guidelines on the nature and extent of the relief that may be granted. In particular, the court’s judgment “shall not exceed the amount or be different in kind from that prayed for nor award unliquidated damages.” As in other civil cases, basic is the rule that the party making allegations has the burden of proving them by a preponderance of evidence. Moreover, parties must rely on the strength of their own evidence, not upon the weakness of the defense offered by their opponent. This principle holds true, especially when the latter has had no opportunity to present evidence because of a default order. Needless to say, the extent of the relief that may be granted can only be as much as has been alleged and proved with preponderant evidence required under Section 1 of Rule 133.” (Citations Omitted).

M. Judgment by default cannot grant relief not prayed for; exception, when filed an answer but were absent during trial Vlason Enterprises Corporation vs. CA, , said that judgment by default cannot grant relief not prayed for; exception, when filed an answer but were absent during trial. The High Court in Vlason: “A declaration or order of default is issued as a punishment for unnecessary delay in joining issues. In such event, defendants lose their standing in court, they cannot expect the trial court to act upon their pleadings, and they are not entitled to notice of the proceeding until the final termination of the case. Thus, the trial court proceeds

with the reception of the plaintiff’s evidence upon which a default judgment is rendered. Section 1 of Rule 18 provides that after the defendant has been declared in default, “the court shall proceed to receive the plaintiff’s evidence and render judgment granting him such relief as the complaint and the facts proven may warrant.” The reliefs that may be granted, however, are restricted by Section 5, which provides that a judgment entered against a party in default shall not exceed the amount or be different in kind from that prayed for. In other words, under Section 1, a declaration of default is not an admission of the truth or the validity of the plaintiff’s claims. The claimant must still prove his claim and present evidence. In this sense the law gives defaulting parties some measure of protection because plaintiffs, despite the default of defendants, are still required to substantiate their allegations in the complaint. The judgment of default against defendants who have not appeared or filed their answers does not imply a waiver of all their rights, except their right to be heard and to present evidence in their favor. Their failure to answer does not imply their admission of the facts and the causes of action of the plaintiffs, because the latter are required to adduce evidence to support their allegations. Moreover, the trial court is not allowed by the Rules to receive evidence that tends to show a relief not sought or specified in the pleadings. The plaintiff cannot be granted an award greater than or different in kind from that specified in the complaint. This case should be distinguished, however, from that of defendants, who filed an answer but were absent during trial. In that case, they can be held liable for an amount greater than or different from that originally prayed for, provided that the award is warranted by the proven facts. This rule is premised on the theory that the adverse party

failed to object to evidence relating to an issue not raised in the pleadings.” (Underscoring Supplied)

N. A default judgment only affects defaulted defendant A default judgment against several defendants cannot affect the rights of the other defendants who are not declared in default. The trial court has no authority to allow plaintiff to evidence ex parte against a defendant is not declared in default. To do otherwise will result to a manifest failure or miscarriage of justice (Vlason enterprises Corporation vs. CA,).

O. A default judgment cannot includean award not prayed for in the complaint A judgment in default proceeding cannot include an award not prayed for in the complaint, even if proven ex parte, because the trial court is not allowed by the Rules to receive evidence that tends to show a relief not sought or specified in the pleadings and the plaintiff cannot be granted an award greater than or different in kind from that specified in the complaint (Vlason enterprises Corporation vs. CA,). P. Remedy of defendant if plaintiff’s motion todeclare defendant in default is granted Settled is the rule that a party in default is entitled to notice of subsequent proceedings but shall not take part in the trial; that he loses his standing in court and his right to adduce evidence and to present his defense, and if pleading is filed he should expect the trial court to act upon their pleadings (Section 3 (a), Rule 9, 1997 Revised Rules of Civil Procedure; Rural Bank of Sta. Catalina vs. Land Bank of the Philippines,). Nevertheless, a party in default has still available remedies. The following are the remedial measures available to defendants in case he is ordered declared in default:

(1) a motion to set aside the order of default under Section 3(b), Rule 9 of the Rules of Court, if the default was discovered before judgment could be rendered; (2) a motion for new trial under Section 1(a) of Rule 37, if the default was discovered after judgment but while appeal is still available; (3) a petition for relief under Rule 38, if judgment has become final and executory; and (4) an appeal from the judgment under Section 1, Rule 41, even if no petition to set aside the order of default has been resorted to (Lina vs. CA, 135 SCRA 637 cited in Indiana Aerospace University vs. CHED, G.R. No. 139371, April 4, 2001). Martinez vs. Republic () allows the filing of appeal under Section 1, Rule 41 of the 1997 Revised Rules of Civil Procedure on the ground that the plaintiff failed to prove the material allegations of the complaint, or that the decision is contrary to law, even without need of the prior filing of a motion to set aside the order of default. In short, there is no need to secure an order lifting an order of default. In Rural Bank of Sta. Catalina vs. LBP, the High Court said: It bears stressing that a defending party declared in default loses his standing in court and his right to adduce evidence and to present his defense. He, however, has the right to appeal from the judgment by default and assail said judgment on the ground, inter alia, that the amount of the judgment is excessive or is different in kind from that prayed for, or that the plaintiff failed to prove the material allegations of his complaint, or that the decision is contrary to law. Such party declare in default is proscribed from seeking a modification or reversal of the assailed decision on the basis of the evidence submitted by him in the Court of Appeals, for if it were otherwise, he would thereby be

allowed to regain his right to adduce evidence, a right which he lost in the trial court when he was declare in default, and which he failed to have vacated. In this case, the petitioner sought the modification of the decision of the trial court based on the evidence submitted by it only in the Court of Appeals.” (Author’s observation: there is no need to secure an order of lifting an order of default because filing a notice of appeal is considered as a post judgment remedy). The foregoing remedial measures is available to party in default if the order declaring him in default is not a patent nullity or void, because if the order declaring him in default is a clear and patent nullity, his available remedy is certiorari under Rule 65. Certiorari under Rule 65. Why certiorari under Rule 65? The explanation is so simple. An order granting a motion to declare defendant in default is interlocutory nature (Indiana Aerospace University vs. CHED,). This is because the order granting a motion to declare defendant in default does not disposes of the case. Such an order is not a final order, because it leaves something to be done by the court before the case is finally decided on the merits (Philgreen Trading Construction Corporation vs. CA,). Therefore the same cannot be a subject of an appeal under Section 1, Rule 41 of the 1997 Revised Rules of Civil Procedure. Paragraph 2(c), Section 1, Rule 41, 1997 Revised Rules of Civil Procedure provides that no appeal may be taken from an interlocutory order. Therefore, the only available remedy is Certiorari under Rule 65 (Cerezo vs. David Tuazon,). Certiorari is a remedy when any tribunal, board or officer exercising judicial or quasi-judicial functions has acted without or in excess its or his jurisdiction, or with grave abuse of discretion amounting to lack or excess of jurisdiction, and there is no appeal, or any plain,

speedy, and adequate remedy in the ordinary course of law, a person aggrieved thereby may file a verified petition in the proper court, alleging the facts with certainty and praying that judgment be rendered annulling or modifying the proceedings of such tribunal, board or officer, and granting such incidental reliefs as law and justice may require (Section 1, Rule 65, 1997 Revised Rules of Civil Procedure). In Guanzon vs. Arradaza, G., the High Court said that a petition for certiorari under Rule 65 to declare the nullity of judgment by default is also available if the trial court improperly decalred a party in default, or even if the trial court properly decalred a party in default, if grave abuse of discretion attended such declaration.

However, there is no harm if he will avail of the foregoing remedial measures laid down in Lina case, but he may no longer later on question the patent nullity of the order because he may deemed to have waive his right to question the fact of nullity. Same remedial measure also is available to plaintiff if his motion to declare defendant in default is denied, or against order setting aside order of default, because an order setting aside order of default is interlocutory (Denso (Phils.), Inc., vs. IAC,). In Indiana Aerospace University vs. CHED,), the subject of the action is CHEd’s regulation or administration of educational institutions, and therefore, treated as action imbued with public interest, hence, the filing of the requisite motion for reconsideration was dispensed with (Liberty Insurance Corporation vs. CA,). The case of Spouse delos Santos vs. Judge Carpio) is also peculiar in the sense that in this case, the trial judge motu proprio issued an order declared defendant in default without waiting for the scheduled hearing on the motion to declare defendant in default. That’s foul according to the High Court, and said: “We could not see any justifiable reason why the trial court chose not to hear the petitioners on the date and time fixed in Metrobank’s motion, and instead, hastily

granted the motion before it could be heard on the ground that it had found the motion to be impressed with merit. Indeed, in totally disregarding the purpose for which the filing of a motion and notice to defending party are required by the Rules, the trial court had acted in a despotic manner that is correctly assailed through a petition for certiorari which petitioners have seasonably filed with the CA.” “Again, respondent Judge acted capriciously when he totally ignored petitioners’ Opposition to Metrobank’s Motion to Declare them in Default and denied their Motion to Admit Answer, both filed on February 15, 2001, a day before the scheduled hearing, which showed their desire to be heard before the motion to declare them in default is resolved by the trial court.” Q. Resorting of plaintiff to filing of motion for summary judgment or judgment on the pleading not proper Motions for summary judgment or judgment on the pleading are not proper. Again, the reason is obvious. For these remedies to apply, it presupposes that an answer is filed. For summary judgment to apply there must exists a genuine issue (Narra Integrated Corporation vs. CA,). Judgment on the pleadings is proper when an answer fails to tender an issue, or otherwise admits the material allegations of the adverse party’s pleading (Municipality of Tiwi vs. Betito,).

R. Requirements must comply with in filing motion to lift order of default The defendant in default may, at any time after discovery thereof and before judgment, file a motion under oath to set aside the order of default on the ground that his failure to answer was due to fraud, accident, mistake or excusable negligence, and that he has a meritorious defense (Section 3(b), Rule 9, 1997 Revised Rules of Civil Procedure).

In short, the motion must be under oath, and must contain an allegation that the defendant’s failure to file his answer was due to fraud, accident, mistake, or excusable negligence.

S. Certiorari (under Rule 65) not always grantable against an order denying motion to lift order of default In David vs. Judge Gutierrez-Fruelda, , it was held that the trial judge did not commit grave abuse of discretion when she denied the motion to lift order of default on ground the motion was not under oath. The motion did not also contain an allegation that the defendant’s failure to file his answer was due to fraud, accident, mistake, or excusable negligence. T. Remedy of plaintiff if his motion to declare defendant in default is denied If there is grave abuse of discretion in denying the motion to declare defendant in default, petition for certiorari under Rule 65 is the remedy. Reason: an order denying the motion to declare defendant in default is an interlocutory order.

U. Rule on default in counterclaim If the defendant (plaintiff in the complaint) fails within the reglementary period, that is, ten (10) days from service (Section 4, Rule 11, 1997 Revised Rules of Civil Procedure) within which to file an answer, he, upon motion of the counterclaimant (or the plaintiff in the counterclaim) may be declared in default (Section 3, Rule 9, 1997 Rules of Civil Procedure; Francisco Motors Corporation vs. CA,). ---0---

DEMURRER TO EVIDENCE A. Demurrer to evidence, definition, concept and purpose

It is defined as an objection by one of the parties in an action, to the effect that the evidence which his adversary produced is insufficient in point of law, whether true or not, to make out a case or sustain the issue (Gutib vs. CA, G). It authorizes a judgment on the merits of the case without the defendant having to submit evidence on his part as he would ordinarily have to do, if plaintiff’s evidence shows that he is not entitled to the relief sought (Dandoy vs. CA,). It is an aid or instrument for the expeditious termination of an action, similar to a motion to dismiss, which the court or tribunal may either grant or deny (Dandoy vs. CA,). Its purpose is precisely to expeditiously terminate the case without the need of the defendant’s evidence (Spouses Condes vs. CA,).

B. May be issued, when? A demurrer to evidence may be issued when, upon the facts adduced and the applicable law, the plaintiff has shown no right to relief. Where the totality of plaintiff’s evidence, together with such inferences and conclusions as may reasonably be drawn therefrom, does not warrant recovery against the defendant, a demurrer to evidence should be sustained (Dandoy vs. CA,). It may be granted if, after the presentation of plaintiff’s evidence, it appears upon the facts and the law that the plaintiff has shown no right to relief (Republic vs. Tuvera,). A demurrer to evidence is likewise sustainable when, admitting every proven fact favorable to the plaintiff and indulging in his favor all conclusions fairly and reasonably inferable therefrom, the plaintiff has failed to make out one or more of the material elements of his case, or when there is no evidence to support an allegation necessary to his claim. It should be sustained where the plaintiff’s evidence is prima facie insufficient for a recovery (Dandoy vs. CA,).

It is, therefore, premature to speak of “preponderance of evidence” in a demurrer to evidence because it is filed before the defendant presents his evidence (Spouses Condes vs. CA).

C. When to file? After the plaintiff has completed the presentation of his evidence, the defendant may move for dismissal on the ground that upon the facts and the law the plaintiff has shown no right to relief. If his motion is denied, he shall have the right to present evidence. If the motion is granted but on appeal the order of dismissal is reversed he shall be deemed to have waived the right to present evidence (Section 1, Rule 33, 1997 Revised Rules of Civil Procedure).

D. Resolving a demurrer to evidence, basis What should be resolved in a motion to dismiss based on a demurrer to evidence is whether the plaintiff is entitled to the relief based on the facts and the law. The evidence contemplated by the rule on demurrer is that which pertains to the merits of the case, excluding technical aspects such as capacity to sue (Casent Realty Development Corp. vs. Philbanking Corporation,). The plaintiff’s evidence should not be the only basis in resolving a demurrer to evidence. The “facts” referred to in Section 8, Rule 8 of the Rules of Court should include all the means sanctioned by the Rules of Court in ascertaining matters in judicial proceedings. These include judicial admissions, matters of judicial notice, stipulations made during the pre-trial and trial, admissions, and presumptions, the only exclusion being the defendant’s evidence (Casent Realty Development Corp. vs. Philbanking Corporation,). In short, Courts may consider other facts within the range of judicial notice as well as relevant laws and jurisprudence which the courts are bound to take into account, and they are also fairly entitled to examine records/documents duly incorporated into the complaint by the pleader himself in ruling on the demurrer to the complaint (U. Bañez Electric Light Company vs. Abra Electric Cooperative, Inc., et al.,).

E. Section 8, Rule 8 of the Rules of Court When an action or defense is founded upon a written instrument, copied in or attached to the corresponding pleading as provided in the preceding section, the genuineness and due execution of the instrument shall be deemed admitted unless the adverse party, under oath, specifically denies them, and sets forth, what he claims to be the facts; but the requirement of an oath does not apply when the adverse party does not appear to be a party to the instrument or when compliance with an order for an inspection of the original instrument is refused2 (See Casent Realty Development Corp. vs. Philbanking Corporation,).

F. Judicial admission also considered in resolving the demurrer to evidence An admission, verbal or written, made by a party in the course of the proceeding in the same case, does not require proof. The admission may be contradicted only by showing that it was made through palpable mistake or that no such admission was made3 (See Casent Realty Development Corp. vs. Philbanking Corporation,). In Casent Realty Development Corp. vs. Philbanking Corporation, , it was held that court must consider the deemed admitted genuineness and due execution of the Dacion and Confirmation Statement under oath in resolving the demurrer to evidence. The High Court said in Casent: “On appeal to the CA, respondent claimed that even though it failed to file a Reply, all the new matters alleged in the Answer are deemed controverted anyway, pursuant to Rule 6, Section 10:

2

Section 8. How to contest such documents. Section 4. Judicial admissions.

3

Section 10. Reply.––A reply is a pleading, the office or function of which is to deny, or allege facts in denial or avoidance of new matters alleged by way of defense in the answer and thereby join or make issue as to such new matters. If a party does not file such reply, all the new matters alleged in the answer are deemed controverted. We agree with petitioner. Rule 8, Section 8 specifically applies to actions or defenses founded upon a written instrument and provides the manner of denying it. It is more controlling than Rule 6, Section 10 which merely provides the effect of failure to file a Reply. Thus, where the defense in the Answer is based on an actionable document, a Reply specifically denying it under oath must be made; otherwise, the genuineness and due execution of the document will be deemed admitted. Since respondent failed to deny the genuineness and due execution of the Dacion and Confirmation Statement under oath, then these are deemed admitted and must be considered by the court in resolving the demurrer to evidence. We held in Philippine American General Insurance Co., Inc. v. Sweet Lines, Inc. that “[w]hen the due execution and genuineness of an instrument are deemed admitted because of the adverse party’s failure to make a specific verified denial thereof, the instrument need not be presented formally in evidence for it may be considered an admitted fact.”

G. Res judicata ground for demurrer to evidence, inappropriate Res judicata is an inappropriate ground for sustaining a demurrer to evidence, even as it stands as a proper ground for a motion to dismiss. A demurrer may be granted if, after the presentation of plaintiff’s evidence, it appears upon the facts and the law that the plaintiff has shown no right to relief. In contrast, the grounds for res judicata present themselves even before the presentation of evidence, and it should be at that stage that the defense of res judicata should be invoked as a ground

for dismissal. Properly speaking, the movants for demurral who wish to rely on a controlling value of a settled case as a ground for demurrer should invoke the ground of stare decisis in lieu of res judicata (Republic vs. Tuvera, G).

H. Effect of judgment on demurrer to evidence Again, after the plaintiff has completed the presentation of his evidence, the defendant may move for dismissal on the ground that upon the facts and the law the plaintiff has shown no right to relief. If his motion is denied, he shall have the right to present evidence. If the motion is granted but on appeal the order of dismissal is reversed he shall have be deemed to have waived the right to present evidence (Section 1, Rule 33, 1997 Revised Rules of Civil Procedure). The general rule is that upon the dismissal of the demurrer in the appellate court, the defendant loses the right to present his evidence and the appellate court shall then proceed to render judgment on the merits on the basis of plaintiff’s evidence (Republic vs. Tuvera,). The rationale behind the rule and doctrine is simple and logical. The defendant is permitted, without waiving his right to offer evidence in the event that his motion is not granted, to move for a dismissal (i.e., demur to the plaintiff’s evidence) on the ground that upon the facts as thus established and the applicable law, the plaintiff has shown no right to relief. If the trial court denies the dismissal motion, i.e., finds that plaintiff’s evidence is sufficient for an award of judgment in the absence of contrary evidence, the case still remains before the trial court which should then proceed to hear and receive the defendant’s evidence so that all the facts and evidence of the contending parties may be properly placed before it for adjudication as well as before the appellate courts, in case of appeal. Nothing is lost. The doctrine is but in line with the established procedural precepts in the conduct of trials that the trial court liberally receive all proffered evidence at the trial to enable it to render its decision with all possibly relevant proofs in the record, thus assuring that the appellate courts upon appeal have all the material before them necessary to make a correct judgment, and avoiding the need of remanding the case for retrial or reception of improperly excluded evidence, with the

possibility thereafter of still another appeal, with all the concomitant delays. The rule, however, imposes the condition by the same token that if his demurrer is granted by the trial court, and the order of dismissal is reversed on appeal, the movant loses his right to present evidence in his behalf and he shall have been deemed to have elected to stand on the insufficiency of plaintiff’s case and evidence. In such event, the appellate court which reverses the order of dismissal shall proceed to render judgment on the merits on the basis of plaintiff’s evidence (Republic vs. Tuvera,).

I. Judicial action on demurrer to evidence is discretionary A judicial action on a motion to dismiss on demurrer to evidence rests within the sound discretion of the court. In addition, an order denying a demurrer to evidence is interlocutory. It is not appealable. Neither can it be the subject of a petition for certiorari in the absence of grave abuse of discretion or excess of jurisdiction, or an oppressive exercise of judicial authority (Katigbak vs. Sandiganbayan,). As experienced in the Bench, petitions for certiorari under Rule 65 against the denial of demurrer to evidence (in criminal cases) are usually filed by aggrieved parties arguing that a one or two liner sentences denying the demurrer and the motion for reconsideration for utter lack of merit violates Section 1, Rule 36 of the 1997 Revised Rules of Civil Procedure, and Section 14, Article VIII of the 1987 Constitution. Section 14, Article VIII of the 1987 Constitution provides that “No decision shall be rendered by any court without expressing therein clearly and distinctly the facts and the law on which it is based.” Section 1, Rule 36 of the 1997 Revised Rules of Civil Procedure provides that “A judgment or final order determining the merits of the case shall be in writing personally and directly prepared by the judge, stating clearly and distinctly the facts and the law on which is it is based, signed by him, and filed with the clerk of court.”

This is not a good ground of filing a petition for certiorari under Rule 65. An Order resolving a demurrer to evidence has double-character: first, a final order, and second, as an interlocutory order. If it is a final order, the, the remedy available to the aggrieved party is appeal, and if it is an interlocutory order, the remedy is petition for certiorari under Rule 65. The relevance of having this matter discussed herein is to determine the importance of observing the decision writing foundations provided in Section 14, Article VIII of the 1987 Constitution and Section 1, Rule 36 of the 1997 Revised Rules of Civil Procedure. The requirement of specificity of rulings is stringently applied only to judgments and final orders,4 but not to interlocutory one. If the order denying a demurrer to evidence and the motion for reconsideration for the denial do not contain clearly and distinctly the facts and the law on which it is based, it is believed that constitutional and procedural requirements are not violated because the order is not in the nature of a final order or judgment that completely disposing of the case. Judicial technique is sometimes observed by some courts wherein they will just issue an order of outright denial of demurrer to evidence in order bit the pressing deadline to resolve pending incidents because of tremendous judicial paper works on equally important cases rather than disposing the case on the merit which undeniably requires more precious time doing research work. An order granting a demurrer to evidence, both in civil and criminal cases, partake the nature of a final order, because nothing is left to the court that issued the order. The issuance of this type of order is the order (final character) that the trial judge should comply with the requisite constitutional and procedural mandates. This, the trial judge must state clearly and distinctly the facts and the law on which is it is based. The rationale behind why the final order should state clearly and distinctly the facts and the law on which is it is based is because for the aggrieved party to easily know why he lost, so that he can easily evaluate what error or errors that he can assign or impute to the court that issued it if he desires 4

Dandoy vs. CA, G.R. No. 150089, August 28, 2007.

and permitted to elevate the matter on appeal. This is the reason why the remedy available to the aggrieved party is appeal and not certiorari under Rule 65. On the other hand, an order denying a demurrer to evidence is interlocutory order. The constitutional provision does not apply to interlocutory order, because after the issuance of the order of denial there will be continuing proceedings that will takes place, i.e., actions/proceedings left to be done by the court, such as such as presentation of defense evidence. This is the reason why the remedy available under paragraph 2 (c), in relation to paragraph 3, Section 1,5 Rule 41, 1997 Revised Rules of Civil Procedure is certiorari, and not appeal. Besides, the order of denial is not subject to the requirement of Section 14, Article VIII of the 1987 Constitution, because as what the High Court held in Dandoy vs. CA, the order of denial neither terminate nor finally dispose of the case as there are proceedings still left to be done by the court before the case is finally decided on the merits.

I. An order denying the demurrer to evidence reviewable by Certiorari under Section 1, Rule 65 of the Rules of Court, and when not Generally, the order denying the motion for leave of court to file demurrer to evidence or the demurrer itself shall not be reviewable by appeal or by certiorari before judgment,6 because action on a demurrer or on a motion to dismiss rests on the sound exercise of judicial discretion.7 It

“No appeal may be taken from: (c) An interlocutory order.” “In all the above instances where judgment or final order is not appealable, the aggrieved party may file an appropriate special civil action under Rule 65. 6 Section 23, Rule 119, 2000 Revised Rules of Criminal Procedure. 7 Tan vs. Court of Appeals, 347 Phil. 320, 329 (1997); Bernardo vs. Court of Appeals, 344 Phil. 335, 346 (1997) cited in Nicolas vs. Sandiganbayan and the companion case, G.R. Nos. 175930-31, February 11, 2008. Also in People vs. Singh, G.R. No. 129782, June 29, 2001, 360 SCRA 404; People vs. Mercado, No. L-33492, March 30, 1988, 159 SCRA 453 cited in People vs. Almendras, G.R. No. 145915, April 24, 2003. 5

is an interlocutory order, not appealable neither can it be the subject of a petition for certiorari.8 However, this admits an exception: when the denial of a demurrer to evidence is attended by grave abuse of discretion, patently erroneous or issued with grave abuse of discretion.9 Though interlocutory in character, an order denying a demurrer to evidence may be the subject of a certiorari proceeding, provided the petitioner can show that it was issued with grave abuse of discretion; and that appeal in due course is not plain, adequate or speedy under the circumstances. When the plaintiff’s evidence is utterly and patently insufficient to prove the complaint, it would be capricious for a trial judge to deny the demurrer and to require the defendant to present evidence to controvert a non- existing case. The denial of the demurrer to evidence will constitute an unwelcome imposition on the court’s docket and an assault on the defendant’s resources and peace of mind, and if denied, it effectively denies justice.10 ---0---

POST JUDGMENT REMEDIES Fairness dictates that a party who has not appealed from a judgment of the trial court is bound by the terms of the judgment.11 Let’s say plaintiff, the party that prevails in the decision, upon finality of the judgment he should move to execute the same to enjoy the fruits and result of his litigation. But, how if the loosing party wants to exhaust other remedies to protect his interest subject of the suit, what are his available remedies?

8

David vs. Rivera, 464 Phil. 1006; Tadeo vs. People, 360 Phil. 914, 919 (1998); Cruz vs. People, 363 Phil. 156; Katigbak vs. Sandiganbayan, 453 Phil. 515 cited in Nicolas vs. Sandiganbayan and the companion case, G.R. Nos. 175930-31, February 11, 2008. 9 People vs. Ong, G.R. No. 140904, October 9, 2000 citing Cruz vs. People (303 SCRA 533 [1999]). 10 Choa vs. Choa, G.R. No. 143376, November 26, 2002. 11 Five Star Bus, Co, Inc. vs. CA, G.R. No. 120496, July 17, 1996.

Depending upon each applicability requirement, the following are the remedies available to the loosing or aggrieved party, namely: 1. 2. 3. 4. 5. 6.

execution of judgment appeal motion for reconsideration motion for new trial petition for relief from judgment annulment of judgment

EXECUTION OF JUDGMENT (Rule 39) Litigation must end and terminate sometime and somewhere, and it is essential to an effective and efficient administration of justice that once a judgment has become final, the winning party be not, through a mere subterfuge, deprived of the fruits of the verdict (Li Kim Tho vs. Sanchez,). Once a judgment becomes final, it is basic that the prevailing party is entitled as a matter of right to a writ of execution the issuance of which is the trial court’s ministerial duty (Torno vs. IAC,). Final and executory judgments are enforced by a writ of execution not by mandamus.12 Execution is the fruit and end of the suit and is very aptly called the life of the law.13 It is the process which carries into effect a decree or judgment.14 Execution of a judgment can be issued only against a party to the action and not against one who did not have his day in court (St. Dominic Corporation vs. IAC,), and therefore, a strangers to a case are not bound by the judgment rendered by a court. It will not divest the rights of a party

12

Vital-Gozon vs. CA, G.R. No. 101428, August 5, 1992. Ipekdjian Merchandising Co. vs. Court of Tax Appeals, 8 SCRA 59 cited in PAL, Inc. vs. CA, G.R. No. L-49188, January 30, 1990. 14 Painter v. Berglund, 31 Cal. App. 2d. 63, 87 P 2d 360, 363; Miller v. London, 294 Mass 300, 1 NE 2d 198, 200; Black's Law Dictionary cited in PAL, Inc. vs. CA, G.R. No. L49188, January 30, 1990. 13

who has not and never been a party to a litigation (Panotes vs. City Townhouse Development Corporation,).

Execution of judgment vs. Cash The sheriff shall demand judgment obligor the immediate payment in CASH of the full stated in the writ of execution and all lawful fees. He is not required to give the judgment debtor some time to raise cash because the property may be placed in danger of being lost or absconded, otherwise the sheriff may be administratively disciplined (Torres vs. Cabling, July 11, 1997). The executing sheriff is prohibited from demanding payment by check payable to him (Section 9 (a), Rule 39, 1997 Revised Rules of Civil Procedure).

A. Third party claim When a third person claims interest in the property levied upon, his remedy is to file a “third party claim” (or “tercera). When the sheriff seizes property of a third person in which the judgment debtor holds no right or interest, the supervisory power of the court which has authorized execution may be invoked by the third person. Upon due application by the third person, and after summary hearing, the court may command that the property be released from the mistaken levy and restored to the rightful owner or possessor. If that is the case, the court’s duty is limited to determine whether the sheriff has acted rightly or wrongly in the performance of his duties in the execution of judgment, more specifically, if the sheriff has indeed taken hold of the property not belonging to the judgment debtor. The court does not and cannot pass upon the question of title to the property, with any character of finality; it can treat that matter only in so far as may be necessary to decide if the sheriff has acted correctly or not (Quebral vs. Garduno,).

The court can require the sheriff to restore the property to the claimant’s possession if warranted by the evidence; if claimant’s proofs do not however persuade the court his title or right of possession thereof, his claim will be denied (Ong vs. Tating, G).

B. Execution of Judgment vs. Bank Deposits Execution of judgment against bank deposits is through garnishment. Garnishment has been defined as a species of attachment for reaching any property or credits pertaining or payable to a judgment debtor. In legal contemplation, it is a forced novation by the substitution of creditors: the judgment debtor, who is the original creditor of the garnishee is, through service of the writ of garnishment, substituted by the judgment creditor who thereby becomes creditor of the garnishee. Garnishment has also been described as a warning to a person having in his possession property or credits of the judgment debtor, not to pay the money or deliver the property to the latter, but rather to appear and answer the plaintiff's suit (Perla Compania De Seguros, Inc., vs. Judge Ramolete,).

C. Alias writ of execution be issued without a prior return of the original writ by the implementing officer The issue of whether an alias writ of execution be issued without a prior return of the original writ by the implementing officer had already been settled by the High Court in PAL, Inc. vs. CA,. Resolving in the affirmative, the High Court said in PAL: “We rule in the affirmative and we quote the respondent court's decision with approval: "The issuance of the questioned alias writ of execution under the circumstances here obtaining is justified because even with the absence of a Sheriff's return on the original writ,

the unalterable fact remains that such a return is incapable of being obtained (sic) because the officer who is to make the said return has absconded and cannot be brought to the Court despite the earlier order of the court for him to appear for this purpose. (Order of Feb. 21, 1978, Annex C, Petition). Obviously, taking cognizance of this circumstance, the order of May 11, 1978 directing the issuance of an alias writ was therefore issued. (Annex D, Petition). The need for such a return as a condition precedent for the issuance of an alias writ was justifiably dispensed with by the court below and its action in this regard meets with our concurrence. A contrary view will produce an abhorent situation whereby the mischief of an erring officer of the court could be utilized to impede indefinitely the undisputed and awarded rights which a prevailing party rightfully deserves to obtain and with dispatch. The final judgment in this case should not indeed be permitted to become illusory or incapable of execution for an indefinite and over extended period, as had already transpired." (Rollo, pp. 35-36) Judicium non debet esse illusorium; suum effectum habere debet (A judgment ought not to be illusory; it ought to have its proper effect). Indeed, technicality cannot be countenanced to defeat the execution of a judgment for execution is the fruit and end of the suit and is very aptly called the life of the law (Ipekdjian Merchandising Co. v. Court of Tax Appeals, 8 SCRA 59 [1963]; Commissioner of Internal Revenue v. Visayan Electric Co., 19 SCRA 697, 698 [1967]). A judgment cannot be rendered nugatory by the unreasonable application of a strict rule of procedure. Vested rights were never intended to rest on the requirement of a return, the

office of which is merely to inform the court and the parties, of any and all actions taken under the writ of execution. Where such information can be established in some other manner, the absence of an executing officer's return will not preclude a judgment from being treated as discharged or being executed through an alias writ of execution as the case may be. More so, as in the case at bar. Where the return cannot be expected to be forthcoming, to require the same would be to compel the enforcement of rights under a judgment to rest on an impossibility, thereby allowing the total avoidance of judgment debts. So long as a judgment is not satisfied, a plaintiff is entitled to other writs of execution (Government of the Philippines v. Echaus and Gonzales, 71 Phil. 318). It is a well known legal maxim that he who cannot prosecute his judgment with effect, sues his case vainly.”

G. Execution cannot be equated with satisfaction of a judgment; execution and satisfaction of judgment, distinguished PAL, Inc. vs. CA, said that execution is the process which carries into effect a decree or judgment (Painter v. Berglund, 31 Cal. App. 2d. 63, 87 P 2d 360, 363; Miller v. London, 294 Mass 300, 1 NE 2d 198, 200; Black's Law Dictionary), whereas the satisfaction of a judgment is the payment of the amount of the writ, or a lawful tender thereof, or the conversion by sale of the debtor's property into an amount equal to that due, and, it may done otherwise, than upon an execution (Section 47, Rule 39). Levy and delivery by an execution officer are not prerequisites to the satisfaction of a judgment when the same has already been realized in fact (Section 47, Rule 39). Execution is for the sheriff to accomplish while satisfaction of the judgment is for the creditor to achieve. Section 15, Rule 39 merely provides the sheriff with his duties as executing officer including delivery of the proceeds of his levy on the debtor's property to satisfy the judgment debt. It is but to stress that the implementing officer's duty should not stop at his receipt of payments but must continue until payment is delivered to the obligor or creditor (G.R. No. L-49188, January 30, 1990). H. Motion to quash execution when proper

A motion to quash execution is only proper where: (a) the writ of execution varies the judgment; (b) there has been a change in the situation of the parties making execution inequitable or unjust; (c) execution is sought to be enforced against property exempt from execution; (d) it appears that the controversy has never been submitted to the judgment of the court; (e) the terms of the judgment are not clear enough and there remains room for interpretation thereof; or (f) it appears that the writ of execution has been improvidently issued, or that it is defective in substance or is issued against the wrong party, or that the judgment debt has been paid or otherwise satisfied, or the writ was issued without authority (Gutierrez vs, Valiente,).

I. Execution pending appeal On motion of the prevailing party will notice to the adverse party filed in the trial court while it has jurisdiction over the case and is in possession of either the original record or the record on appeal, as the case may be, at the time of the filing of such motion, said court may, in its discretion, order execution of a judgment or final order even before the expiration of the period to appeal. After the trial court has lost jurisdiction, the motion for execution pending appeal may be filed in the appellate court.

Discretionary execution may only issue upon good reasons to be stated in a special order after due hearing (Section 2, Rule 39, 1997 Revised Rules of Civil Procedure). Discretionary execution of appealed judgments may be allowed upon concurrence of the following requisites: (a) there must be a motion by the prevailing party with notice to the adverse party; (b) there must be a good reason for execution pending appeal; and (c) the good reason must be stated in a special order (Manacop vs. Equitable Banking Corporation,). The exercise of the power to grant or deny immediate execution is addressed to the sound discretion of the court and the existence of good reasons is precisely what confers such discretionary power upon the court (Philippine National Bank vs. Puno,). J. Good Reasons Good reasons consist of compelling circumstances justifying immediate execution lest judgment becomes illusory, or the prevailing party after the lapse of time be unable to enjoy it, considering the tactics of the adverse party who may have apparently no cause but to delay. Such reasons must constitute superior circumstances demanding urgency which will outweigh the injury or damages should the losing party secure a reversal of the judgment. Execution of a judgment pending appeal is an exception to the general rule that only a final judgment may be executed. Thus, the existence of “good reasons” is essential for it is what confers discretionary power on a court to issue a writ of execution pending appeal (Villamor vs. National Power Corporation,).

J. Supersedeas Bond

The purpose of the supersedeas bond is to answer for the rents, damages and costs accruing down to the judgment of the inferior court appealed from, the amount of which is to be determined from the judgment of said court. The postulation of complainants and their counsel that the execution sought was effectively stayed by the filing of a supersedeas bond was sufficiently refuted and justifiably rejected when we consider the circumstances then obtaining (Cordova vs. Judge Labayen, A). ---0---

REVIVAL OF JUDGMENT A. The principle and Rule An action for revival of judgment is no more than a procedural means of securing the execution of a previous judgment which has become dormant after the passage of five years without it being executed upon motion of the prevailing party. It is not intended to re-open any issue affecting the merits of the judgment debtor’s case nor the propriety or correctness of the first judgment (Panotes, vs. City Townhouse Development Corporation,). It is premised on the assumption that the decision to be revived, either by motion or by independent action, is already final and executory (Saligumba vs. Palanog,). A final and executory judgment or order may be executed on motion within five (5) years from the date of its entry. After the lapse of such time, and before it is barred by the statute of limitations, a judgment may be enforced by action. The revived judgment may also be enforced by motion within five (5) years from the date of its entry and thereafter by action before it is barred by the statute of limitations (Section 6, Rule 39, 1997 Revised Rules of Civil Procedure [Execution by motion or by independent action]).

The purpose of the law in prescribing time limitations for enforcing judgment by action is precisely to prevent the winning parties from sleeping on their rights (Macias vs. Lim,).

B. It is a new and independent action An action for revival of judgment is a new and independent action, different and distinct from either the recovery of property case or the reconstitution case, wherein the cause of action is the decision itself and not the merits of the action upon which the judgment sought to be enforced is rendered (Juco vs. Heirs of Tomas Siy Chung Fu,). C. Period within which to file action for revival of judgment An action may be revived by mere motion within the period between five (5) years from the date of entry of judgment, and by independent action within the period or between five (5) years and one (1) day from the date of entry of judgment to ten (10) years. The guide in filing a revival of action is that: first, observe the 10 year prescription period counted from the date said judgment become final or from the date of its entry, and second, observe the five (5) year period mentioned in Section 6, Rule 39 of the Rules of Court – final and executory judgment or order may be executed on motion within five (5) years from the date of its entry. Again, this may be by mere motion or by action. If the action for revival of judgment is filed after the five (5) year period provided for in the Rules of Court, but beyond the ten (10) year period provided for in the Civil Code, the action is considered barred by the statute of limitations, thus, an action can no longer be enforced by action. In Quesada vs. CA, , a writ of execution was issued on September 15, 1976, but was not enforced. An action for revival of judgment was filed on August 26, 1985. The CA ruled that the action was already barred by prescription. The High Court ruled that the action to revived judgment was not barred by prescription because it was filed within the 10 year period.

The period within which to file action for revival of judgment was explained by the High Court in Shipside Incorporated vs. CA, , thus: “The action instituted by the Solicitor General in the trial court is one for revival of judgment which is governed by Article 1144(3) of the Civil Code and Section 6, Rule 39 of the 1997 Rules on Civil Procedure. Article 1144(3) provides that an action upon a judgment “must be brought within 10 years from the time the right of action accrues." On the other hand, Section 6, Rule 39 provides that a final and executor judgment or order may be executed on motion within five (5) years from the date of its entry, but that after the lapse of such time, and before it is barred by the statute of limitations, a judgment may be enforced by action. Taking these two provisions into consideration, it is plain that an action for revival of judgment must be brought within ten years from the time said judgment becomes final.” In this case (Shipside Incorporated), the High Court said that: “From the records of this case, it is clear that the judgment sought to be revived became final on October 23, 1973. On the other hand, the action for revival of judgment was instituted only in 1999, or more than twenty-five (25) years after the judgment had become final. Hence, the action is barred by extinctive prescription considering that such an action can be instituted only within ten (10) years from the time the cause of action accrues.” 0-0-0 In Villeza vs. German Management and Services, Inc, , made to emphasize that an action for revival of judgment is governed by Article 1144 (3), Article 1152 of the Civil Code and Section 6, Rule 39 of the Rules of Court; and that judgment must be enforced by the institution of a

complaint in a regular court within ten years from the time the judgment becomes final. The High Court said: “Art. 1144. The following actions must be brought within ten years from the time the right of action accrues: xxxx (3) Upon a judgment Article 1152 of the Civil Code states: Art. 1152. The period for prescription of actions to demand the fulfillment of obligations declared by a judgment commences from the time the judgment became final. Apropos, Section 6, Rule 39 of the Rules of Court reads: Sec. 6. Execution by motion or by independent action. –A final and executory judgment or order may be executed on motion within five (5) years from the date of its entry. After the lapse of such time, and before it is barred by the statute of limitations, a judgment may be enforced by action. The revived judgment may also be enforced by motion within five (5) years from the date of its entry and thereafter by action before it is barred by the statute of limitations. (emphasis supplied) The rules are clear. Once a judgment becomes final and executory, the prevailing party can have it executed as a matter of right by mere motion within five years from the date of entry of judgment. If the prevailing party fails to have the decision enforced by a motion after the lapse of

five years, the said judgment is reduced to a right of action which must be enforced by the institution of a complaint in a regular court within ten years from the time the judgment becomes final.” “When petitioner Villeza filed the complaint for revival of judgment on October 3, 2000, it had already been eleven (11) years from the finality of the judgment he sought to revive. Clearly, the statute of limitations had set in” said the High Court.15 The High Court added: “xxx xxx xxx it was petitioner Villeza, the prevailing party himself, who moved to defer the execution of judgment. The losing party never had any hand in the delay of its execution. Neither did the parties have any agreement on that matter. After the lapse of five years (5) from the finality of judgment, petitioner Villeza should have instead filed a complaint for its revival in accordance with Section 6, Rule 39 of the Rules of Court. He, however, filed a motion to execute the same which was a wrong course of action. On the 11th year, he finally sought its revival but he requested the aid of the courts too late.”16 -0-0-0D. Instances that prescriptive period within which to revive action is interrupted or tolled (1). If it is impossible for the winning party to have sought the execution of the judgment because of the dilatory schemes and maneuvers resorted to by the other party (Republic vs. CA,). (2). There is an agreement between parties to defer or suspend the enforcement of the judgment, or when the order of ejectment was not carried out, however, due to the judgment debtor’s begging to withhold the execution of judgment because of financial difficulties (Torralba vs. delos Angeles,).

15

Villeza vs. German Management and Services, Inc, G.R. No. 182937, August 8, 2010. Villeza vs. German Management and Services, Inc, G.R. No. 182937, August 8, 2010.

16

(3). When the oblior moves for the suspension of the writ of execution, or when the obligee was not in delay because he exhausted all legal means within his power to eject the obligor from his land; or where the writs of execution issued by the lower court were not complied with and/or were suspended by reason of acts or causes not of obligee’s own making and against his objections (Casela vs. CA).

E. Venue of revival of judgment involving personal action (accion in personam) and real action In revival of judgment, if the revival of judgment action is a personal one (accion in personam), the venue of the action is in the residence of the plaintiff, or at the latter’s option, residence of the defendant, and if the revival of judgment action involves a real action, the venue of the action is in the place where the property subject of the revival action is situated (Infante vs, Aran Builders,). The High Court in Infante: “xxx xxx xxx. Petitioner claims that the CA erred in finding that the complaint for revival of judgment is an action in rem which was correctly filed with the RTC of the place where the disputed real property is located. The petition is unmeritorious. Petitioner insists that the action for revival of judgment is an action in personam; therefore, the complaint should be filed with the RTC of the place where either petitioner or private respondent resides. Petitioner then concludes that the filing of the action for revival of judgment with the RTC of Muntinlupa City, the place where the disputed property is located, should be dismissed on the ground of improper venue. Private respondent is of the opinion that the judgment it is seeking to revive involves interest over real property.

As such, the present action for revival is a real action, and venue was properly laid with the court of the place where the realty is located. Thus, the question that must be answered is: where is the proper venue of the present action for revival of judgment? Section 6, Rule 39 of the 1997 Rules of Civil Procedure provides that after the lapse of five (5) years from entry of judgment and before it is barred by the statute of limitations, a final and executor judgment or order may be enforced by action. The Rule does not specify in which court the action for revival of judgment should be filed. In Aldeguer v. Gemelo, the Court held that: x x x an action upon a judgment must be brought either in the same court where said judgment was rendered or in the place where the plaintiff or defendant resides, or in any other place designated by the statutes which treat of the venue of actions in general. (Emphasis supplied) but emphasized that other provisions in the rules of procedure which fix the venue of actions in general must be considered. Under the present Rules of Court, Sections 1 and 2 of Rule 4 provide: Section 1. Venue of real actions. Actions affecting title to or possession of real property, or interest therein, shall be commenced and tried in the proper court which has jurisdiction over the area wherein the real property involved, or a portion thereof, is situated.

xxxx Section 2. Venue of personal actions. All other actions may be commenced and tried where the plaintiff or any of the principal plaintiffs resides, or where the defendant or any of the principal defendants resides, or in the case of a non-resident defendant where he may be found, at the election of the plaintiff. Thus, the proper venue depends on the determination of whether the present action for revival of judgment is a real action or a personal action. Applying the afore-quoted rules on venue, if the action for revival of judgment affects title to or possession of real property, or interest therein, then it is a real action that must be filed with the court of the place where the real property is located. If such action does not fall under the category of real actions, it is then a personal action that may be filed with the court of the place where the plaintiff or defendant resides. In support of her contention that the action for revival of judgment is a personal action and should be filed in the court of the place where either the plaintiff or defendant resides, petitioner cites the statements made by the Court in Aldeguer v. Gemelo and Donnelly v. Court of First Instance of Manila. Petitioner, however, seriously misunderstood the Court's rulings in said cases. In Aldeguer, what the Court stated was that “[t]he action for the execution of a judgment for damages is a personal one, and under section 377 [of the Code of Civil Procedure], it should be brought in any province where the plaintiff or the defendant resides, at the election of the plaintiff” (Emphasis and underscoring supplied). Petitioner apparently took such statement to mean that any action for revival of judgment should be considered as a personal one. This thinking is incorrect. The Court specified that the judgment sought to be revived in said case was a judgment

for damages. The judgment subject of the action for revival did not involve or affect any title to or possession of real property or any interest therein. The complaint filed in the rivival case did not fall under the category of real actions and, thus, the action necessarily fell under the category of personal actions. In Donnelly, the portion of the Decision being relied upon by petitioner stated thus: Petitioner raises before this Court two (2) issues, namely: (a) whether an action for revival of judgment is one quasi in rem and, therefore, service of summons may be effected thru publication; and (b) whether the second action for revival of judgment (Civil Case No. 76166) has already prescribed. To our mind, the first is not a proper and justiciable issue in the present proceedings x x x. Nevertheless, let it be said that an action to revive a judgment is a personal one. (Emphasis supplied) The Court clearly pointed out that in said case, the issue on whether an action for revival of judgment is quasi in rem was not yet proper and justiciable. Therefore, the foregoing statement cannot be used as a precedent, as it was merely an obiter dictum. Moreover, as in Aldeguer, the judgment sought to be revived in Donnelly involved judgment for a certain sum of money. Again, no title or interest in real property was involved. It is then understandable that the action for revival in said case was categorized as a personal one. Clearly, the Court's classification in Aldeguer and Donnelly of the actions for revival of judgment as being personal in character does not apply to the present case.

The allegations in the complaint for revival of judgment determine whether it is a real action or a personal action. The complaint for revival of judgment alleges that a final and executor judgment has ordered herein petitioner to execute a deed of sale over a parcel of land in Ayala Alabang Subdivision in favor of herein private respondent; pay all pertinent taxes in connection with said sale; register the deed of sale with the Registry of Deeds and deliver to Ayala Corporation the certificate of title issued in the name of private respondent. The same judgment ordered private respondent to pay petitioner the sum of P321,918.25 upon petitioner's compliance with the aforementioned order. It is further alleged that petitioner refused to comply with her judgment obligations despite private respondent's repeated requests and demands, and that the latter was compelled to file the action for revival of judgment. Private respondent then prayed that the judgment be revived and a writ of execution be issued to enforce said judgment. The previous judgment has conclusively declared private respondent's right to have the title over the disputed property conveyed to it. It is, therefore, undeniable that private respondent has an established interest over the lot in question; and to protect such right or interest, private respondent brought suit to revive the previous judgment. The sole reason for the present action to revive is the enforcement of private respondent's adjudged rights over a piece of realty. Verily, the action falls under the category of a real action, for it affects private respondent's interest over real property. The present case for revival of judgment being a real action, the complaint should indeed be filed with the Regional Trial Court of the place where the realty is located. Section 18 of Batas Pambansa Bilang 129 provides:

Sec. 18. Authority to define territory appurtenant to each branch. - The Supreme Court shall define the territory over which a branch of the Regional Trial Court shall exercise its authority. The territory thus defined shall be deemed to be the territorial area of the branch concerned for purposes of determining the venue of all suits, proceedings or actions, whether civil or criminal, as well as determining the Metropolitan Trial Courts, Municipal Trial Courts and Municipal Circuit Trial Courts over which the said branch may exercise appellate jurisdiction. The power herein granted shall be exercised with a view to making the courts readily accessible to the people of the different parts of the region and making the attendance of litigants and witnesses as inexpensive as possible. (Emphasis supplied) From the foregoing, it is quite clear that a branch of the Regional Trial Court shall exercise its authority only over a particular territory defined by the Supreme Court. Originally, Muntinlupa City was under the territorial jurisdiction of the Makati Courts. However, Section 4 of Republic Act No. 7154, entitled An Act to Amend Section Fourteen of Batas Pambansa Bilang 129, Otherwise Known As The Judiciary Reorganization Act of 1981, took effect on September 4, 1991. Said law provided for the creation of a branch of the Regional Trial Court in Muntinlupa. Thus, it is now the Regional Trial Court in Muntinlupa City which has territorial jurisdiction or authority to validly issue orders and processes concerning real property within Muntinlupa City. Thus, there was no grave abuse of discretion committed by the Regional Trial Court of Muntinlupa City, Branch 276 when it denied petitioner's motion to dismiss; and the CA did not commit any error in affirming the same.”

F. Judgment does not attain finality when record is destroyed during pendency of motion for reconsideration

When the records of the original case were destroyed in the fire during the pendency of the motion for reconsideration of the disapproval of the record on appeal, a motion for reconsideration has the effect of suspending the statutory period after which an order, decision, or judgment, in connection with which said motion was filed, becomes final. The motion for reconsideration prevents the decision from attaining finality. Cannot therefore be a proper subject of an action for revival of judgment (Juco vs. Heirs of Tomas Siy Chung Fu,). Juco said: “As pointed out by the appellate court, an action for revival of judgment is a new and independent action, different and distinct from either the recovery of property case or the reconstitution case, wherein the cause of action is the decision itself and not the merits of the action upon which the judgment sought to be enforced is rendered. However, revival of judgment is premised on the assumption that the decision to be revived, either by motion or by independent action, is already final and executory. Hence, the need to make a determination of whether or not the decision in Civil Case No. 7281 has indeed become final and executory. For if the subject decision has already reached finality, then the conclusion of the appellate court is correct that the dismissal of the reconstitution case would not prevent respondents from reviving and thereafter executing the said decision. A decision issued by a court is final and executory when such decision disposes of the subject matter in its entirety or terminates a particular proceeding or action, leaving nothing else to be done but to enforce by execution what has been determined by the court, such as when after the lapse of the reglementary period to appeal, no appeal

has been perfected. In the case at bar, it is an undisputed fact that when the records of the original case were destroyed in the fire there was a pending motion for reconsideration of the disapproval of the record on appeal filed by petitioner. A motion for reconsideration has the effect of suspending the statutory period after which an order, decision, or judgment, in connection with which said motion was filed, becomes final. In effect, such motion for reconsideration has prevented the decision from attaining finality. The findings of the Court of Appeals that notwithstanding the pendency of the motion for reconsideration, the decision in Civil Case No. 7281 has become final and executory by reason of laches cannot be sustained. As discussed above, the doctrine of laches cannot operate to lend finality to the decision since petitioner’s failure to pursue the motion for reconsideration was not due to her negligence or abandonment, but was rather brought upon by the dismissal of the reconstitution case. Therefore, it is clear that the case has not reached finality at the time the records of the case were burnt.”

---0---

APPEAL A. Right to appeal is a statutory privilege The right to appeal is a statutory privilege and of statutory origin, not a constitutional, natural or inherent right. Therefore, available only if granted or as provided by statutes. It may be exercised only in the manner prescribed by the provisions of the law. The period to appeal is specifically governed by Section 39 of Batas Pambansa Blg. 129 (BP 129), as amended amd Section 3 of Rule 41 of the 1997 Rules of Civil Procedure (Yu vs. Judge Samson-Tatad,).

B. When to appeal? The period for appeal from final orders, resolutions, awards, judgments, or decisions of any court in all cases shall be fifteen (15) days counted from the notice of the final order, resolution, award, judgment, or decision appealed from (Section 39, BP 129, as amended). Under Section 39 of Batas Pambansa Bilang (B.P. Blg.) 129, the petitioners had fifteen (15) days within which to file their notice of appeal, from the time their counsel received notice or was served a copy of the trial court’s decision. The fifteen-day period provided therein is mandatory and jurisdictional. It bears stressing that the right to appeal is not a natural right or a part of due process. It is a procedural remedy of statutory origin and, as such, may be exercised only in the manner and within the time frame provided by the provisions of law authorizing its exercise. Failure of a party to perfect an appeal within the period fixed by law renders the decision sought to be appealed final and executory. As a result, no court could exercise appellate jurisdiction to review the decision. After a decision is declared final and executory, vested rights are acquired by the winning party who has the right to enjoy the finality of the case (Alon vs. CA, ). Section 39, BP 129, as amended, is the substantive source of the remedy of appeal, while Section 3, Rule 41 of the 1997 Rules of Civil Procedure is the procedural source.

In Ortigas and Company Limited Partnership v. Velasco, this Court expounded on the interpretation of certain phrases used by the courts in their judgments or resolutions – C. Denial of motion for reconsideration with words “with finality” is of no consequence While the denial of a motion for reconsideration of a judgment or final order is normally accompanied by the modifier, “final,” or “with finality,” there may be a denial not so qualified. That is of no

consequence. By no means may it be taken as indicating any uncertainty or indecisiveness on the part of the Court regarding its denial of reconsideration, or an encouragement or expectation of a second motion for reconsideration. The modifier serves simply to emphasize the import and effect of the denial of the motion for reconsideration, i.e., that the Court will entertain and consider no further arguments or submissions from the parties respecting its correctness; that in the Court’s considered view, nothing more is left to be discussed, clarified or done in the case, all issues raised having been passed upon and definitely resolved, and any other which could have been raised having been waived and no longer being available as ground for a second motion. A denial with finality stresses that the case is considered closed (Ortigas and Company Limited Partnership v. Velasco,).

APPEALS FROM FIRST TO SECOND LEVEL COURT A. How, where and when to appeal? The appeal is taken by filing a notice of appeal with the court that rendered the judgment or final order appealed from. The notice of appeal shall indicate the parties to the appeal, the judgment or final order or part thereof appealed from, and state the amterial dates showing the timeliness of the appeal (Section 3, Rule 40, 1997 Revised Rules of Civil Procedure). An appeal from a judgment or final order of a Municipal Trial Court may be taken to the Regional Trial Court exercising jurisdiction over the area to which the former pertains. The title of the case shall remain as it was in the court of origin, but the party appealing the case shall be further referred to as the appellant and the adverse party as the appellee (Section 1, Rule 40, 1997 Revised Rules of Civil Procedure). An appeal may be taken within fifteen (15) days after notice to the appellant of the judgment or final order appealed from. Where a record on appeal is required, the appellant shall file a notice of appeal and a record on appeal within thirty (30) days after notice of the judgment or final order (Section 2, Rule 40, 1997 Revised Rules of Civil Procedure).

The period of appeal shall be interrupted by a timely motion for new trial or reconsideration. No motion for extension of time to file a motion for new trial or reconsideration shall be allowed (Paragraph 2, Section 2, Rule 40, 1997 Revised Rules of Civil Procedure).

B. Perfection of appeals and its effects According to Section 4 of Rule 40, 1997 Revised Rules of Civil Procedure, the perfection of the appeal and the effect thereof shall be governed by the provisions of Section 9, Rule 41. Section 4 of Rule 40, 1997 Revised Rules of Civil Procedure says: Section 9. Perfection of appeal; effect thereof. — A party's appeal by notice of appeal is deemed perfected as to him upon the filing of the notice of appeal in due time. A party's appeal by record on appeal is deemed perfected as to him with respect to the subject matter thereof upon the approval of the record on appeal filed in due time. In appeals by notice of appeal, the court loses jurisdiction over the case upon the perfection of the appeals filed in due time and the expiration of the time to appeal of the other parties. In appeals by record on appeal, the court loses jurisdiction only over the subject matter thereof upon the approval of the records on appeal filed in due time and the expiration of the appeal of the other parties. In either case, prior to the transmittal of the original record or the record on appeal, the court may issue orders for the protection and preservation of the rights of the parties which do not involve any matter litigated by the appeal, approve compromises, permit appeals of indigent

litigants, order execution pending appeal in accordance with 2 of Rule 39, and allow withdrawal of the appeal.

C. Payment of appellate and another lawful fees is a requirement The general rule is that appeal is perfected by filing a notice of appeal and paying the requisite docket fees and other lawful fees (Baniqued v. Ramos, cited in Tanenglian vs.. Lorenzo, G.R. No. 173415, March 28, 2008). Within the period for taking an appeal, the appellant shall pay to the clerk of the court which rendered the judgment or final order appealed from the full amount of the appellate court docket and other lawful fees. Proof of payment thereof shall be transmitted to the appellate court together with the original record or the record on appeal, as the case may be (Section 5, Rule 40, 1997 Revised Rules of Civil Procedure).

D. Duty of clerk of court of the first level court upon perfection of appeal Within fifteen (15) days from the perfection of the appeal, the clerk of court or the branch clerk of court of the lower court shall transmit the original record or the record on appeal, together with the transcripts and exhibits, which he shall certify as complete, to the proper Regional Trial Court. A copy of his letter of transmittal of the records to the appellate court shall be furnished the parties (Section 5, Rule 40, 1997 Revised Rules of Civil Procedure).

E. Duty of clerk of court of the second level court (appellate court) upon receipt of record Upon receipt of the complete record or the record on appeal, the clerk of court of the Regional Trial Court shall notify the parties of such fact (Section 7 (a), Rule 40, 1997 Revised Rules of Civil Procedure).

Within fifteen (15) days from such notice, it shall be the duty of the appellant to submit a memorandum which shall briefly discuss the errors imputed to the lower court, a copy of which shall be furnished by him to the adverse party. Within fifteen (15) days from receipt of the appellant's memorandum, the appellee may file his memorandum. Failure of the appellant to file a memorandum shall be a ground for dismissal of the appeal (Section 7 (b), Rule 40, 1997 Revised Rules of Civil Procedure). Upon the filing of the memorandum of the appellee, or the expiration of the period to do so, the case shall be considered submitted for decision. The Regional Trial Court shall decide the case on the basis of the entire record of the proceedings had in the court of original and such memoranda as are filed (Section 7 (c), Rule 40, 1997 Revised Rules of Civil Procedure). F. Appeal of dismissed case without trial or for lack of jurisdiction If an appeal is taken from an order of the lower court dismissing the case without a trial on the merits, the Regional Trial Court may affirm or reverse it, as the case may be. In case of affirmance and the ground of dismissal is lack of jurisdiction over the subject matter, the Regional Trial Court, if it has jurisdiction thereover, shall try the case on the merits as if the case was originally filed with it. In case of reversal, the case shall be remanded for further proceedings (Section 8, Rule 40, 1997 Revised Rules of Civil Procedure). If the case was tried on the merits by the lower court without jurisdiction over the subject matter, the Regional Trial Court on appeal shall not dismiss the case if it has original jurisdiction thereof, but shall decide the case in accordance with the preceding section, without prejudice to the admission of amended pleadings and additional evidence in the interest of justice (Paragraph 2, Section 8, Rule 40, 1997 Revised Rules of Civil Procedure).

APPEALS FROM SECOND LEVEL COURT TO COURT OF APPEALS

A. How and when to appeal? The appeal is taken by filing a notice of appeal with the court that rendered the judgment or final order appealed from. The notice of appeal shall indicate the parties to the appeal, the judgment or final order or part thereof appealed from, and state the amterial dates showing the timeliness of the appeal (Section 3, Rule 40, 1997 Revised Rules of Civil Procedure). The appeal shall be taken within fifteen (15) days from notice of the judgment or final order appealed from. Where a record on appeal is required, the appellant shall file a notice of appeal and a record on appeal within thirty (30) days from notice of the judgment or final order (Section 3, Rule 41, 1997 Revised Rules of Civil Procedure). The period of appeal shall be interrupted by a timely motion for new trial or reconsideration. No motion for extension of time to file a motion for new trial or reconsideration shall be allowed (Paragraph 2, Section 3, Rule 41, 1997 Revised Rules of Civil Procedure). The notice of appeal shall indicate the parties to the appeal, specify the judgment or final order or part thereof appealed from, specify the court to which the appeal is being taken, and state the material dates showing the timeliness of the appeal (Section 5, Rule 41, 1997 Revised Rules of Civil Procedure).

B. Subject of appeal An appeal may be taken from a judgment or final order that completely disposes of the case, or of a particular matter therein when declared by these Rules to be appealable (Section 1, Rule 41, 1997 Revised Rules of Civil Procedure).

C. Appeal is not permitted No appeal may be taken from:

(a) An order denying a motion for new trial or reconsideration; (b) An order denying a petition for relief or any similar motion seeking relief from judgment; (c) An interlocutory order; (d) An order disallowing or dismissing an appeal; (e) An order denying a motion to set aside a judgment by consent, confession or compromise on the ground of fraud, mistake or duress, or any other ground vitiating consent; (f) An order of execution; (g) A judgment or final order for or against one or more of several parties or in separate claims, counterclaims, crossclaims and third-party complaints, while the main case is pending, unless the court allows an appeal therefrom; and (h) An order dismissing an action without prejudice. In all the above instances where the judgment or final order is not appealable, the aggrieved party may file an appropriate special civil action under Rule 65 (Paragraph 2, Section 1, Rule 41, 1997 Revised Rules of Civil Procedure).

D. The modes of appeal The appeal to the Court of Appeals in cases decided by the Regional Trial Court in the exercise of its original jurisdiction shall be taken by filing a notice of appeal with the court which rendered the judgment or final order appealed from and serving a copy thereof upon the adverse party. No record on appeal shall be required except in special proceedings and other cases of multiple or separate appeals where law on these Rules so require. In such cases, the record on appeal shall be filed and served in like manner (Section 2 (a), Rule 41, 1997 Revised Rules of Civil Procedure).

This is the so-called “ordinary appeal”. The appeal to the Court of Appeals in cases decided by the Regional Trial Court in the exercise of its appellate jurisdiction shall be by petition for review in accordance with Rule 42 (Section 2 (b), Rule 41, 1997 Revised Rules of Civil Procedure). This is the so-called “petition for review”. In all cases where only questions of law are raised or involved, the appeal shall be to the Supreme Court by petition for review on certiorari in accordance with the Rule 45 (Section 2 (c), Rule 41, 1997 Revised Rules of Civil Procedure). This is the so-called “appeal by certiorari”.

E. Perfection of appeal and its effect A party's appeal by notice of appeal is deemed perfected as to him upon the filing of the notice of appeal in due time (Section 9, Rule 41, 1997 Revised Rules of Civil Procedure). A party's appeal by record on appeal is deemed perfected as to him with respect to the subject matter thereof upon the approval of the record on appeal filed in due time (Paragraph 2, Section 9, Rule 41, 1997 Revised Rules of Civil Procedure). In appeals by notice of appeal, the court loses jurisdiction over the case upon the perfection of the appeals filed in due time and the expiration of the time to appeal of the other parties (Paragraph 3, Section 9, Rule 41, 1997 Revised Rules of Civil Procedure). In appeals by record on appeal, the court loses jurisdiction only over the subject matter thereof upon the approval of the records on appeal filed in due time and the expiration of the appeal of the other parties (Paragraph 4, Section 9, Rule 41, 1997 Revised Rules of Civil Procedure).

In either case, prior to the transmittal of the original record or the record on appeal, the court may issue orders for the protection and preservation of the rights of the parties which do not involve any matter litigated by the appeal, approve compromises, permit appeals of indigent litigants, order execution pending appeal in accordance with 2 of Rule 39, and allow withdrawal of the appeal (Paragraph 5, Section 9, Rule 41, 1997 Revised Rules of Civil Procedure).

F. Duty of clerk of court of the lower court upon perfection of appeal Within thirty (30) days after perfection of all the appeals in accordance with the preceding section, it shall be the duty of the clerk of court of the lower court: (a) To verify the correctness of the original record or the record on appeal, as the case may be aid to make certification of its correctness; (b) To verify the completeness of the records that will be, transmitted to the appellate court; (c) If found to be incomplete, to take such measures as may be required to complete the records, availing of the authority that he or the court may exercise for this purpose; and (d) To transmit the records to the appellate court. If the efforts to complete the records fail, he shall indicate in his letter of transmittal the exhibits or transcripts not included in the records being transmitted to the appellate court, the reasons for their nontransmittal, and the steps taken or that could be taken to have them available. The clerk of court shall furnish the parties with copies of his letter of transmittal of the records to the appellate court (Section 10, Rule 41, 1997 Revised Rules of Civil Procedure).

Upon the perfection of the appeal, the clerk shall immediately direct the stenographers concerned to attach to the record of the case five (5) copies of the transcripts of the testimonial evidence referred to in the record on appeal. The stenographers concerned shall transcribe such testimonial evidence and shall prepare and affix to their transcripts an index containing the names of the witnesses and the pages wherein their testimonies are found, and a list of the exhibits and the pages wherein each of them appears to have been offered and admitted or rejected by the trial court. The transcripts shall be transmitted to the clerk of the trial court who shall thereupon arrange the same in the order in which the witnesses testified at the trial, and shall cause the pages to be numbered consecutively (Section 10, Rule 41, 1997 Revised Rules of Civil Procedure). The clerk of the trial court shall transmit to the appellate court the original record or the approved record on appeal within thirty (30) days from the perfection of the appeal, together with the proof of payment of the appellate court docket and other lawful fees, a certified true copy of the minutes of the proceedings, the order of approval, the certificate of correctness, the original documentary evidence referred to therein, and the original and three (3) copies of the transcripts. Copies of the transcripts and certified true copies of the documentary evidence shall remain in the lower court for the examination of the parties (Section 12, Rule 41, 1997 Revised Rules of Civil Procedure).

G. Payment of appellate and another lawful fees is a requirement Within the period for taking an appeal, the appellant shall pay to the clerk of the court which rendered the judgment or final order appealed from the full amount of the appellate court docket and other lawful fees. Proof of payment thereof shall be transmitted to the appellate court together with the original record or the record on appeal, as the case may be (Section 5, Rule 40, 1997 Revised Rules of Civil Procedure).

H. Duty of clerk of court of the first level court upon perfection of appeal

I. Record on appeal; form and contents Again, the Notice of Appeal shall indicate the parties to the appeal, specify the judgment or final order or part thereof appealed from, specify the court to which the appeal is being taken, and state the material dates showing the timeliness of the appeal (Section 5, Rule 41, 1997 Revised Rules of Civil Procedure). The record on appeal shall contain the full names of all the parties to the proceedings shall be stated in the caption of the record on appeal and it shall include the judgment or final order from which the appeal is taken and, in chronological order, copies of only such pleadings, petitions, motions and all interlocutory orders as are related to the appealed judgment or final order for the proper understanding of the issue involved, together with such data as will show that the appeal was perfected on time. If an issue of fact is to be raised on appeal, the record on appeal shall include by reference all the evidence, testimonial and documentary, taken upon the issue involved. The reference shall specify the documentary evidence by the exhibit numbers or letters by which it was identified when admitted or offered at the hearing, and the testimonial evidence by the names of the corresponding witnesses. If the whole testimonial and documentary evidence in the case is to be included, a statement to that effect will be sufficient without mentioning the names of the witnesses or the numbers or letters of exhibits. Every record on appeal exceeding twenty (20) pages must contain a subject index (Section 6, Rule 41, 1997 Revised Rules of Civil Procedure). Copies of the notice of appeal and the record on appeal where required, shall be served on the adverse party (Paragraph 4, Section 3, Rule 40, 1997 Revised Rules of Civil Procedure). J. Approval of record on appeal; and when both parties are appellants Upon the filing of the record on appeal for approval and if no objection is filed by the appellee within five (5) days from receipt of a copy thereof, the trial court may approve it as presented or upon its own motion or at the instance of the appellee, may direct its amendment by the

inclusion of any omitted matters which are deemed essential to the determination of the issue of law or fact involved in the appeal. If the trial court orders the amendment of the record, the appellant, within the time limited in the order, or such extension thereof as may be granted, or if no time is fixed by the order within ten (10) days from receipt thereof, shall redraft the record by including therein, in their proper chronological sequence, such additional matters as the court may have directed him to incorporate, and shall thereupon submit the redrafted record for approval, upon notice to the appellee, in like manner as the original draft (Section 7, Rule 41, 1997 Revised Rules of Civil Procedure). Where both parties are appellants, they may file a joint record on appeal within the time fixed by section 3 of this Rule, or that fixed by the court (Section 8, Rule 41, 1997 Revised Rules of Civil Procedure).

K. Payment of appellate and another lawful fees is a requirement Within the period for taking an appeal, the appellant shall pay to the clerk of the court which rendered the judgment or final order appealed from, the full amount of the appellate court docket and other lawful fees. Proof of payment of said fees shall be transmitted to the appellate court together with the original record or the record on appeal (Section 4, Rule 41, 1997 Revised Rules of Civil Procedure). The appellate docket and other lawful fees must be paid within the same period for taking an appeal (Enriquez vs. Enriquez, G.R. No. 139303, August 25, 2005). The High Court in Enriquez vs. Enriquez, G.R. No. 139303, August 25, 2005, said: “Prior to the effectivity of the 1997 Rules of Civil Procedure, as amended, payment of appellate court docket fee is not a prerequisite for the perfection of an appeal. In Santos vs. Court of Appeals, this Court held that although an appeal fee is required to be paid in case of an appeal taken from the Municipal Trial Court to the Regional Trial Court, it is not a prerequisite for the perfection of an appeal

under Sections 20 and 23 of the Interim Rules and Guidelines issued by this Court on January 11, 1983 implementing the Judiciary Reorganization Act of 1981 (B.P. Blg. 129). Under these sections, there are only two requirements for the perfection of an appeal, to wit: (a) the filing with the trial court of a notice of appeal within the reglementary period; and (b) the expiration of the last day to appeal by any party. However, the 1997 Rules of Civil Procedure, as amended, which took effect on July 1, 1997, now require that appellate docket and other lawful fees must be paid within the same period for taking an appeal. This is clear from the opening sentence of Section 4, Rule 41 of the same Rules that, “(W)ithin the period for taking an appeal, the appellant shall pay to the clerk of the court which rendered the judgment or final order appealed from, the full amount of the appellate court docket and other lawful fees.” The use of the word “shall” underscores the mandatory character of the Rule. The term “shall” is a word of command, and one which has always or which must be given a compulsory meaning, and it is generally imperative or mandatory. Petitioners cannot give a different interpretation to the Rule and insist that payment of docket fee shall be made only upon their receipt of a notice from the trial court to pay. For it is a rule in statutory construction that every part of the statute must be interpreted with reference to the context, i.e., that every part of the statute must be interpreted together with the other parts, and kept subservient to the general intent of the whole enactment. Indeed, petitioners cannot deviate from the Rule. Also under Rule 41 of the same Rules, an appeal to the Court of Appeals from a case decided by the RTC in the exercise of the latter’s original jurisdiction, shall be taken within fifteen (15) days from the notice of judgment or final order appealed from. Such appeal is made by filing a

notice thereof with the court that rendered the judgment or final order and by serving a copy of that notice upon the adverse party. Furthermore, within this same period, appellant shall pay to the clerk of court which rendered the judgment or final order appealed from, the full amount of the appellate court docket and other lawful fees. The payment of docket fee within this period is mandatory for the perfection of appeal. Otherwise, the appellate court would not be able to act on the subject matter of the action, and the decision sought to be appealed from becomes final and executory. Time and again, this Court has consistently held that payment of docket fee within the prescribed period is mandatory for the perfection of an appeal. Without such payment, the appellate court does not acquire jurisdiction over the subject matter of the action and the decision sought to be appealed from becomes final and executory.”

The appeal shall be taken within fifteen (15) days from notice of the judgment or final order appealed from. Where a record on appeal is required, the appellant shall file a notice of appeal and a record on appeal within thirty (30) days from notice of the judgment or final order (Section 3, Rule 41, 1997 Rules of Civil Procedure). The period of appeal shall be interrupted by a timely motion for new trial or reconsideration. No motion for extension of time to file a motion for new trial or reconsideration shall be allowed (Paragraph 2, Section 3, Rule 41, 1997 Rules of Civil Procedure).

L. Failure to pay the appellate docket fee within the reglementary period bestows on the appellate court a directory power to dismiss an appeal The appellant’s failure to pay the appellate docket fee within the fifteen-day reglementary period bestows on the appellate court a directory, not a mandatory, power to dismiss an appeal (Fontanar vs. Bonsubre).

The High Court’s explanation in the Badillo consolidated cases, thus: “We agree with the RTC that, insofar as appeals from the MTC to the RTC are concerned, the 1997 Rules of Civil Procedure do not mandate the dismissal of an appeal as a consequence of the nonpayment of the required fee. Martinez v. Court of Appeals holds that in such appeals, “the failure to pay the appellate docket fees does not automatically result in the dismissal of the appeal, the dismissal being discretionary on the part of the appellate court.” While that case was governed by Sections 20 and 23 of the Interim Rules and Guidelines issued by the Court on January 11, 1983 to implement the Judiciary Reorganization Act of 1981 (BP Blg. 129), the present Rules lead to a similar conclusion. Under the 1997 Rules of Civil Procedure, parties perfect an appeal from the judgment of the MTC to the RTC by filing a notice of appeal within the fifteen day reglementary period, as provided under Section 4 of Rule 40 and Section 9 of Rule 41: Rule 40 -“SEC. 4. Perfection of appeal; effect thereof. – The perfection of the appeal and the effect thereof shall be governed by the provisions of section 9, Rule 41. Rule 41-“SEC. 9. Perfection of appeal; effect thereof. - A party’s appeal by notice of appeal is deemed perfected as to him upon filing of the notice of appeal in due time. xxx

xxx

xxx

“In appeals by notice of appeal, the court loses jurisdiction over the case upon the perfection of the appeals filed in due time and the expiration of the time to appeal of the other party.” Fontanar v. Bonsubre is a case in point. It holds that in appeals from the MTC to the RTC, failure to pay the appellate docket fee within the fifteen-day reglementary period bestows on the appellate court a directory, not a mandatory, power to dismiss an appeal. The Court ratiocinated as follows: “x x x [T]his Court restated the importance and real purpose of the remedy of appeal as an essential part of our judicial system and advised the courts to proceed with caution so as not to deprive a party of a right to appeal with the instruction that every party-litigant should be afforded the amplest opportunity for the proper and just disposition if his cause, freed from the constraints of technicalities. Rightly so, for the payment of the appellate docket fee is not a requirement for the protection of the prevailing party, and non-compliance therewith within the time prescribed causes no substantial prejudice to anyone.” On the other hand, the cases cited by petitioners involve appeals -- not from the MTC to the RTC -- but from the RTC to the CA and from the CA to the SC, for which the payment of appellate fees is indeed mandatory according to the Rules. We quote Manalili v. Arsenio and De Leon: “Appeal is not a right, but a mere statutory privilege. Corollary to this principle is that the appeal must be exercised strictly in accordance with provisions set by law. x x x

“x x x [T]he payment of the appellate docket fee is not a mere technicality of law or procedure. It is an essential requirement, without which the decision or final order appealed from would become final and executory as if no appeal was filed at all.” In the instant cases, when the NHA filed a Notice of Appeal on February 22, 2000 -- two days before the appeal period lapsed – it perfected its appeal and the MTC thereby lost its jurisdiction. The MTC therefore acted without jurisdiction in issuing the May 23, 2000 Order and the May 30, 2000 Writ of Execution.” -0-0-0A notice of appeal was timely filed but the appeal docket fee was paid after three months when the notice of appeal because of the messenger’s inadvertence in securing postal money order. The trial court approved the notice of appeal basing the order in the phrase “in the interest of substantial justice.” Is the trial court correct? No. The bare invocation of the phrase “in the interest of substantial justice” is not a magic spell that will automatically allow the court to suspend procedural rules, despite jurisdictional bar. The rule may be relaxed only in exceptionally meritorious cases. The messenger’s alleged inadvertence to secure a postal money order for appellate docket fees is not a meritorious reason to justify as exception in our jurisprudence (Ilusorio vs. Ilusorio-Yap,). The finality of a decision is a jurisdictional event which cannot be made to depend on the convenience of the parties (Ocampo vs. CA,).

M. Appellate court may motu proprio dismiss the appeal

Prior to the transmittal of the original record or the record on appeal to the appellate court, the trial court may motu propio or on motion dismiss the appeal for having been taken out of time (Section 13, Rule 41, 1997 Revised Rules of Civil Procedure).

N. A final judgment or order is one that finally disposes of a case A final judgment or order is one that finally disposes of a case, leaving nothing more for the court to do in respect thereto, such as an adjudication on the merits which, on the basis of the evidence presented at the trial, declares categorically what the rights and obligations of the parties are and which party is in the right, or a judgment or order that dismisses an action on the ground of res judicata or prescription, for instance (Intramuros Tennis Club, Inc. vs. Philippine Tourism Authority,).

O. Questions of fact and questions of law distinguished A question of law arises when there is doubt as to what the law is on a certain state of facts while there is a question of fact when the doubt arises as to the truth or falsity of the alleged facts. A question of law may be resolved by the court without reviewing or evaluating the evidence. No examination of the probative value of the evidence would be necessary to resolve a question of law. The opposite is true with respect to questions of fact, which necessitate a calibration of the evidence (Macababbad vs. Masirag,). The nature of the issues to be raised on appeal can be gleaned from the appellant’s notice of appeal filed in the trial court and in his or her brief as appellant in the appellate court. In their Notice of Appeal, the respondents manifested their intention to appeal the assailed RTC order on legal grounds and “on the basis of the environmental facts.” Further, in their Brief, the petitioners argued that the RTC erred in ruling that their cause of action had prescribed and that they had “slept on their rights.” All these indicate that questions of facts were involved, or were at least raised, in the respondents’ appeal with the CA.

P. Issue on prescription may be a question of fact or question of law Issue on prescription may either be a question of law or fact; it is a question of fact when the doubt or difference arises as to the truth or falsity of an allegation of fact; it is a question of law when there is doubt or controversy as to what the law is on a given state of facts. The test of whether a question is one of law or fact is not the appellation given to the question by the party raising the issue; the test is whether the appellate court can determine the issue raised without reviewing or evaluating the evidence. Prescription, evidently, is a question of fact where there is a need to determine the veracity of factual matters such as the date when the period to bring the action commenced to run (Crisostomo vs. Garcia, G.R. No. 164787, January 31, 2006 cited in Macababbad vs. Masirag, G.R. No. 161237, January 14, 2009). The High Court in Macababbad said: “Ingjug-Tiro v. Casals instructively tells us too that a summary or outright dismissal of an action is not proper where there are factual matters in dispute which require presentation and appreciation of evidence. In this cited case whose fact situation is similar to the present case, albeit with a very slight and minor variation, we considered the improvident dismissal of a complaint based on prescription and laches to be improper because the following must still be proven by the complaining parties: first, that they were the co-heirs and co-owners of the inherited property; second, that their coheirs-co-owners sold their hereditary rights thereto without their knowledge and consent; third, that forgery, fraud and deceit were committed in the execution of the Deed of Extrajudicial Settlement and Confirmation of Sale since Francisco Ingjug who allegedly executed the deed in 1967 actually died in 1963, hence, the thumbprint found in the document could not be his; fourth, that Eufemio Ingjug

who signed the deed of sale is not the son of Mamerto Ingjug, and, therefore, not an heir entitled to participate in the disposition of the inheritance; fifth, that respondents have not paid the taxes since the execution of the sale in 1965 until the present date and the land in question is still declared for taxation purposes in the name of Mamerto Ingjug, the original registered owner, as of 1998; sixth, that respondents had not taken possession of the land subject of the complaint nor introduced any improvement thereon; and seventh, that respondents are not innocent purchasers for value. As in Ingjug-Tiro, the present case involves factual issues that require trial on the merits. This situation rules out a summary dismissal of the complaint.” Proper Mode of Appeal Since the appeal raised mixed questions of fact and law, no error can be imputed on the respondents for invoking the appellate jurisdiction of the CA through an ordinary appeal. Rule 41, Sec. 2 of the Rules of Court provides: Modes of appeal.

(a) Ordinary appeal - The appeal to the Court of Appeals in cases decided by the Regional Trial Court in the exercise of its original jurisdiction shall be taken by filing a notice of appeal with the court which rendered the judgment or final order appealed from and serving a copy thereof upon the adverse party. In Murillo v. Consul, this Court had the occasion to clarify the three (3) modes of appeal from decisions of the

RTC, namely: (1) ordinary appeal or appeal by writ of error, where judgment was rendered in a civil or criminal action by the RTC in the exercise of original jurisdiction, covered by Rule 41; (2) petition for review, where judgment was rendered by the RTC in the exercise of appellate jurisdiction, covered by Rule 42; and (3) petition for review to the Supreme Court under Rule 45 of the Rules of Court. The first mode of appeal is taken to the CA on questions of fact or mixed questions of fact and law. The second mode of appeal is brought to the CA on questions of fact, of law, or mixed questions of fact and law. The third mode of appeal is elevated to the Supreme Court only on questions of law.”

Q. Perfected is the appeal by filing a notice of appeal and paying the requisite docket fees, exception The general rule is that appeal is perfected by filing a notice of appeal and paying the requisite docket fees and other lawful fees, but admits an exception (Tanenglian vs. Lorenzo,). Tanenglian said: “However, all general rules admit of certain exceptions. In Mactan Cebu International Airport Authority v. Mangubat where the docket fees were paid six days late, we said that where the party showed willingness to abide by the rules by immediately paying the required fees and taking into consideration the importance of the issues raised in the case, the same calls for judicial leniency, thus: In all, what emerges from all of the above is that the rules of procedure in the matter of paying the docket fees must be followed. However, there are exceptions to the stringent requirement as to call for a relaxation of the application of the rules, such as: (1) most

persuasive and weighty reasons; (2) to relieve a litigant from an injustice not commensurate with his failure to comply with the prescribed procedure; (3) good faith of the defaulting party by immediately paying within a reasonable time from the time of the default; (4) the existence of special or compelling circumstances; (5) the merits of the case; (6) a cause not entirely attributable to the fault or negligence of the party favored by the suspension of the rules; (7) a lack of any showing that the review sought is merely frivolous and dilatory; (8) the other party will not be unjustly prejudiced thereby; (9) fraud, accident, mistake or excusable negligence without appellant’s fault; (10) peculiar legal and equitable circumstances attendant to each case; (11) in the name of substantial justice and fair play; (12) importance of the issues involved; and (13) exercise of sound discretion by the judge guided by all the attendant circumstances. Concomitant to a liberal interpretation of the rules of procedure should be an effort on the part of the party invoking liberality to adequately explain his failure to abide by the rules. Anyone seeking exemption from the application of the Rule has the burden of proving that exceptionally meritorious instances exist which warrant such departure. We have not been oblivious to or unmindful of the extraordinary situations that merit liberal application of the Rules, allowing us, depending on the circumstances, to set aside technical infirmities and give due course to the appeal. In cases where we dispense with the technicalities, we do not mean to undermine the force and effectivity of the periods set by law. In those rare cases where we did not stringently apply the procedural rules, there always existed a clear need to prevent the commission of a grave injustice. Our judicial system and the courts have always tried to

maintain a healthy balance between the strict enforcement of procedural laws and the guarantee that every litigant be given the full opportunity for the just and proper disposition of his cause. If the Highest Court of the land itself relaxes its rules in the interest of substantive justice, then what more the administrative bodies which exercise quasi-judicial functions? It must be emphasized that the goal of courts and quasi-judicial bodies, above else, must be to render substantial justice to the parties. In this case, petitioner was only one day late in paying the appeal fee, and he already stands to lose his titles to the subject properties. We find this too harsh a consequence for a day’s delay. Worthy to note is the fact that petitioner actually paid the appeal fee; only, he was a day late. That petitioner immediately paid the requisite appeal fee a day after the deadline displays his willingness to comply with the requirement therefor.”

---0---

MOTION FOR RECONSIDERATION (Rule 37) A. Two-fold nature of motion for reconsideration, purpose and importance As a rule, not all orders denying or granting motions for reconsideration are final orders. It depends upon the kind of incident to which the motion for reconsideration is filed or applied. If it is filed arising out of an interlocutory order, let say for instance, against a motion for bill of particulars, the order granting or denying the same is interlocutory in character, because the court has still something to do with the case. It does not finally dispose of the case. If it is filed against a judgment, the order granting or denying the same is in the nature of a final order. This is the so-called the two-fold

functional character of a motion for reconsideration – the interlocutory and final, depending upon its usage and applicability. The importance of knowing whether an order granting or denying a motion for reconsideration is to determine what will be the next applicable remedy is available to the aggrieved party. The aggrieved party is not allowed to file a second motion for reconsideration of a judgment or final order (Paragraph 2, Section 5, rule 37, 1997 Revised Rules of Civil Procedure). If an aggrieved party files a second motion for reconsideration of a judgment or final order, thus allowing the period to appeal to lapsed. Therefore, he shall loose his right to appeal, and the consequence of which, he cannot resort to the filing of a petition for certiorari under Rule 65, because it is not a substitute for a lost appeal (Zarate vs. Maybank Philippines, Inc,).

A. Motion for reconsideration that fails to comply with the rules on motion not toll the period to file appeal The motion for reconsideration which is filed without compliance with the rules on motion is treated as a mere scrap of paper and will not toll the period to file appeal. In People vs. Court of Appeals, , the High Court held that a motion without a notice of hearing is pro forma, a mere scrap of paper that does not toll the period to appeal. Said the High Court: “Not only did the defect render the motion for reconsideration itself unworthy of consideration, it more crucially failed to toll the period to appeal. A motion without a notice of hearing is pro forma, a mere scrap of paper that does not toll the period to appeal, and upon the expiration of the 15-day period, the questioned order or

decision becomes final and executory. The rationale behind this rule is plain: unless the movant sets the time and place of hearing, the court will be unable to determine whether the adverse party agrees or objects to the motion, and if he objects, to hear him on his objection, since the rules themselves do not fix any period within which he may file his reply or opposition.” ---0---

MOTION FOR NEW TRIAL (Rule 37) A. Concept, purpose and its allowance A new trial is not a refuge for the obstinate.17 New trial is a remedy that seeks to ‘temper the severity of a judgment or prevent the failure of justice.’ The Rules allows the courts to grant a new trial when there are errors of law or irregularities prejudicial to the substantial rights of the accused committed during the trial, or when there exists newly discovered evidence. (Ybiernas vs. Tanco-Gabaldon,).

B. Grant or denial is discretionary The grant or denial of a new trial is addressed to the sound discretion of the court which cannot be interfered with unless a clear abuse thereof is shown (Ybiernas vs. Tanco-Gabaldon,).

17

Viking Industrial Corporation vs. Court of Appeals, G.R. No. 143794, July 13, 2004 cited in Atlas Consolidated Mining and Development Corporation vs. CIR, G.R. Nos. 141104 & 148763, June 8, 2007.

C. Only on newly discovered evidence, requirements A new trial may be granted on the ground of newly discovered evidence, provided that the following must be shown: (1) that the evidence was discovered after trial; (2) that such evidence could not have been discovered and produced at the trial even with the exercise of reasonable diligence; (3) that it is material, not merely cumulative, corroborative, or impeaching; and (4) that the evidence is of such weight that it would probably change the judgment if admitted (Custodio vs. Sandiganbayan,). If the alleged newly discovered evidence could have been very well presented during the trial with the exercise of reasonable diligence, the same cannot be considered newly discovered (Custodio vs. Sandiganbayan,). The movant for a new trial must not only act in a timely fashion in gathering evidence in support of the motion; he must act reasonably and in good faith as well (Custodio vs. Sandiganbayan,). Only a final judgment or order may be the subject of a motion for new trial (Ybiernas vs. Tanco-Gabaldon,).

D. Ignorance, inexperience or incompetence of counsel do not qualify as a ground for new trial. Blunders and mistakes in the conduct of the proceedings in the trial court as a result of the ignorance, inexperience or incompetence of counsel do not qualify as a ground for new trial. If such were to be admitted as valid reasons for re-opening cases, there would never be an end to

litigation so long as a new counsel could be employed to allege and show that the prior counsel had not been sufficiently diligent, experienced or learned. This will put a premium on the willful and intentional commission of errors by counsel, with a view to securing new trials in the event of of conviction, or an adverse decision (Uy vs. First Metro Integrated Steel Corp,).

E. Remedy of adverse party if motion for new trial is granted by the trial court. If a motion for new trial is granted by the trial court, the remedy of the adverse party if the act of granting the motion is believed to be tainted with grave abuse of discretion is to file a petition for certiorari under Rule 65 of the 1997 Revised Rules of Civil Procedure, because: first, this is authorized by Section 1, paragaraph 2 (a) of Rule 41 of the same Rules of Procedure says that no appeal may be taken from an order denying a motion for new trial or reconsideration; second, this is because an order granting a motion for new trial is interlocutory character (See Section 1, paragraph 2 (c), Rule 41, 1997 Revised Rules of Civil Procedure). It is an order which does not dispose of the case completely because it leaves something for the trial court to do (Tan vs. Republic, ), as the original judgment or final order is vacated (See Section 6, Rule 37, 1997 Revised Rules of Civil Procedure, Effect of granting of motion for new trial). The trial court in trial de novo shall re-try the case and to decide it after the retrial; and third, it is only when such interlocutory order was rendered without or in excess of jurisdiction or with grave abuse of discretion that certiorari under Rule 65 may be resorted to (Silverio, Jr. vs. CA, G.R. No. 178933, September 16, 2009 citing 1 F. Regalado, Remedial Law Compendium 540 (8th revised ed.). If it is believed that the act of the trial court in granting the motion for new trial is not tainted with grave abuse of discretion, the remedy of the adverse party is to wait for the new judgment to be rendered after trial de novo, and if the judgment is not favorable, then file a notice of appeal. Again, this is because an interlocutory order generally cannot be appealed separately from the judgment. It is only after a judgment has been rendered in the case that the ground for the appeal of the interlocutory

order may be included in the appeal of the judgment itself (Silverio, Jr. vs. CA,).

F. Remedy of the aggrieved party if motion for new trial is denied by the trial court. The remedy available is not petition for certiorari under Rule 65, but appeal under Rule 41 of the 1997 Revised Rules of Civil Procedure. This is because: First, Section 9 of Rule 37 of the 1997 Revised Rules of Civil Procedure is very clear on the matter that an order denying a motion for new trial is not appealable (the order itself denying a motion for new trial is not appealable), the remedy being an appeal from the judgment or final order; Second, an order denying a motion for new trial is no longer an interlocutory character; Third, certiorari is only available if there is no appeal, or any plain, speedy, and adequate remedy in the ordinary course of law (See Rule 65). Appeal is available. Lastly, it is a final order, because it decisively puts to a close, or disposes of a case or a disputed issue leaving nothing else to be done by the court in respect thereto (Santo Tomas University Hospital vs. Surla, G).

Section 9, Rule 37 of the Rules of Court provides: “SEC. 9. Remedy against order denying a motion for new trial or reconsideration. –An order denying a motion for new trial or reconsideration is not appealable, the remedy being an appeal from the judgment or final order.”

Section 1, Rule 41 of the Rules of Court provides: “SECTION 1. Subject of appeal. –An appeal may be taken from a judgment or final order that completely disposes of the case, or of a particular matter therein when declared by these Rules to be appealable. No appeal may be taken from: (a) An order denying a motion for new trial or reconsideration; “xxx xxx In all the above instances where the judgment or final order is not appealable, the aggrieved party may file an appropriate special civil action under Rule 65.” Section 1, Rule 65 of the Rules of Court provides: “SECTION 1. Petition for certiorari. –When any tribunal, board or officer exercising judicial or quasijudical functions has acted with grave abuse of discretion amouting to lack or excess of jurisdiction, and there is no appeal, or any plain, speedy, and adequate remedy in the course of law, a person aggrieved thereby may file a verified petition in the proper court, alleging the facts with certainty and prayuing that judgment be rendered annulling or modifying the proceedings of such tribunal, board or officer, and granting such incidental releifs as law and justice may require.”

Section 9 of Rule 39, 1997 Revised Rules of Civil Procedure says that an order denying a motion for new trial or reconsideration is not appealable, the remedy being an appeal from the judgment or final order, while, Section 1, paragaraph 2 (a) of Rule 41 of the same Rules of

Procedure says that no appeal may be taken from an order denying a motion for new trial or reconsideration. Santo Tomas University Hospital case had the occasion to distinguish the concepts of a final judgment or order from an interlocutory issuance, thus: a final judgment or order is that the former decisively puts to a close, or disposes of a case or a disputed issue leaving nothing else to be done by the court in respect thereto. Once that judgment or order is rendered, the adjudicative task of the court is likewise ended on the particular matter involved;18 and it is interlocutory if its effects would only be provisional in character and would still leave substantial proceedings to be further had by the issuing court in order to put the controversy to rest.19 While it is true that Section 9 of Rule 39, 1997 Revised Rules of Civil Procedure says an order denying a motion for new trial or reconsideration is not appealable, however, it also says that the remedy is appeal from the judgment or final order. Therefore, the aggrieved party should concentrate to the decision or judgment itself after the motion for new trial is denied for purposes of filing an appeal (via notice of appeal, See Section 3, Rule 40 on how to appeal) under Rule 41, and not to elevate the order of denial (of motion for new trial) via certiorari under Rule 65. Thus, applying Section 9, Rule 37 in relation to Section 1, paragaraph 2 (a), Rule 41 and Section 1, Rule 65 of the 1997 Rules of Civil Procedure it is humbly opined that the available remedy to the aggrieved party is not certiorari under Rule 65, but appeal. However, the case of Uy vs. First Metro Integrated Steel Corp, is different. In this case, it was held that if a motion for new trial is denied, the aggrieved party may resort to the filing of a petition for certiorari under Rule 65 of the 1997 Revised Rules of Civil Procedure from the denial of the motion for new trial. Certiorari was allowed by the High Court because the denial of the motion for new trial is not on the substance of the motion but on ground of technicality. In that case the trial court denied the motion for new trial because the same was filed out of time, 18

Santo Tomas University Hospital vs. Surla, G.R. No. 129718, August 17, 1998 citing Investments, Inc. vs. Court of Appeals, 147 SCRA 334; Denso (Phils.,) Inc. vs. Intermediate Appellate Court, 148 SCRA 280. 19 Santo Tomas University Hospital vs. Surla, G.R. No. 129718, August 17, 1998 citing Bairan vs. Tan Siu Lay, 18 SCRA 1235.

and was found out by the High Court that the same was filed within the reglementary period. According to the High Court, citing Section 1, Rule 41, that certiorari under Rule 65 is applicable because no appeal may be taken from an order denying a motion for new trial or reconsideration. The High Court’s observation in Uy: “A scrutiny of the records discloses that while the Motion for New Trial was received by the trial court on April 28, 2003, the date on the Registry Receipt attached to the Affidavit of Service as well as that stamped on the envelope which contained the copy of the motion, reveals that it was filed and served by registered mail on April 21, 2003, a Monday, because April 19, 2003, the last day for filing the same was a Saturday. Section 1, Rule 22 of the Rules of Court states in no uncertain terms that if the last day of the period thus computed falls on a Saturday, a Sunday, or a legal holiday in the place where the court sits, the time shall not run until the next working day. Thus, the motion was actually filed on time it having been filed on April 21, 2003, the next working day, following the last day for filing which fell on a Saturday. Section 9, Rule 37 of the Rules of Court which provides that the remedy to an order denying a motion for new trial is to appeal the judgment or final order, must be read in conjunction with Section 1, Rule 41 which provides that: SEC. 1. Subject of appeal. – An appeal may be taken from a judgment or final order that completely disposes of the case, or of a particular matter therein when declared by these rules to be appealable. No appeal may be taken from: (a) An order denying a motion for new trial or reconsideration;

xxxx In all the above instances where the judgment or final order is not appeasable, the aggrieved party may file an appropriate special civil action under Rule 65. (Emphasis supplied) Thus, the filing by the petitioner of a petition for certiorari with the Court of Appeals from the denial of the motion for new trial by the trial court is proper.”

G. Failure of counsel to attend the hearings for the reception of evidence is inexcusable negligence In Uy vs. First Metro Integrated Steel Corp, it was held that: “xxx xxx xxx, we find that the trial court correctly denied petitioner’s motion for new trial. Section 1, Rule 37 provides that a motion for new trial may be filed within the period for taking an appeal based on the following grounds: (a) Fraud, accident, mistake or excusable negligence which ordinary prudence could not have guarded against and by reason of which such aggrieved party has probably been impaired in his rights; or xxxx Negligence to be excusable must be one which ordinary diligence and prudence could not have guarded against.

In the instant case, we find the negligence of petitioner’s counsel in failing to attend the hearings for the reception of evidence inexcusable. The trial court scheduled the hearing for the reception of petitioner’s evidence seven times. The initial hearing set on February 28, 2001 was cancelled because petitioner allegedly had influenza. The hearings scheduled on April 26, 2001 and May 10, 2001 were cancelled and moved to October 25, 2001 and December 13, 2001. Petitioner was represented by Atty. Carpio, Jr. as collaborating counsel during the hearing on October 25, 2001 but no evidence was presented. Instead, the hearing was cancelled. On December 13, 2001, Atty. Bañares, petitioner’s new counsel, appeared but he requested for a resetting. On February 14, 2002, Atty. Bañares moved to postpone the hearing to February 28, 2002 as previously scheduled. On February 28, 2002, Atty. Bañares arrived late. Scrutiny of the records disclose that the hearings were postponed or cancelled without any justification. However, the trial court accommodated the requests for postponement or resetting in order to accord petitioner due process. Under the circumstances, we find petitioner’s counsel’s failure to attend the seven scheduled hearings without justifiable reason tantamount to inexcusable neglect. As such, it cannot be a ground for new trial.”

H. Affidavit of merit is required in filing a motion for new trial Also in Uy vs. First Metro Integrated Steel Corp, , it was held that: “In addition, the Rule requires that motions for new trial founded on fraud, accident, mistake or excusable negligence must be accompanied by affidavits of merits, i.e., affidavits showing the facts (not mere conclusions or opinions) constituting the valid cause of action or defense

which the movant may prove in case a new trial is granted, because a new trial would serve no purpose and would just waste the time of the court as well as the parties if the complaint is after all groundless or the defense is nil or ineffective. Under the Rules, the moving party must show that he has a meritorious defense. The facts constituting the movant’s good and substantial defense, which he may prove if the petition were granted, must be shown in the affidavit which should accompany the motion for a new trial.20 We examined petitioner’s Affidavit of Merit and find that it did not contain clear statements of the facts constituting a good and valid defense which he might prove if given the chance to introduce evidence. The allegations that he has a “meritorious defense” and a “good cause” are mere conclusions which did not provide the court with any basis for determining the nature and merit of the case. An affidavit of merit should state facts, and not mere opinion or conclusions of law. Petitioner’s motion for new trial and affidavit of merit did not mention the evidence which he was prevented from introducing, nor did it allege that such evidence would change the outcome of the case.” ---0---

PETITION FOR RELIEF FROM JUDGMENT (Rule 38)

A. Concept, nature and principles Section 3, Rule 38 of the 1997 Rules of Civil Procedure provides that a verified petition for relief must be filed within sixty (60) days after the petitioner learns of the judgment, final order, or other proceeding to be

set aside and not more than six (6) months after such judgment or final order has been entered or such proceeding has been taken. Rule 38 of the Rules of Court only applies when the one deprived of his right is a party to the case. If the movant is not a party or never been a party to the case or even summoned to appear in case the remedy of petition for relief from judgment under Rule 38 of the Rules of Court is not proper. The word used is “against a party” (Lagula vs. Casimiro,). A petition for relief from judgment is an exception to the public policy of immutability of final judgments (Madarang vs. Morales,). It is premised on equity; an act of grace and not regarded with favor, and granted only in exceptional cases (Dirige vs. Biranya,), especially to any person against whom a decision or order is entered into through fraud, accident, mistake or excusable negligence (Somoso vs. CA,). It is dismissible outright if filed beyond the reglementary period (Madarang vs. Morales,). Good and substantial defense are also included as requirements sine qua non conditions for its proper allowance (Dirige vs. Biranya,). When a party has another remedy available to him, which may be either a motion for new trial or appeal from an adverse decision of the trial court, and he was not prevented by fraud, accident, mistake or excusable negligence from filing such motion or taking such appeal, he cannot avail himself of this petition. Indeed, relief will not be granted to a party who seeks avoidance from the effects of the judgment when the loss of the remedy at law was due to his own negligence; otherwise the petition for relief can be used to revive the right to appeal which had been lost thru inexcusable negligence (Tuason vs. Court of Appeals,). If a party has another adequate remedy available to him, which was either a motion for new trial or appeal from adverse decisions of the lower court, and he was not prevented by fraud, accident, mistake or excusable negligence from filing such motion or taking the appeal he cannot avail himself of the relief provided in Rule 38 (Somoso vs. CA,). A party is not entitled to relief under Rule 38, Section 2, of the Rules of Court if he was not prevented from filing his notice of appeal by fraud, accident, mistake, or excusable negligence. Such relief will not be

granted to a party who seeks to be relieved from the effects of the judgment, when the loss of the remedy at law was due to his own negligence or to a mistaken mode of procedure for that matter; otherwise, the petition for relief will be tantamount to reviving the right of appeal, which has already been lost either due to inexcusable negligence or due to a mistake of procedure by counsel (Fukuzumi vs. Sanritsu Great International Corporation,). The petition for relief from judgment must be accompanied with affidavits showing the fraud, accident, mistake, or excusable negligence relied upon, and the facts constituting petitioner’s good and substantial cause of action or defense (Public Estates Authority vs. Yujuico,). This is an indispensable statutory requirement under Section 3, Rule 38 of the 1997 Revised Rules of Civil Procedure which must be strictly observed. Affidavit of merit serves as the jurisdictional basis for a court to entertain a petition for relief, and when a petition for relief is flawed by such serious defect, the court with which the petition is filed is not called upon to entertain the same (Torno vs. IAC,).

B. Fraud as ground must be extrinsic or collateral Like annulment of judgment, the fraud as ground for filing a petition for relief from judgment must be extrinsic or collateral. The facts upon which the extrinsic fraud is based have not been controverted or resolved in the case where the judgment sought to be annulled was rendered. For this purpose, fraud is regarded as extrinsic or collateral where it has prevented a party from having a trial or from presenting all of his case to the court. Intrinsic fraud takes the form of acts of the party in a litigation during the trial, such as the use of forged instruments or perjured testimony which did not affect the presentation of the case, but did prevent a fair and just determination of the case (Libudan vs. Gil,).

C. Mistake and excusable negligence as a grounds The mistake contemplated by Rule 38 of the Rules of Court pertains generally to mistake of fact, not of law, which relates to the case. The word “mistake” which grants relief from judgment does not apply and

was never intended to apply to a judicial error which the court might have committed in the trial. Such error may be corrected by means of an appeal (Agan vs. Heirs of Sps. Andres Nueva and Diosdada Nueva,), and not by petition for relief form judgment. In Madarang vs. Morales, , it was held that a failure of petitioners’ former counsel to file the notice of appeal within the reglementary period is not excusable negligence. The High Court said that that kind of an argument stereotypes and demeans senior citizens. According to the High Court, this cannot be done as it asked to assume that a person with advanced age is prone to incompetence.

D. Reglementary period, when reckoned The 60-day period must be counted after petitioner learns of the judgment or final order, and the 6-month period is counted from the finality of judgment or final order. These are the so-called double period. These double periods are both inextendible and uninterruptible; is jurisdictional and should be strictly complied with (Madarang vs. Morales,). The 60-day reglementary period shall reckone from the time the party’s counsel receives notice of the decision for notice to counsel of the decision is notice to the party for purposes of Section 3 of Rule 38 (Mercury Drugs Corporation vs. CA,). The grace period allowed by the rule within which to file petition for relief from judgment must be taken as absolutely fixed, inextendible, never interrupted and cannot be subjected to any condition or contingency. Because the period fixed is itself devised to meet a condition or contingency, the equitable remedy is an act of grace, as it were, designed to give the aggrieved party another and last chance and failure to avail of such last chance within the grace period fixed is fatal (Martinez vs. Workmen's Compensation Commission,).

E. Must be filed in the same court that rendered the judgment

It must be filed in the same court that that rendered the judgment and in the same cause wherein the judgment was rendered. If the court finds the allegations of the petition to be true, it shall set aside the judgment and try the principal case upon its merits as if a timely motion for new trial had been granted therein (Braca vs. Tan,).

F. Petition for relief is only available against a final and executory judgment In Gomez vs.Montalban, , it was held that petitioner’s petition for relief from judgment under Rule 38 of the Rules of Court was premature and inappropriate, because when he filed his petition for relief the judgment on May 28, 2004 against the judgment dated May 4, 2004 which he received on May 14, 2004, the judgment had not attained finality, and the 5-day period within which to file a motion for reconsideration or appeal had not yet lapsed.

G. Petition is deniable when the loss of the remedy at law was due to his own negligence or mistake The petition for relief from judgment will not be granted to a party who seeks to be relieved from the effects of the judgment when the loss of the remedy at law was due to his own negligence or to a mistaken mode of procedure for that matter, because if it will be granted the petition for relief will be tantamount to reviving the right of appeal, a remedy which has already been lost either due to inexcusable negligence or due to a mistake of procedure by counsel (Fukuzumi vs. Sanritsu Great International Corporation,).

H. Cases: When a petition for relief is filed 61 days from receipt of the notice of dismissal or one day late, the petition for relief must be ordered denied. While it is true that the law gives an exception or ‘last chance’ of a timely petition for relief from judgment within the reglementary period (within

60 days from knowledge and 6 months from entry of judgment) under Rule 38 supra, but such grace period must be taken as 'absolutely fixed, inextendible, never interrupted and cannot be subject to any condition or contingency. Because the period fixed is itself devised to meet a condition or contingency (fraud, accident, mistake or excusable neglect), the equitable remedy is an act of grace, as it were, designed to give the ag-grieved party another and last chance' "and failure to avail of such last chance within the grace period fixed by the statute or Rules of Court is fatal." (Turqueza vs. Hernando,). A petition for relief which was filed 61 days from receipt of the notice of dismissal or one day late was held filed out of time (Philippine Rabbit Bus Lines vs. Judge Arciaga,). The High Court said in Philippine Rabbit Bus Lines: “It is undisputed that the Petition for Relief in this case was filed 61 days from receipt of the notice of dismissal or one day late. In fact, the records show that counsel for private respondent learned of the dismissal on the same day, April 29, 1967, when he arrived late for the hearing so that the Petition for Relief was at least eight (8) days late. The records further show that counsel for private respondent did not move for reconsideration of the Order of dismissal, nor for new trial. Neither did he appeal, thereby allowing the decision to become final and executory. As a last resort, he could have availed of the sixty day period provided for by Rule 38 to file a Petition for Relief from judgment but again he allowed this opportu-nity to lapse. Indeed, to him is applicable, the well known maxim that "equity aids the vigilant, not those who slumber on their rights." (Henson v. Director of Lands, 55 Phil. 586). In the case of Turqueza v. Hernando (97 SCRA 488 [1980]) the Supreme Court in disallowing the reopening of the case which has become final, ruled that there is no justification in law and in fact, for respondent judge's void act of ordering the reopening of the case which has become final and executory.

Thus, the Court held: "The Court has said time and again that the doctrine of finality of judgments is grounded on fundamental considerations of public policy and sound practice that at the risk of occasional error the judgments of courts must become final at some definite date fixed by law. The law gives an exception or 'last chance’ of a timely petition for relief from judgment within the reglementary period (within 60 days from knowledge and 6 months from entry of judgment) under Rule 38 supra, but such grace period must be taken as 'absolutely fixed, inextendible, never interrupted and cannot be subject to any condition or contingency. Because the period fixed is itself devised to meet a condition or contingency (fraud, accident, mistake or excusable neg-lect), the equitable remedy is an act of grace, as it were, designed to give the ag-grieved party another and last chance' "and failure to avail of such last chance within the grace period fixed by the statute or Rules of Court is fatal." (Turqueza v. Hernando, supra). In expressly reiterating the above-quoted decision the Supreme Court in Arcilla v. Arcilla (138 SCRA 56 [1985]), held that the Rule is that, for a petition for relief under Rule 38 to be entertained by the court, the petitioner must satisfactorily show that he has faithfully and strictly complied with the provisions of said Rule. Consequently, it is incumbent upon the petitioner to show that the said petition was filed within the reglementary period specified in Sec. 3, of the same, otherwise on this ground alone, the petition should be dismissed.”

I. Petition for relief will not lie when plaintiff filed to personally notify defendant of court processes Petition for relief will not lie if plaintiff fails to notify the defendant of court processes and/or fails to search for the address of defendant so that he may be properly notified by the Court. Plaintiff is not duty bound to personally notify the defendant of court processes. It is not the plaintiff’s duty to search for the defendant’s address so that they may be properly notified by the Court. This is not the kind of fraud contemplated by law. Bad faith cannot be presumed from inaction where there is no duty to act. The grounds not having been clearly established, petition for relief will not lie (David vs. CA,).

J. Counsel’s negligence not a ground for petition for relief from judgment The excusable negligence referred to in Rule 38 to warrant the filing of the petition for relief from judgment is negligence of party NOT negligence of counsel (Somoso vs. CA,). However, where reckless or gross negligence of counsel deprives the client of due process of law; or (2) when its application will result in outright deprivation of the client’s liberty or property; or (3) where the interests of justice so require, the courts must step in and accord relief to a client who suffered thereby (Sarraga, Sr., vs. Banco Filipino Savings and Mortgage Bank,).

K. Petition for relief from judgment filed out of time shall not be granted, exceptions The following are the exceptions: (1) where reckless or gross negligence of counsel deprives the client of due process of law; (2) when its application will result in outright deprivation of the client’s liberty or property; or

(3) where the interests of justice so require (Sarraga, Sr., vs. Banco Filipino Savings and Mortgage Bank, ). In People’s Homesite and Housing Corporation vs.), it was held that the counsel’s client was denied of due process when the lawyer failed to inform his clients of the scheduled hearing of the case and the hearing proceeded despite the client’s absence. In that case, the Court sensed that there was something fishy with the actuations of the lawyer which deprived client of their day in court. In Sarraga vs. Banco Filipino Savings and Mortgage Bank, , the negligence of newly-hired clerk with lack of work experience in the law office who merely left the court order in her desk and eventually misplaced it and failed to bring the matter to the attention of the lawyer before the lawyer was appointed as Senior State Prosecutor in the Department of Justice is gross negligence on the part of the lawyer, and because such negligence will result in the deprivation of the client’s property petition for relief from judgment was allowed. Obtaining these circumstances, the client should not be made to suffer the consequences.

L. Court can grant the relief different from that what is prayed in the petition Obviously, the court can grant relief which is different from that what is prayed for in the petition. The prayer in a petition for relief from judgment under Rule 38 is not necessarily the same prayer in the petitioner's complaint, answer or other basic pleading, because once a petition for relief is granted and the judgment subject thereof set aside, and further proceedings are thereafter had, the court in its judgment on the merits may properly grant the relief sought in the petitioner's basic pleadings, although different from that stated in his petition for relief (Cheesman vs. IAC,).

---0---

PETITION FOR RELIEF IS DENIED; REMEDY A. If denied, petition for certiorari under Rule 65 may be filed When a petition for relief for judgment is denied, the aggrieved party may file a petition for certiorari under Rule 65 of the 1997 Revised Rules of Civil Procedure. Section 1(b), Rule 41 of the 1997 Revised Rules of Civil Procedure is clear. It says that no appeal may be taken from an order denying a petition for relief from judgment, and instead, the aggrieved party may file an appropriate special civil action under Rule 65. The normal remedy against a judgment denying relief under Rule 38 is appeal wherein the aggrieved party may assail the judgment on the merits, upon the ground that it is not supported by the evidence or it is contrary to law. However, if appeal would not be a remedy adequate under the circumstances, since it would not promptly relieve the aggrieved party from the injurious effects of acts of the trial court or tribunal wherein in the latter for example has authorized execution of the judgment, a resort to the special civil action of certiorari may exceptionally be allowed.21 ---0---

REMEDY OF ADVERSE PARTY IF PETITION FOR RELIEF IS GRANTED How if the petition for relief from judgment is granted, what is the remedy available to the adverse party? Wait for the judgment, and if the judgment is not favorable, then file a notice of appeal. This is because an interlocutory order generally cannot be appealed separately from the judgment. It is only after a 21

PCGG vs. Sandiganbayan, G.R. No. 100733, June 18, 1992 citing Saludes vs. Pajarillo, 78 Phil. 754; Liwanag vs. Castillo, Oct. 20, 1959; Occeña vs. Jabson, Oct. 29, 1986, Silvestre vs. Torres, 57 Phil. 885; Pachoco vs. Tumungday, May 25, 1960, Lopez vs. Alvendia, Dec. 24, 1964, cited in Moran, op. cit., Vol. 3, p. 177.

judgment has been rendered in the case that the ground for the appeal of the interlocutory order may be included in the appeal of the judgment itself (Silverio, Jr. vs. CA, G.R. No. 178933, September 16, 2009). But if the order granting the petition for relief is believed to be tainted with graved abuse of discretion, though not covered by Section 1(b), Rule 41 of the 1997 Revised Rules of Civil Procedure as it only covers a situation wherein a petition for relief from judgment is denied allowing the aggrieved party to resort to special civil action under Rule 65, if tainted with graved abuse of discretion, the adverse party may still resort to filing a petition certiorari under Rule 65 of the 1997 Revised Rules of Civil Procedure by authority of Section 1(c), Rule 41 of the same Rules, because the order interlocutory nature; an an order is one which does not dispose of the case completely but leaves something to be decided upon (Tan vs. Republic,). It is only when such interlocutory order was rendered without or in excess of jurisdiction or with grave abuse of discretion that certiorari under Rule 65 may be resorted to (Silverio, Jr. vs. CA,).

B. Motion for reconsideration is a requirement A motion for reconsideration of the order denying the petition for relief from judgment is the plain, speedy, and adequate remedy in the ordinary course of law. Therefore it is a mandatory requirement to resort first to the filing of a motion for reconsideration. In Madarang vs. Morales, , the petition for certiorari assailing the denial of the petition for relief from judgment was ordered dismissed for failure of the petitioner to avail first the remedy of filing a motion for reconsideration.

---0---

ANNULMENT OF JUDGMENT (Rule 47) A. Concept, where to file and grounds

Annulment of judgment under Rule 47 of the 1997 Revised Rules of Civil Procedure is a recourse equitable in character and allowed only in exceptional cases where the ordinary remedies of new trial, appeal, petition for relief or other appropriate remedies are no longer available through no fault of petitioner.22

Rule 47 of the 1997 Revised Rules of Civil Procedure covers annulment by the Court of Appeals of judgments or final orders and resolutions in civil actions of Regional Trial Courts for which the ordinary remedies of new trial, appeal, petition for relief or other appropriate remedies could no longer be availed of through no fault of the petitioner. It is a remedy in law independent of the case where the judgment sought to be annulled is rendered (Macalalag vs. Ombusdman,). This remedy may not be invoked: (1) where the party has availed himself of the remedy of new trial, appeal, petition for relief or other appropriate remedy and lost therefrom, or (2) where he has failed to avail himself of those remedies through his own fault or negligence (Macalalag vs. Ombusdman,).

All annulment of judgment shall be within the exclusive original jurisdiction of the Court of Appeals. The grounds are extrinsic fraud and lack of jurisdiction (Paulino vs. CA,). Through jurisprudential pronouncements, denial of due process is included as an additional ground (Biaco vs. Philippine Countryside Rural Bank, G). 22

City Government of Tagaytay vs. Judge Guerrero, G.R. Nos. 140743 & 140745 and Justice Ameurfina Melencio-Herrera vs. Judge Guerrero, G.R. Nos. 141451-52, September 17, 2009.

B. Fraud as ground, extrinsic or collateral not intrinsic Fraud has been regarded as extrinsic or collateral. It is only extrinsic or collateral fraud NOT intrinsic fraud that can serve as a basis for annulment of judgment (Macabingkil vs. People's Homesite and Housing Corporation,). To become a basis for annulment of judgment, fraud has to be extrinsic or actual (Ancheta vs. Guersey-Dalaygon,), and refers to acts outside the trial (Ybañez vs. CA,). Fraud is extrinsic or collateral where it is on the effect of which prevent a party from having a trial, or real contest, or from presenting all of his case to the court, or where it operates upon matters pertaining, not to the judgment itself, but to the manner in which it was procured so that there is not a fair submission of the controversy (Macabingkil vs. People's Homesite and Housing Corporation,). It takes the form of acts of a party in a litigation during the trial, such as the use of forged instruments or perjured testimony, which did not affect the presentation of the case, but did prevent a fair and just determination of the case (Libudan vs. Palma,). Extrinsic fraud refers to any fraudulent act of the prevailing party in the litigation which is committed outside of the trial of the case, whereby the defeated party has been prevented from exhibiting fully his side of the case, by fraud or deception practiced on him by his opponent (Macabingkil vs. People's), or acts of a party at a trial which prevented a fair and just determination of the case and which could have been litigated and determined at the trial or adjudication of the case. Examples of intrinsic fraud are falsification and false testimony (Ybañez vs. CA,). According to Ancheta vs. Guersey-Dalaygon, , fraud takes on different shapes and faces. In that case, the High Court stated that “man in his ingenuity and fertile imagination will always contrive new schemes to fool the unwary.” Said the High Court: “There is extrinsic fraud within the meaning of Sec. 9 par. (2), of B.P. Blg. 129, where it is one the effect of

which prevents a party from hearing a trial, or real contest, or from presenting all of his case to the court, or where it operates upon matters, not pertaining to the judgment itself, but to the manner in which it was procured so that there is not a fair submission of the controversy. In other words, extrinsic fraud refers to any fraudulent act of the prevailing party in the litigation which is committed outside of the trial of the case, whereby the defeated party has been prevented from exhibiting fully his side of the case by fraud or deception practiced on him by his opponent. Fraud is extrinsic where the unsuccessful party has been prevented from exhibiting fully his case, by fraud or deception practiced on him by his opponent, as by keeping him away from court, a false promise of a compromise; or where the defendant never had any knowledge of the suit, being kept in ignorance by the acts of the plaintiff; or where an attorney fraudulently or without authority connives at his defeat; these and similar cases which show that there has never been a real contest in the trial or hearing of the case are reasons for which a new suit may be sustained to set aside and annul the former judgment and open the case for a new and fair hearing.”

C. Extrinsic fraud already availed of as ground in petition for relief or new trial, barred. If extrinsic fraud was availed of or could have been availed of in a motion for new trial or petition for relief, the extrinsic fraud can no longer be availed of as a ground for annulment of judgment. It is considered barred. In Tolentino vs. Judge Leviste, , the High Court said that it is provided in Section 2 of Rule 47 that extrinsic fraud shall not be a valid ground if it was availed of, or could have been availed of, in a motion for new trial or petition for relief. In other words, it is effectively barred if it could have been raised as a ground in an available remedial measure.

D. Extrinsic fraud, prescriptive period According to Article 1391 of the Civil Code of the Philippines, in case of fraud an action for annulment must be brought within four (4) years from the time of the discovery of the same. It commences to run from the discovery of the fraud or fraudulent act or acts. Therefore, annulment of judgment based on extrinsic fraud under Rule 47 of the 1997 Revised Rules of Civil Procedure must be brought within four (4) years from the discovery of the fraud (Ancheta vs. Guersey-Dalaygon, G).

E. Intrinsic fraud, concept, examples and cases Intrinsic fraud is not sufficient to attack a judgment (Yatco vs. Sumagui,).

F. Introduction of false affidavit purely intrinsic The use of a false affidavit of loss is intrinsic because it is similar to the use of it during trial of forged instruments or perjured testimony. It does not constitute extrinsic fraud to warrant the invalidation of a final judgment (Demetriou vs. CA, The use of a forged instrument constituted only intrinsic fraud for while perhaps it prevented a fair and just determination of a case, the use of such instrument or testimony did not prevent the adverse party from presenting his case fully and fairly (Palanca vs. Republic,).

G. Lack of jurisdiction as a ground for annulment Lack of jurisdiction as a ground for annulment of judgment refers to either lack of jurisdiction over the person of the defending party or over the subject matter of the claim (Paulino vs. CA, June 04, 2014). A petition for annulment of judgment must be based on lack of jurisdiction. Meaning, absence of or no jurisdiction; and that the court should not have taken cognizance of the petition because the law does not

vest it with jurisdiction over the subject matter. The petitioner must show not merely an abuse of jurisdictional discretion but an absolute lack of jurisdiction (Durisol Philippines, Inc. vs. CA,). If a complaint is filed and there is absence, or lack, of jurisdiction, and the trial court dismisses the case outright, the dismissal cannot be a subject of annulment of judgment. Appeal or certiorari under Rule 65, as the case may be, is the proper remedy, or to re-file the case to the appropriate court having jurisdiction over the case or subject matter. The High Court in Paulino vs. CA, , gave emphasis that if the case is dismissed on ground of lack of jurisdiction, the dismissal cannot be a subject of an annulment of judgment. And no need to allege in the petition that the ordinary remedy of new trial or reconsideration of the final order or judgment or appeal therefrom is no longer available through no fault of his own, precisely because the judgment rendered or the final order issued by the RTC without jurisdiction is null and void. The High Court said in Paulino: “The Court finds the petitions devoid of merit. Under Section 2 of Rule 47, the only grounds for annulment of judgment are extrinsic fraud and lack of jurisdiction. Lack of jurisdiction as a ground for annulment of judgment refers to either lack of jurisdiction over the person of the defending party or over the subject matter of the claim. In case of absence, or lack, of jurisdiction, a court should not take cognizance of the case. In these cases, the petition for annulment was based on lack of jurisdiction over the subject matter. The rule is that where there is want of jurisdiction over a subject matter, the judgment is rendered null and void. A void judgment is in legal effect no judgment, by which no rights are divested, from which no right can be obtained, which neither binds nor bars any one, and under which all acts performed and all claims flowing out are void. It is not a decision in contemplation of law and, hence, it can never become executory. It also follows that such a void

judgment cannot constitute a bar to another case by reason of res judicata. Accordingly, the Court agrees with the CA that LRA was not estopped from assailing the July 20, 2011 RTC Decision because it never attained finality for being null and void, having been rendered by a court without jurisdiction over the reconstitution proceedings. As early as the case of Strait Times, Inc. v. CA, the Court has held that when the owner’s duplicate certificate of title has not been lost, but is, in fact, in the possession of another person, then the reconstituted certificate is void, because the court that rendered the decision had no jurisdiction. Reconstitution can be validly made only in case of loss of the original certificate. This rule was reiterated in the cases of Villamayor v. Arante, Rexlon Realty Group, Inc. v. Court of Appeals, Eastworld Motor Industries Corporation v. Skunac Corporation, Rodriguez v. Lim, Villanueva v. Viloria, and Camitan v. Fidelity Investment Corporation. Thus, with evidence that the original copy of the TCT was not lost during the conflagration that hit the Quezon City Hall and that the owner’s duplicate copy of the title was actually in the possession of another, the RTC decision was null and void for lack of jurisdiction. For the aforecited reason, the Court agrees that the public respondent correctly availed of the remedy of petition for annulment of judgment under Rule 47 without need of exhausting other ordinary remedies of new trial, appeal, petition for relief, or other appropriate remedies because the RTC judgment was null and void. Indeed, where a petition for annulment of a judgment or a final order of the RTC filed under Rule 47 of the Rules of Court is grounded on lack of jurisdiction over the person of the respondent or over the nature or subject of the action, the petitioner need not allege in the petition that the

ordinary remedy of new trial or reconsideration of the final order or judgment or appeal therefrom is no longer available through no fault of his own, precisely because the judgment rendered or the final order issued by the RTC without jurisdiction is null and void and may be assailed any time either collaterally or in a direct action or by resisting such judgment or final order in any action or proceeding whenever it is invoked, unless barred by laches.” In Ybañez vs. CA, The High Court dismissed the annulment of judgment case, because what was filed with the CA was an annulment of judgment and not petition for review under Rule 42. So, when the RTC ruled on the issue of validity of the substituted service of summons over the persons of the petitioners in the MTC level, it exercised its appellate jurisdiction, thus, not a ground for annulment of judgment.

H. Denial of due process, as a ground Section 2 of the said Rule provides that the annulment may be based only on the grounds of extrinsic fraud and lack of jurisdiction, although jurisprudence recognizes denial of due process as an additional ground.

I. Cases Demetriou case was permitted by the considered by the High Court as annulment of judgment case but not on extrinsic fraud but on lack of jurisdiction. In Demetriou vs. CA, , petitioners and private respondent became co-owners of the realty brought about by the execution of deeds of sale executed by respondents’ co-owners in favor of the petitioner which at that time the property was then under lease agreement. That time copy of the title was delivered to petitioners, but because of the existing lease agreement, petitioners waited for the lease agreement to expire. At the registry of deeds, upon termination of the lease agreement, petitioners

tried to facilitate the transfer of the title over portions pertaining to them but they failed because the duplicate owner’s copy of the title had already been cancelled by virtue of a court order upon petitioner’s co-owners’ petition for issuance of the second owner’s duplicate title by reason of its loss as it was destroyed by the typhoon. Because the use of a false affidavit of loss does not constitute extrinsic fraud to warrant the invalidation of a final judgment, the CA dismissed the action. However, the High Court reversed the Court of Appeals’ ruling and remanded the case for further proceedings. The High Court said: “On the basis of these allegations the appellate court ruled that the fraud alleged was, if at all, only intrinsic and not extrinsic in character: An action to annul a final judgment on the ground of fraud will lie only if the fraud is extrinsic or collateral in character. Extrinsic fraud refers to any fraudulent act of the prevailing party in the litigation which is committed outside of the trial of the case, whereby the defeated party has been prevented from exhibiting fully his side of the case, by fraud or deception practiced on him by his opponent (Macabingkil vs. People's Homesite and Housing Corporation, 72 SCRA 326 cited in Canlas vs. CA, 164 SCRA 160). On the other hand, intrinsic fraud takes the form of "acts of a party in a litigation during the trial such as the use of forged or false document or perjured testimony, which did not affect the presentation of the case, but did prevent a fair and just determination of the case" (Libudan vs. Gil, 45 SCRA 17). In the present petition, the allegation of fraud involves admission by the respondent court of an alleged false affidavit of loss, which alleged fraud is intrinsic in character. Thus, as the alleged fraud committed by the private respondent is not extrinsic in character, the instant petition for annulment of the said

December 1, 1990 order of the lower court should be dismissed. The appellate court is certainly right in holding that the use of a false affidavit of loss does not constitute extrinsic fraud to warrant the invalidation of a final judgment. The use of the alleged false affidavit of loss by private respondent is similar to the use during trial of forged instruments or perjured testimony. In the leading case of Palanca v. Republic, it was held that the use of a forged instrument constituted only intrinsic fraud for while perhaps it prevented a fair and just determination of a case, the use of such instrument or testimony did not prevent the adverse party from presenting his case fully and fairly. In the case at bar, petitioners were not really kept out of the proceedings because of the fraudulent acts of the private respondent. They could have rebutted or opposed the use of the affidavit and shown its falsity since they were theoretically parties in the case to whom notice had been duly given. But a judgment otherwise final may be annulled not only on the ground of extrinsic fraud but also because of lack of jurisdiction of the court which rendered it. In Serra Sera v. Court of Appeals, on facts analogous to those involved in this case, this Court already held that if a certificate of title has not been lost but is in fact in the possession of another person, the reconstituted title is void and the court rendering the decision has not acquired jurisdiction. Consequently the decision may be attacked any time. Indeed, Rep. Act No. 26, § 18 provides that "in case a certificate of title, considered lost or destroyed be found or recovered, the same shall prevail over the reconstituted certificate of title." It was, therefore, error for the Court of Appeals to dismiss the petition for annulment of judgment of the petitioners.”

J. Withdrawal of the appeal will not give rise to the filing of annulment of judgment Resorting to the filing of a petition for certiorari after the trial court had issued a writ of execution pending appeals and later on withdrew the appeal, the withdrawal of the appeal will not give rise to the filing of annulment of judgment (Philippine Tourism Authority vs. Philippine Golf Development & Equipment,).

K. Taxpayer is a proper party in annulment of judgment In Remulla vs. Maliksi, , the High Court ruled that if the petition for annulment of judgment is filed by a taxpayer who is representing the interests of the province itself, the petition for annulment of judgment should not be dismissed on ground of improper party. The petitioner being a taxpayer who will be either benefited or injured by the execution of the compromise judgment is a real-party-in-interest.

L. Dismissal on ground of specific reason, duty of the court If the dismissal is for specific reasons the dismissal shall be clearly set out so that the High Court will not be at sea if the resolution will be later on assailed, and to make a definitive determination as to whether the CA committed a reversible error in dismissing the petition. The dismissal must contain the details whether the dismissal is on ground of extrinsic fraud or lack of jurisdiction, procedural or substantial (Castigador vs. Nicolas,).

M. Remedy if Court of Appeals denies or grants annulment of judgment If the Court of Appeals denies/dismisses or grants the petition for annulment of judgment, the remedy available to the aggrieved party is to file a Petition for Review on Certiorari under Rule 45 of the Revised Rules of Civil Procedure with High Court (Supreme Court) within fifteen (15)

days from notice of the judgment or final order or resolution appealed from, or of the denial of the petitioner's motion for new trial or reconsideration filed in due time after notice of the judgment (Sections 1 23 and 2 24, Rule 45, 1997 Revised Rules of Civil Procedure). ---0---

---0---

SPECIAL CIVIL ACTIONS Special civil actions are those actions governed by Rules 62, 63, 64, 65, 66, 67, 68 69, 70 and 71. These are the interpleader (Rules 62), declaratory relief and similar remedies (Rules 63), review of judgments and final orders of the Commission on Elections and the Commission on Audit (Rules 64), certiorari, prohibition and mandamus (Rules 65); quo warranto (Rules 66); expropriation (Rules 67); foreclosure of real estate mortgage (Rules 68); partition (Rules 69); forcible entry and unlawful detainer (Rules 70); and contempt (Rules 71).

1. CERTIORARI (RULE 65) 23

Section 1. Filing of petition with Supreme Court. — A party desiring to appeal by certiorari from a judgment or final order or resolution of the Court of Appeals, the Sandiganbayan, the Regional Trial Court or other courts whenever authorized by law, may file with the Supreme Court a verified petition for review on certiorari. The petition shall raise only questions of law which must be distinctly set forth. 24

Section 2. Time for filing; extension. — The petition shall be filed within fifteen (15) days from notice of the judgment or final order or resolution appealed from, or of the denial of the petitioner's motion for new trial or reconsideration filed in due time after notice of the judgment. On motion duly filed and served, with full payment of the docket and other lawful fees and the deposit for costs before the expiration of the reglementary period, the Supreme Court may for justifiable reasons grant an extension of thirty (30) days only within which to file the petition.

A. The Rule Certiorari under Rule 65 is a remedy designed for the correction of errors of jurisdiction and not errors of judgment. An error of judgment is not correctible by certiorari, and as long as the public respondent acted with jurisdiction, any error committed by him or it in the exercise thereof will amount to nothing more than an error of judgment which may be reviewed or corrected only by appeal (Lim vs. Judge Vianzon,). The acceptance of a petition for certiorari, as well as the grant of due course thereto is, in general, addressed to the sound discretion of the court. It must be stressed that certiorari, being an extraordinary remedy, the party who seeks to avail of the same must strictly observe the rules laid down by the law and non-observance thereof may not be brushed aside as mere technicality (Garcia, Jr. vs. CA,). The rule is that “when any tribunal, board or officer exercising judicial or quasi-judicial functions has acted without or in excess its or his jurisdiction, or with grave abuse of discretion amounting to lack or excess of jurisdiction, and there is no appeal, or any plain, speedy, and adequate remedy in the ordinary course of law, a person aggrieved thereby may file a verified petition in the proper court, alleging the facts with certainty and praying that judgment be rendered annulling or modifying the proceedings of such tribunal, board or officer, and granting such incidental reliefs as law and justice may require” (Section 1, Rule 65, 1997 Revised Rules of Civil Procedure). The petition shall be accompanied by a certified true copy of the judgment, order or resolution subject thereof, copies of all pleadings and documents relevant and pertinent thereto, and a sworn certification of nonforum shopping as provided in the third paragraph of section 3, Rule 46 (Paragraph 2, Section 1, Rule 65, 1997 Revised Rules of Civil Procedure). In Lim vs. Judge Vianzon, , the held that the petition is procedurally flawed as it is not accompanied by copies of relevant pleadings mandated by the second paragraph of Section 1, Rule 65 of the 1997 Rules of Civil Procedure.

The High Court said in In Lim: “Moreover, the instant petition is procedurally flawed as it is not accompanied by copies of relevant pleadings mandated by the second paragraph of Section 1, Rule 65 of the 1997 Rules of Civil Procedure. Said provision reads as follows: SECTION 1. Petition for certiorari. – When any tribunal, board or officer exercising judicial or quasi-judicial functions has acted without or in excess of its or his jurisdiction, or with grave abuse of discretion amounting to lack or excess of jurisdiction, and there is no appeal, nor any plain, speedy, and adequate remedy in the ordinary course of law, a person aggrieved thereby may file a verified petition in the proper court, alleging the facts with certainty and praying that judgment be rendered annulling or modifying the proceedings of such tribunal, board or officer, and granting such incidental reliefs as law and justice may require. The petition shall be accompanied by a certified true copy of the judgment, order or resolution subject thereof, copies of all pleadings and documents relevant and pertinent thereto, and a sworn certification of no-forum shopping as provided on the third paragraph of Section 3, Rule 46. (Emphasis supplied.) Specifically, as pointed out by respondents, the instant petition is not accompanied by copies of the Motion to Dismiss and Motion for Reconsideration that petitioners filed with the trial court. These are documents important for the Court’s appraisal, evaluation and judicious disposition of the case. Failing to fully apprise the Court of the relevant details of the case, we find this egregious error a sufficient

cause for the dismissal of the instant petition. As held in Santiago, Jr. v Bautista, to wit: x x x the lower court’s holding that appellant’s failure to accompany his petition with a copy of the judgment or order subject thereof together with copies of all pleadings and documents relevant and pertinent thereto “is fatal to his cause” is supported not only by the provision of that Rule but by precedents as well. A party who seeks to avail of the extraordinary remedy of certiorari must observe the rules laid down by law, and non-observance of the said rules may not be brushed aside as mere technicality.”

B. Petition must be accompanied with clearly legible duplicate original or certified copy of the judgment, order, resolution, or ruling subject What really required is that a petition shall be accompanied with clearly legible duplicate original or certified copy of the judgment, order, resolution, or ruling subject of the petition. Other relevant documents and pleadings attached to it may be mere machine copies thereof is sufficient (Garcia, Jr. vs. CA,).

The High Court in Garcia, Jr: “We dismiss the Petition. The acceptance of a petition for for certiorari, as well as the grant of due course thereto is, in general, addressed to the sound discretion of the court. It must be stressed that

certiorari, being an extraordinary remedy, the party who seeks to avail of the same must strictly observe the rules laid down by the law and non-observance thereof may not be brushed aside as mere technicality. In the matter of the requirement that a petition for certiorari be accompanied by a certified true copy of the judgment, order or resolution subject thereof, , Section 1, Rule 65 of the 1997 Rules of Civil Procedure, as amended, provides: SECTION 1. Petition for certiorari.xxxx The petition shall be accompanied by a certified true copy of the judgment, order or resolution subject thereof, copies of all pleadings and documents relevant and pertinent thereto, and a sworn certification of non-forum shopping as provided in the third paragraph of Section 3, Rule 46. Significantly, Section 3, Rule 46 of the same Rules, provides: SECTION 3. Contents and filing of petition; effect of non-compliance with requirements. – xxxx It shall be filed in seven (7) clearly legible copies together with proof of service thereof on the respondent with the original copy intended for the court indicated as such by the petitioner, and shall be accompanied by a clearly legible duplicate original or certified true copy of the judgment, order,

resolution, or ruling subject thereof, such material portions of the record as are referred to therein, and other document or pertinent thereto. The certification shall be accomplished by the proper clerk of court or by his duly authorized representative, or by the proper officer of the court, tribunal, agency or office involved or by his duly authorized representative. The other requisite number of copies of the petition shall be accompanied by clearly legible plain copies of all documents attached to the original. xxxx The failure of the petitioner to comply with any of the foregoing requirements shall be sufficient ground for the dismissal of the petition.” It is true that Section 3 of Rule 46 does not require that all supporting papers and documents accompanying a petition be duplicate originals or certified true copies. However, it explicitly directs that all cases originally filed in the Court of Appeals shall be accompanied by a clearly legible duplicate original or certified true copy of the judgment, order, resolution or ruling subject thereof. Similarly, under Rule 65, which covers certiorari, prohibition and mandamus, petitions need to be accompanied only by duplicate originals or certified true copies of the questioned judgment, order or resolution. Other releval documents and pleadings attached to it may be mere machine copies thereof. In the case at bar, petitioners failed to attach duplicate originals or certified true copies of the assailed Orders of the RTC, dated 22 February 2005 and 7 April 2005. What they affixed were machine or Xerox copies of the same. Plainly put, petitioners contravened the obvious rudiments of the rules.

In Circular No. 3-96, we made the following clarifications and supplemental rules on what is a duplicate original or certified true copy: 1. The "duplicate original copy shall" be understood to be that copy of the decision, judgment, resolution or order which is intended for and furnished to a party in the case or proceeding in the court or adjudicative body which rendered and issued the same. The "certified true copy" thereof shall be such other copy furnished to a party at his instance or in his behalf, duly authenticated by the authorized officers or representatives of the issuing entity as hereinbefore specified. 2. The duplicate original copy must be duly signed or initialed by the authorities or the corresponding officer or representative of the issuing entity, or shall at least bear the dry seal thereof or any other official indication of the authenticity and completeness of such copy. For this purpose, all courts, offices or agencies furnishing such copies which may be used in accordance with Paragraph (3) of Revised Circular No. 1-88 shall make arrangements for and designate the personnel who shall be charged with the implementation of this requirement. 3. The certified true copy must further comply with all the regulations therefor of the issuing entity and it is the authenticated original of such certified true copy, and not a mere Xerox copy thereof, which shall be utilized as an annex to the petition or other initiatory pleading. 4. Regardless of whether a duplicate original copy or a certified true copy of the adjudicatory document is annexed to the petition or initiatory pleading, the same must be exact and complete copy of the original and all the pages thereof must be clearly legible and printed on white bond or equivalent paper of good quality with the same dimensions as the original copy. Either of the aforesaid copies shall be annexed to the original copy of the petition

or initiatory pleading filed in court, while plain copies thereof may be attached to the other copies of the pleading. 5. It shall be the duty and responsibility of the party using documents required by Paragraph (3) of Circular No. 1-88 to verify and ensure compliance with all the requirements thereof as detailed in the proceeding paragraphs. Failure to do so shall result in the rejection of such annexes and the dismissal of the case. Subsequent compliance shall not warrant any reconsideration unless the court is fully satisfied that the noncompliance was not in any way attributable to the party despite due diligence on his part, and that there are highly justifiable and compelling reasons for the court to make such other disposition as it may deem just and equitable. Based on the foregoing, it is incontrovertible that a certified true copy is not a mere Xerox copy. Further, it is imperative that the duplicate original copyu required by the rules must be duly signed or initialed by the authorities or the corresponding officer or representative of the issuing entity, or shall at least bear the dry seal thereof or any other official indication of the authenticity and completeness of such copy. Petitioners’ Xerox copies are wanting in this respect. Petitioners seek a liberal application of the procedural rules. For their failure to attach certified true copies of the assailed orders of the RTC, petitioners place the blame on the appellate court. Petitioners brazenly suggest that what the Court of Appeals should have done was to issue an Order directing them to comply with the rule on attaching certified true copies, instead of dismissing the case on its face. We do not see reason to grant liberality in the application of the rules. It must be emphasized that the liberality in the interpretation and application of the rules applies only in proper cases and under justifiable causes and circumstances. While it is true that litigation is not a game of technicalities, it is equally tue that every case must

be prosecuted in accordance with the prescribed procedure to insure an orderly and speedy administration of justice. Only strong considerations of equity, which are wanting in this case, will lead us to allow an exception to the procedural rule in the interest of substantial justice. To further suggest petitioners’ impervious attitude towards rules, they even failed to attach certified true copies or duplicate original copies of the assailed Orders in their Motion for Reconsideration filed with the Court of Appeals. Concomitant to a liberal application of the rules of procedure should be an effort on the party invoking liberality to at least explain its failure to comply with the rules. Circular No. 3-96 is also unequivocal that it shall be the duty and responsibility of the party to verify and ensure compliance with all the requirements detailed therein. In fact, failure to do so shall result in the rejection of such annexes and the dismissal of the case.” Take note the requirement provided in A.M. No. 10-3-7-SC [Rules on e-Filing] and A.M. No. 10-3-7-SC [Rule for the Efficient Use of Paper] dated September 10, 2013.

A. The assessment of treble costs provided in Section 8 of Rule 65, 1997 Revised Rules of Civil Procedure, as amended by A.M. No. 07-7-12-SC is not automatic or mandatory After the comment or other pleadings required by the court are filed, or the time for the filing thereof has expired, the court may hear the case or require the parties to submit memoranda. If, after such hearing or filing of memoranda or upon the expiration of the period for filing, the court finds that the allegations of the petition are true, it shall render judgment for such relief to which the petitioner is entitled. However, the court may dismiss the petition if it finds the same patently without merit or prosecuted manifestly for delay, or if the questions raised therein are too unsubstantial to require consideration. In such event, the court may award in favor of the respondent treble costs solidarily against the petitioner and counsel, in addition to subjecting counsel to administrative sanctions

under Rule 139 and 139-B of the Rules of Court (First paragraph, Section 8 of Rule 65, as amended by A.M. No. 07-7-12-SC). The Court may impose motu proprio, based on res ipso loquitur, other disciplinary sanctions or measures on erring lawyers for patently dilatory and unmeritorious petitions for certiorari (Second paragraph, Section 8 of Rule 65, as amended by A.M. No. 07-7-12-SC). The use of the word "may" in the last sentence of the second paragraph or Section 8, Rule 65, indicates that the assessment of treble costs is not automatic or mandatory. It merely gives the court the discretion and latitude to impose further sanctions where a petition is dismissed for being "patently without merit," "prosecuted manifestly for delay," or upon finding that the questions raised in the petition for certiorari were "too to require consideration" (City of Davao vs. CA, G.R. No. 200538, Augsut 13, 2014). The court is afforded judicial discretion in imposing treble costs, there remains a need to show that it is sound and with basis - that is "taking all the pertinent circumstances into due consicleration.'' (Diaz vs. People, 180677,).

B. When is treble cost imposable or awarded? The parties and their counsels resort to deplorable dilatory tactics to frustrate the fruition of justice (City of Davao vs. CA,). Examples: 1. When the losing litigant repeatedly frustrated the execution of a final and executory decision (Central Surety and Insurance Company vs. Planters Product, Inc, 546 Phil. 479 cited in City of Davao vs. CA, G.R. No. 200538, August 13, 2014). In Central Surety and Insurance Company vs. Planters Product, Inc,), the execution was delayed for more than five years because of his dilatory tactics. When the winning party sought the execution by motion

beyond the period, he still opposed it despite the fact that the period was suspended because of reasons attributable to him. 2. Treble costs were awarded because of the deplorable course resorted to by the losing litigants in the hope of evading manifest obligations. The Court stated that it viewed with disfavor the unjustified delay in the enforcen1ent or the final decision and orders in the said case. Once a judgment becomes tinal and executory, the prevailing party should not be denied the fruits of his victory by some subterfuge devised by the losing party. Unjustified delay in the enforcement of a judgment sets at naught the role of courts in disposing justiciable controversies with finality (Spouses Aguilar vs. The Manila Banking Corporation,). 3. Treble costs were imposed because the parties took the law into their own hands or resorted to a wrong remedy (Uypitching vs. Quiamco,). In Uypitching vs. Quiamco, , instead of bringing the proper civil action necessary to acquire legal possession of a motorcycle, the petitioner took the law into his own hands and seized it without a search warrant or court order. Worse, in the course or the illegal seizure or the motorcycle, the petitioner even mouthed a slanderous statement. By doing so, he transgressed the proper norms of human relations. 4. In Solen vs. A-1 Investors., the party claimed that he was denied due process when the court adjudged a case against him even if he was not served the summons. He then resorted to filing a complaint for injunction to stop the execution of the final judgment. The Court said that he should have resorted to an action for annulment under Rule 47 and so awarded treble costs against him. In City of Davao vs. CA, G, the imposition of treble costs was striken off by the High Court for failure of the Court of Appeals to explain and/or to give any reason for such imposition.

C. Timeliness of the petition for certiorari

The petition shall be filed not later than sixty (60) days from notice of judgment, order or resolution. In case a motion for reconsideration or new trial is timely filed, whether such motion is required or not, the sixty (60) day period shall be counted from notice of the denial of said motion (Section 4, Rule 65, 1997 Revised Rules of Civil Procedure). The 60-day period starts to run from the date petitioner receives the assailed judgment, final order or resolution, or the denial of the motion for reconsideration or new trial timely filed, whether such motion is required or not (Vinuya vs. Secretary Romulo,). To establish the timeliness of the petition for certiorari, the date of receipt of the assailed judgment, final order or resolution or the denial of the motion for reconsideration or new trial must be stated in the petition; otherwise, the petition for certiorari must be dismissed. The importance of the dates cannot be understated, for such dates determine the timeliness of the filing of the petition for certiorari (Vinuya vs. Secretary Romulo).

D. Three (3) essential dates that must be stated in a petition for certiorari brought under Rule 65 First, the date when notice of the judgment or final order or resolution was received; second, when a motion for new trial or reconsideration was filed; and third, when notice of the denial thereof was received. Failure of petitioner to comply with this requirement shall be sufficient ground for the dismissal of the petition. Substantial compliance will not suffice in a matter involving strict observance with the Rules (Tambong vs. R. Jorge Development Corporation). The requirement of setting forth the three (3) dates in a petition for certiorari under Rule 65 is for the purpose of determining its timeliness. Such a petition is required to be filed not later than sixty (60) days from notice of the judgment, order or Resolution sought to be assailed. Therefore, that the petition for certiorari was filed forty-one (41) days from receipt of the denial of the motion for reconsideration is hardly relevant. The Court of Appeals was not in any position to determine when this period commenced to run and whether the motion for reconsideration itself was filed on time since the material dates were not stated. It should

not be assumed that in no event would the motion be filed later than fifteen (15) days. Technical rules of procedure are not designed to frustrate the ends of justice. These are provided to effect the proper and orderly disposition of cases and thus effectively prevent the clogging of court dockets. Utter disregard of the Rules cannot justly be rationalized by harking on the policy of liberal construction (Santos vs. CA,).

E. Motion for reconsideration is required before filing a petition for certiorari under Rule 65 in the Higher Court; exception Petition for certiorari before a Higher Court will not prosper unless the inferior court has been given, through a motion for reconsideration, a chance to correct the errors imputed to it. EXCEPTIONS: (1) when the issue raised is purely of law; (2) when public interest is involved, or (3) in case of urgency (Philippine International Trading Corporation vs. COA,).

F. Order denying the demurrer to evidence reviewable by Certiorari under Section 1, Rule 65 of the Rules of Court Generally, the order denying the motion for leave of court to file demurrer to evidence or the demurrer itself shall not be reviewable by appeal or by certiorari before judgment,25 because action on a demurrer or on a motion to dismiss rests on the sound exercise of judicial discretion. 26 It is an interlocutory order, not appealable neither can it be the subject of a petition for certiorari.27 However, this admits an exception: when the denial of a demurrer to evidence is attended by grave abuse of discretion, patently erroneous or 25

Section 23, Rule 119, 2000 Revised Rules of Criminal Procedure. Tan vs. Court of Appeals, 347 Phil. 320, 329 (1997); Bernardo vs. Court of Appeals, 344 Phil. 335, 346 (1997) cited in Nicolas vs. Sandiganbayan and the companion case, G.R. Nos. 175930-31, February 11, 2008. Also in People vs. Singh, G.R. No. 129782, June 29, 2001, 360 SCRA 404; People vs. Mercado, No. L-33492, March 30, 1988, 159 SCRA 453 cited in People vs. Almendras, G.R. No. 145915, April 24, 2003. 27 David vs. Rivera, 464 Phil. 1006; Tadeo vs. People, 360 Phil. 914, 919 (1998); Cruz vs. People, 363 Phil. 156; Katigbak vs. Sandiganbayan, 453 Phil. 515 cited in Nicolas vs. Sandiganbayan and the companion case, G.R. Nos. 175930-31, February 11, 2008. 26

issued with grave abuse of discretion.28 Though interlocutory in character, an order denying a demurrer to evidence may be the subject of a certiorari proceeding, provided the petitioner can show that it was issued with grave abuse of discretion; and that appeal in due course is not plain, adequate or speedy under the circumstances. When the plaintiff’s evidence is utterly and patently insufficient to prove the complaint, it would be capricious for a trial judge to deny the demurrer and to require the defendant to present evidence to controvert a non- existing case. The denial of the demurrer to evidence will constitute an unwelcome imposition on the court’s docket and an assault on the defendant’s resources and peace of mind, and if denied, it effectively denies justice.29 “Interlocutory” refers to intervening developments between the commencement of a suit and its complete termination; hence, it is a development that does not end the whole controversy. An “interlocutory order” merely rules on an incidental issue and does not terminate or finally dispose of the case; it leaves something to be done before the case is finally decided on the merits (Marmot vs. Anacay) . G. A one liner sentence denying a motion for reconsideration to the final order or judgment for lack of merit is not certiorariable (Rule 65) A one liner sentence denying a motion for reconsideration to the final order or judgment for lack of merit is not certiorariable. The filing of a motion to amend order denying the reconsideration on ground of null and void as it does not conform to the requirements of Section 14, Article VIII of the Constitution and Section 1, Rule 36 of the 1997 Revised Rules of Civil Procedure will not and does not toll the running of the period towards finality. The aggrieved party may avail any of the following remedies: 1. He may file a Motion for New Trial under Rule 37; or

28

People vs. Ong, G.R. No. 140904, October 9, 2000 citing Cruz vs. People (303 SCRA 533 [1999]). 29 Choa vs. Choa, G.R. No. 143376, November 26, 2002.

2. He may file a Motion for Reconsideration, also under Rule 37; or 3. He may file an Appeal under Rule 41; or 4. He may file a petition for Relief from Judgment under Rule 38, or; 5. He may file a petition for Annulment of Judgment under Rule 47. A decision that has acquired finality becomes immutable and unalterable and may no longer be modified in any respect even if the modification is intended to correct erroneous conclusions of fact or law and whether it will be made by the court that rendered it or by the highest court of the land.30 0-0-0

APPEAL (RULE 41) VS. CERTIORARI (RULE 65) “An appeal may be taken from a judgment or final order that completely disposes of the case or of a particular matter therein when declared by these Rules to be appealable” (Section 1, Rule 41, 1997 Revised Rules of Civil Procedure).

“When any tribunal, board or officer exercising judicial or quasi-judicial functions has acted without or in excess of jurisdiction, or with grave abuse of discretion

30

Sofio vs. Valenzuela, G.R. No. 157810, February 15, 2012.

amounting to lack or excess of jurisdiction, and there is no appeal, or any plain, speedy and adequate remedy in the ordinary course of law, a person aggrieved thereby may file a verified petition in the proper court, alleging the facts with certainty and praying that judgment be rendered annulling or modifying the proceedings of such tribunal, board or officer, and granting such incidental reliefs as law and justice may require” (Section 1, Rule 65, 1997 Revised Rules of Civil Procedure). Tible & Tible Company, Inc. vs. Royal Savings and Loan Association, G.R. No. 155806, April 8, 2008 citing Madrigal Transport, Inc. vs. Lapanday Holdings Corporation, G.R. No. 156067, August 11, 2004 d discussed the differences between the remedies provided under Rule 41 and Rule 65, thus: “Appeal and Certiorari Distinguished Between an appeal and a petition for certiorari, there are substantial distinctions which shall be explained below. As to the Purpose. Certiorari is a remedy designed for the correction of errors of jurisdiction, not errors of judgment. In Pure Foods Corporation v. NLRC, we explained the simple reason for the rule in this light: “When a court exercises its jurisdiction, an error committed while so engaged does not deprive it of the jurisdiction being exercised when the error is committed. If it did, every error committed by a court would deprive it of its jurisdiction and every erroneous judgment would be a void judgment. This cannot be allowed. The administration of justice would not survive such a rule. Consequently, an error of judgment that the court may commit in the exercise of its jurisdiction is not correctable through the original civil action of certiorari.”

The supervisory jurisdiction of a court over the issuance of a writ of certiorari cannot be exercised for the purpose of reviewing the intrinsic correctness of a judgment of the lower court – on the basis either of the law or the facts of the case, or of the wisdom or legal soundness of the decision. Even if the findings of the court are incorrect, as long as it has jurisdiction over the case, such correction is normally beyond the province of certiorari. Where the error is not one of jurisdiction, but of an error of law or fact – a mistake of judgment – appeal is the remedy. As to the Manner of Filing. Over an appeal, the CA exercises its appellate jurisdiction and power of review. Over a certiorari, the higher court uses its original jurisdiction in accordance with its power of control and supervision over the proceedings of lower courts. An appeal is thus a continuation of the original suit, while a petition for certiorari is an original and independent action that was not part of the trial that had resulted in the rendition of the judgment or order complained of. The parties to an appeal are the original parties to the action. In contrast, the parties to a petition for certiorari are the aggrieved party (who thereby becomes the petitioner) against the lower court or quasi-judicial agency, and the prevailing parties (the public and the private respondents, respectively). As to the Subject Matter. Only judgments or final orders and those that the Rules of Court so declare are appealable. Since the issue is jurisdiction, an original action for certiorari may be directed against an interlocutory order of the lower court prior to an appeal from the judgment; or where there is no appeal or any plain, speedy or adequate remedy. As to the Period of Filing. Ordinary appeals should be filed within fifteen days from the notice of judgment or final order appealed from. Where a record on appeal is required, the appellant must file a notice of appeal and a

record on appeal within thirty days from the said notice of judgment or final order. A petition for review should be filed and served within fifteen days from the notice of denial of the decision, or of the petitioner’s timely filed motion for new trial or motion for reconsideration. In an appeal by certiorari, the petition should be filed also within fifteen days from the notice of judgment or final order, or of the denial of the petitioner’s motion for new trial or motion for reconsideration. On the other hand, a petition for certiorari should be filed not later than sixty days from the notice of judgment, order, or resolution. If a motion for new trial or motion for reconsideration was timely filed, the period shall be counted from the denial of the motion. As to the Need for a Motion for Reconsideration. A motion for reconsideration is generally required prior to the filing of a petition for certiorari, in order to afford the tribunal an opportunity to correct the alleged errors. Note also that this motion is a plain and adequate remedy expressly available under the law. Such motion is not required before appealing a judgment or final order.” With these distinctions, it is plainly discernible why a party is precluded from filing a petition for certiorari when appeal is available, or why the two remedies of appeal and certiorari are mutually exclusive and not alternative or successive. Where appeal is available, certiorari will not prosper, even if the ground availed of is grave abuse of discretion.” ---0---

2. PROHIBITION A. The Rule

When the proceedings of any tribunal, corporation, board, officer or person, whether exercising judicial, quasi-judicial or ministerial functions, are without or in excess of its or his jurisdiction, or with grave abuse of discretion amounting to lack or excess of jurisdiction, and there is no appeal or any other plain, speedy, and adequate remedy in the ordinary course of law, a person aggrieved thereby may file a verified petition in the proper court, alleging the facts with certainty and praying that judgment be rendered commanding the respondent to desist from further proceedings in the action or matter specified therein, or otherwise granting such incidental reliefs as law and justice may require (Section 2, Rule 65, 1997 Revised Rules of Civil Procedure). The petition shall likewise be accompanied by a certified true copy of the judgment, order or resolution subject thereof, copies of all pleadings and documents relevant and pertinent thereto, and a sworn certification of non-forum shopping as provided in the third paragraph of section 3, Rule 46 (Paragraph 2, Section 2, Rule 65, 1997 Revised Rules of Civil Procedure).

B. Writ of prohibition an extraordinary writ A writ of prohibition is an extraordinary writ. It may be issued only in the absence of a plain, speedy and adequate remedy in the ordinary course of law Purok Bagong Silang Association, Inc. vs. Judge Yuipco,).

C. General rule, a writ of prohibition cannot be issued against an inferior court, exception A writ of prohibition will not be issued against an inferior court unless the attention of the court whose proceedings are sought to be stayed has been called to the alleged lack or excess of jurisdiction. The foundation of this rule is the respect and consideration due to the lower court and the expediency of preventing unnecessary litigation; it cannot be presumed that the lower court would not properly rule on a jurisdictional

objection if it were property presented to it (Esquivel vs. Hon. Ombudsman,).

D. Prohibition lies against judicial or ministerial functions, but not against legislative or quasi-legislative functions Prohibition lies against judicial or ministerial functions, but not against legislative or quasi-legislative functions. Generally, the purpose of a writ of prohibition is to keep a lower court within the limits of its jurisdiction in order to maintain the administration of justice in orderly channels. Prohibition is the proper remedy to afford relief against usurpation of jurisdiction or power by an inferior court, or when, in the exercise of jurisdiction in handling matters clearly within its cognizance the inferior court transgresses the bounds prescribed to it by the law, or where there is no adequate remedy available in the ordinary course of law by which such relief can be obtained. Where the principal relief sought is to invalidate an IRR, petitioners’ remedy is an ordinary action for its nullification, an action which properly falls under the jurisdiction of the Regional Trial Court. In any case, petitioners’ allegation that “respondents are performing or threatening to perform functions without or in excess of their jurisdiction” may appropriately be enjoined by the trial court through a writ of injunction or a temporary restraining order (Holy Spirit Homeowners’ Association vs. Defensor,). E. Prohibition does not lie to restrain an act which is already a fait accompli Prohibition does not lie to restrain an act which is already a fait accompli. The “operative fact” doctrine protecting vested rights bars the grant of the writ of prohibition (Chavez vs. NHA,).

F. Petition for prohibition venue of action The petition for prohibition filed by respondents is a special civil action which may be filed in the Supreme Court, the Court of Appeals, the Sandiganbayan or the regional trial court, as the case may be. It is also a

personal action because it does not affect the title to, or possession of real property, or interest therein. Thus, it may be commenced and tried where the plaintiff or any of the principal plaintiffs resides, or where the defendant or any of the principal defendants resides, at the election of the plaintiff. The choice of venue is sanctioned by Section 2, Rule 4 of the Rules of Court (GSIS Board of Trustees vs. Velasco,). ---0---

1. MANDAMUS A. The Rule When any tribunal, corporation, board, officer or person unlawfully neglects the performance of an act which the law specifically enjoins as a duty resulting from an office, trust, or station, or unlawfully excludes another from the use and enjoyment of a right or office to which such other is entitled, and there is no other plain, speedy and adequate remedy in the ordinary course of law, the person aggrieved thereby may file a verified petition in the proper court, alleging the facts with certainty and praying that judgment be rendered commanding the respondent, immediately or at some other time to be specified by the court, to do the act required to be done to protect the rights of the petitioner, and to pay the damages sustained by the petitioner by reason of the wrongful acts of the respondent (Section 3, Rule 65, 1997 Revised Rules of Civil Procedure). The petition shall also contain a sworn certification of non-forum shopping as provided in the third paragraph of section 3, Rule 46 (Paragraph 2, Section 3, Rule 65, 1997 Revised Rules of Civil Procedure).

B. Concept of ministerial duty A ministerial duty is one which is so clear and specific as to leave no room for the exercise of discretion in its performance. It is a duty which an officer performs in a given state of facts in a prescribed manner in obedience to the mandate of legal authority, without regard to the

exercise of his/her own judgment upon the propriety of the act done (Symaco v. Hon. Aquino,). Mandamus lies to compel the performance, when refused, of a ministerial duty, but not to compel the performance of a discretionary duty. A purely ministerial act or duty is one which an officer or tribunal performs in a given state of facts, in a prescribed manner, in obedience to the mandate of a legal authority, without regard to or the exercise of his own judgment upon the propriety or impropriety of the act done. The duty is ministerial only when the discharge of the same requires neither the exercise of official discretion or judgment. When an official is required and authorized to do a prescribed act upon a prescribed contingency, his functions are ministerial only, and mandamus may be issued to control his action upon the happening of the contingency (Paloma vs. Mora,). The remedy of mandamus is available only to compel the performance of a ministerial duty. The distinction between a ministerial and discretionary act is well delineated. A purely ministerial act or duty is one which an officer or tribunal performs in a given state of facts, in a prescribed manner, in obedience to the mandate of a legal authority, without regard to or the exercise of his own judgment upon the propriety or impropriety of the act done. If the law imposes a duty upon a public officer and gives him the right to decide how or when the duty shall be performed, such duty is discretionary and not ministerial. The duty is ministerial only when the discharge of the same requires neither the exercise of official discretion or judgment (Torregoza vs. CSC,). For a writ of mandamus to be issued, it is essential that petitioner should have a clear legal right to the thing demanded and it must be the imperative duty of the respondent to perform the act required. The writ neither confers powers nor imposes duties. It is simply a command to exercise a power already possessed and to perform a duty already imposed. Mandamus applies as a remedy only where petitioner’s right is founded clearly in law and not when it is doubtful. The writ will not be granted where its issuance would be unavailing, nugatory, or useless (Philippine Coconut Authority vs. Primex Coco Products). If the law imposes a duty upon a public officer and gives him the right to decide how or when the duty shall be performed, such duty is

discretionary and not ministerial (Philippine Coconut Authority vs. Primex Coco Products, Inc,). Mandamus is an extraordinary writ and discretionary remedy and should not be granted when it will achieve no beneficial result such as when act sought to be compelled has been performed (Philippine Coconut Authority vs. Primex Coco Products,). Mandamus will not be issued to compel the renewal of a license for a period which has expired. If the right sought to be enforced by writ of mandamus is or has become a mere abstract right, enforcement of which will be of no substantial or practical benefit to the plaintiff, the writ will not issue though the applicant would otherwise be entitled to it. To warrant the issuance of a writ of mandamus, it must appear that the writ will be effectual as a remedy, it should be denied where it would be useless by reason of events occurring subsequent to commencement proceedings (Philippine Coconut Authority vs. Primex Coco Products, Inc,). Mandamus is never granted to compel the performance of an act until there has been an actual, as distinguished from an anticipated, refusal to act. This is true even if there is a strong presumption that the persons whom it is sought to coerce by the writ will refuse to perform their duty when the proper time arrives. Its function is to compel the performance of a present existing duty as to which there is default. It is not granted to take effect prospectively, and it contemplates the performance of an act which is incumbent on respondent when the application for a writ is made (Philippine Coconut Authority vs. Primex Coco Products,). ---0---

FINALITY OF JUDGMENT AND ENTRY OF JUDGMENT A. Doctrine of finality of judgment The doctrine of finality of judgments is grounded on fundamental considerations of public policy and sound practice that at the risk of

occasional errors, the judgments of courts must become final at some definite date fixed by law (Torno vs. IAC, ). After the lapse of the 15-day reglementary period to appeal, an order becomes executory, and it goes beyond the jurisdiction of the court that rendered it to further amend or revoke. The subsequent filing of a Motion for Reconsideration cannot disturb the finality of a judgment, nor restore jurisdiction that had already been lost. A final and executor judgment or order cannot be modified in any respect, even if the modification sought is to correct an erroneous conclusion by the court that rendered it (Tupaz vs. Judge Apurillo,).

B. Date of entry of judgment, determinative of 1. for purposes of ensuring the finality of judgment for execution purposes; 2. for purposes of determining the period to file petition for relief from judgment; and 3. for purposes of determining the period to file action for revival of judgment. The 60-day period must be counted after petitioner learns of the judgment or final order, and the 6-month period is counted from the finality of judgment or final order. These are the so-called double period. These double periods are both inextendible and uninterruptible; is jurisdictional and should be strictly complied with (Madarang vs. Morales,). The 60-day reglementary period shall reckone from the time the party’s counsel receives notice of the decision for notice to counsel of the decision is notice to the party for purposes of Section 3 of Rule 38 (Mercury Drugs Corporation vs. CA,). ---0---

LIBERAL INTERPRETATION OF THE RULES

A. The Rule The 1997 Revised Rules of Civil Procedure shall be liberally construed in order to promote their objective of securing a just, speedy and inexpensive disposition of every action and proceeding (Section 6, Rule 1, 1997 Revised Rules of Civil Procedure).

B. Liberal interpretation on right to appeal The right to appeal is not a natural right and is not part of due process, but merely a statutory privilege to be exercised only in accordance with the law. Being the party who sought to appeal, he must comply with the requirements of the relevant rules; otherwise, he would lose the statutory right to appeal (Magsino vs. Ocampo,). The procedures regulating appeals as laid down in the Rules of Court must be followed because strict compliance with them is indispensable for the orderly and speedy disposition of justice (Polintan vs. People,). For appealed cases, the High Court in Galvez vs. CA, , recognized three (3) guideposts for the Court of Appeals to consider in determining whether or not the rules of procedures should be relaxed: “First, not all pleadings and parts of case records are required to be attached to the petition. Only those which are relevant and pertinent must accompany it. The test of relevancy is whether the document in question will support the material allegations in the petition, whether said document will make out a prima facie case of grave abuse of discretion as to convince the court to give due course to the petition. Second, even if a document is relevant and pertinent to the petition, it need not be appended if it is shown that the contents thereof can also found in another document already attached to the petition. Thus, if the material allegations in a position paper are summarized in a

questioned judgment, it will suffice that only a certified true copy of the judgment is attached. Third, a petition lacking an essential pleading or part of the case record may still be given due course or reinstated (if earlier dismissed) upon showing that petitioner later submitted the documents required, or that it will serve the higher interest of justice that the case be decided on the merits.”

C. Liberal interpretation on strict adherence to procedural rules The following are the justification for a court to suspend a strict adherence to procedural rules: (a) matters of life, liberty, honor or property in involved; (b) the existence of special or compelling circumstances; (c) the merits of the case; (d) a cause not entirely attributable to the fault or negligence of the party favored by the suspension of the rules; (e) a lack of any showing that the review sought is merely frivolous and dilatory; and (f) the fact that the other party will not be unjustly prejudiced thereby (Sanchez vs. CA,). ---0---

View more...

Comments

Copyright ©2017 KUPDF Inc.
SUPPORT KUPDF